State Bank of India PO (Pre.) Exam-2016 Held on 2-7-2016 Numerical Ability Question Paper With Answer Key

State Bank of India PO (Pre.) Exam-2016 Held on 2-7-2016
State Bank of India PO (Pre.) Exam-2016 Held on 2-7-2016 Numerical Ability Question Paper With Answer Key

State Bank of India PO (Pre.) Exam-2016 Held on 2-7-2016

Numerical Ability

Directions- (Q. 1 to 5) In the Bar-chart, total members enrolled in different years from 1990 to 1994 in gymnasium A and B. Based on this Bar-chart solve the following questions.

1. In the year 1995, 30% increase in total number of members enrolled in 1994 of both gymnasium find the total no. of members enrolled in 1995?

(A)  282

(B)  296

(C)  292

(D)  286

(E)  None of these

Answer: (D)

2. The ratio between total members of both gymnasium in 1991 to total members in 1994 of both gymnasium is-

(A)  22 : 27

(B)  21 : 11

(C)  11 : 21

(D)  25 : 13

(E)  27 : 22

Answer: (E)

3. The number of members of gymnasium A in 1991 is what % of the number of members of gymnasium B in 1994-

(A)  50%

(B)  55%

(C)  58%

(D)  62%

(E)  None of these

Answer: (E)

4. The total number of members enrolled in gymnasium A from 1991 to 1994 together is what per cent more than the total number of members enrolled in gymnasium B in 1993 and 1994 together ? (Rounded off to 2 decimal places)

(A)  10351%

(B)  20.51%

(C)  15.51%

(D)  17.51%

(E)  None of these

Answer: (B)

5. Total member enrolled in gymnasium B in 1993 and 1994 together is what % more than members enrolled in gymnasium A in 1990 and 1994 together?

(A)  60%

(B)  65%

(C)  62.5%

(D)  61.5%

(E)  None of these

Answer: (C)

Directions- (Q. 6-10) What should come in place of question mark  (?) in the following number series?

6. 4 3  4  7 15  ?

(A)  38.5

(B)  40

(C)  45

(D)  37.5

(E)  None of these

Answer: (A)

7. 7 5  7  17  63  ?

(A)  321

(B)  309

(C)  305

(D)  301

(E)  None of these

Answer: (B)

8. 11 14  19  28  43  ?

(A)  60

(B)  63

(C)  66

(D)  70

(E)  None of these

Answer: (C)

9. 2 60  10  120  30  ?

(A)  222

(B)  216

(C)  208

(D)  230

(E)  None of these

Answer: (E)

10. 23 50  108  232  492  ?

(A)  1028

(B)  1024

(C)  1010

(D)  1032

(E)  None of these 

Answer: (A)

Directions- (Q. 11-15) There are five companies and we have given the no. of employees working in different companies, in the table we have also given the percentage of male and female employees HR and Marketing department.

11. If 60% of the employees of company T in HR department have MBA degree and 40% of the employees of the same company in the Marketing dept., have MBA degree, then how many employees have MBA degree in company T in both depts., Together?

(A)  98

(B)  108

(C)  106

(D)  92

(E)  66

Answer: (E)

12. The ratio of female employee of company Q in HR dept., to male employee of company R in Marketing dept.?

(A)  4 : 13

(B)  5 : 22

(C)  22 : 5

(D)  13 : 4

(E)  None of these

Answer: (D)

13. Total number of HR employees of company P is what % more than the total no. of marketing employee in company?

(A)  236.76%

(B)  226.67%

(C)  276.76%

(D)  246.67%

(E)  None of these

Answer: (D)

14. The ratio of male employees in HR dept., of company P and R together to female employees of Marketing department in company S and T together?

(A)  187 : 27

(B)  43 : 188

(C)  188 : 43

(D)  27 : 187

(E)  None of these

Answer: (C)

15. Difference between female employees of HR dept., in all companies together (excluding company S) and the female employees of Marketing dept., in all companies together (excluding company Q) ?

(A)  139

(B)  129

(C)  135

(D)  141

(E)  None of these

Answer: (A)

16. A mixture contains wine and water in the ratio 3 : 2 and another mixture contains them in the ratio 4 : 5. How many litres of the latter must be mixed with 3 litres of the former so that the resultant mixture may contain equal quantities of wine and water?

(A)    

(B)   

(C)   

(D)    

(E)  None of these

Answer: (E)

17. A trader sells two bullocks for Rs 8,400 each, neither losing nor gaining in total, if he said one of the bullocks at a gain of 20%, the other’s sold at a loss of-

(A)  20%

(B)    

(C)    

(D)  21%

(E)  None of these

Answer: (C)

18. Two trains, A and B, start from stations X and Y towards each other they take 4 hours 48 minutes and 3 hours 20 minutes to reach Y and X respectively after they meet if train A is 45 km/hr then the speed of the train B-

(A)  60 km/hr

(B)  64.8 km/hr

(C)  54 km/hr

(D)  37.5 km/hr

(E)  None of these

Answer: (C)

19. Out of his total income, Mr. Kapoor spends 20% on house rent and 70% of the rest on home expenses. If he saves Rs 1,800, what is his total income (in rupees)?

(A)  Rs 7,800

(B)  Rs 7,000

(C)  Rs 8,000

(D)  Rs 7,500

(E)  None of these

Answer: (D)

20. A can do a piece of work in 8 days which B can destroy in 3 days. A has worked for 6 days since last 2 days of which B has been destroying. How many days must a now work alone to complete the remaining work?

(A)  7 days

(B)   

(C)   

(D)  8 days

(E)  None of these

Answer: (C)

Directions- (Q. 21-25) What approximate value should come in place of the question mark in the following questions ? (You are not expected to calculate the exact value).

21. 57% of 394 – 2.5% of 996 = ?

(A)  215

(B)  175

(C)  200

(D)  180

(E)  205

Answer: (C)

22. 96.996 × 9.669 + 0.96 = ?

(A)  860

(B)  870

(C)  1020

(D)  940

(E)  1100

Answer: (D)

23. 3/5 × 1125/1228 × 7 = ?

(A)  7

(B)  12

(C)  9

(D)  14

(E)  15

Answer: (D)

24. (√339 × 25) ÷ 30 = ?

(A)  12

(B)  15

(C)  24

(D)  21

(E)  9

Answer: (B)

25. (638 + 9709 – 216) ÷ 26 = ?

(A)  275

(B)  365

(C)  420

(D)  300

(E)  390

Answer: (E)

Directions – (Q. 26-30) Solve the equations given below and answer, if-

(A) x < y

(B) x > y

(C) x ≥ y

(D) x ≤ y

(E) or no relation can be established

26. 6x2 + 31x + 35 = 0

2y2 + 3y + 1 =0

Answer: (A)

27. 2x2 – (4 + √13) x + 2√13 = 0

10y2 – (18 + 5√13) y + 9.√13 = 0

Answer: (C)

28. 2x2 + 9x + 10 = 0

4y2 + 28y + 45 = 0

Answer: (C)

29. 15x2 – 11x – 12 = 0

20y2 – 49y + 30 = 0

Answer: (E)

30. 2x2 – 15 = 7x

17y = −7 – 6y2

Answer: (E)

31. A and B are partners in a business. They invest in the ratio 5 : 6, at the end of 8 months A withdraw. If they receive profits in the ratio of 5 : 9, find how long B’s investment was used?

(A)  12 months

(B)  10 months

(C)  15 months

(D)  14 months

(E)  18 months

Answer: (A)

32. There are 3 red balls, 4 blue balls and 5 white balls. 2 balls are chosen randomly. Find the probability that 1 is red and the other is white ?

(A)  5/22

(B)  5/23

(C)  7/22

(D)  4/9

(E)  None of these

Answer: (A)

33. According to a new plan rolled out by HISP Bank, the rate of simple interest on a sum of money is 8% p.a. for the next three years and 6% p.a. for the period beyond that first five years. Simple interest accrued on a sum for a period of eight years is Rs 12,800. Find the sum-

(A)  Rs 24,000

(B)  Rs 16,000

(C)  Rs 15,000

(D)  Rs 13,500

(E)  None of these

Answer: (E)

34. Three Science classes A, B and C take a Life Science test. The average score of students of class A is 83. The average score of students of class B is 76. The average score of students of class C is 85. The average score of class A and B is 79 and average score of class B and C is 81. Then the average score of class A, B and C is-

(A)  80

(B)  80.5

(C)  81

(D)  81.5

(E)  None of these

Answer: (D)

35. A hemispherical bowl of internal diameter 54 cm contains a liquid. The liquid is to be filled in cylindrical bottles of radius 3 cm and height 9 cm. How many bottles are required to empty the bowl?

(A)  221

(B)  343

(C)  81

(D)  243

(E)  None of these

Answer: (E)

SBI PO (Pre.) Examination Held on 1-7-2018 Quantitative Aptitude Question Paper With Answer Key

SBI PO (Pre.) Examination Held on 1-7-2018 Quantitative Aptitude
SBI PO (Pre.) Examination Held on 1-7-2018 Quantitative Aptitude Question Paper With Answer Key

SBI PO (Pre.) Examination Held on 1-7-2018

Quantitative Aptitude

Directions- (Q. 1to 5) Bar graph given  below shows pens sold by retailer on five different days. Study the data carefully and answer the following questions-

1. Find the difference between total number of pens sold on Monday and Tuesday together to total number of pens sold on Thursday and Friday together?

(A) 15

(B) 10

(C) 5

(D) 20

(E) 0

Answer: (C)

2. Total number of pens sold on Saturday is 40% more than total number of pens sold on Wednesday. Find total number of pens sold on Friday and Saturday together?

(A) 92

(B) 110

(C) 72

(D) 108

(E) 85

Answer: (A)

3. Total number of pens sold on Tuesday are 25% more than total number of pens sold on Sunday. Find total number of pens sold on Sunday?

(A) 64

(B) 50

(C) 94

(D) 60

(E) 55

Answer: (D)

4. Out of total pens sold on Thursday, 20% are blue ink pen. Out of remaining 25% are red ink pen and remaining are black in pen. Find total number of blue and black ink pen sold on Thursday?

(A) 27

(B) 36

(C) 45

(D) 39

(E) 30

Answer: (B)

5. Out of total pens sold on Tuesday ratio between total defective pens sold to total pens sold is 7 : 15. Find total number of non-defective pens sold on Tuesday by retailer?

(A) 20

(B) 25

(C) 30

(D) 35

(E) 40

Answer: (E)

Directions- (Q. 6 to 10) – In each of given questions two quantities numbered I and II are given. You have to solve both the quantities and mark the appropriate answer.

6. Quantity :

(I) ‘x’ : x2 + x – 6 = 0

(II) ‘y’ : y2 + 7y + 12 = 0

(A) Quantity I > Quantity II

(B) Quantity I < Quantity II

(C) Quantity I ≥ Quantity II

(D) Quantity I ≤ Quantity II

(E) Quantity I = Quantity II or No relation

Answer: (C)

7. A’s efficiency is 25% more than B

Quantity:

(I) ‘x’ : A can do 5/6th of total work in ‘x’ days.

(II) ‘y’ : B can do 4/5th of total work ‘y’ days

(A) Quantity I > Quantity II

(B) Quantity I < Quantity II

(C) Quantity I ≥ Quantity II

(D) Quantity I ≤ Quantity II

(E) Quantity I = Quantity II or No relation

Answer: (B)

8. Sum of 8 consecutive even number is S1.

Quantity :

(I) Sum of second number and eight number in S1

(II) Sum of third number and sixth number in S1

(A) Quantity I > Quantity II

(B) Quantity I < Quantity II

(C) Quantity I ≥ Quantity II

(D) Quantity I ≤ Quantity II

(E) Quantity I = Quantity II or No relation

Answer: (A)

9. An article is sold at Rs 1,500 after allowing discount of 12.5% on Marked price.

Quantity :

(I) Rs 550

(II) Mark price of article.

(A) Quantity I > Quantity II

(B) Quantity I < Quantity II

(C) Quantity I ≥ Quantity II

(D) Quantity I ≤ Quantity II

(E) Quantity I = Quantity II or No relation

Answer: (B)

10. If a speed of boat is 500% more than the speed of a current.

Quantity :

(I) ‘x’ : If boat can travel a distance of 63 km in 3 hr., in downstream then ‘x’ is the speed of the boat in upstream (km/hr.)

(II) 15 km/hr

(A) Quantity I > Quantity II

(B) Quantity I < Quantity II

(C) Quantity I ≥ Quantity II

(D) Quantity I ≤ Quantity II

(E) Quantity I = Quantity II or No relation

Answer: (E)

Directions – (Q. 11 to 15) What number is wrong according to given number series pattern :-

11. 1, 3, 9, 31, 128, 651, 3913

(A) 9

(B) 1

(C) 128

(D) 31

(E) 3913

Answer: (C)

12. 291, 147, 75, 39, 22, 12, 7.5

(A) 22

(B) 291

(C) 147

(D) 75

(E) 7.5

Answer: (A)

13. 26, 27, 34, 58, 106, 186, 306

(A) 26

(B) 34

(C) 58

(D) 106

(E) 27

Answer: (E)

14. 9, 6, 6.1, 6.4, 7.9, 18.5, 112.9

(A) 6

(B) 5.9

(C) 6.1

(D) 18.5

(E) 112.9

Answer: (D)

15. 330, 80, 280, 120, 250, 130, 240

(A) 330

(B) 130

(C) 280

(D) 240

(E) 80

Answer: (B)

16. Sum of volume of cylinder (S) and volume of cone (C) is 2190π cm2 & height of both cylinder and cone is same i.e., 10 cm. If radius of cone is 15 cm then find the ratio of radius of S to radius of C?

(A) 1 : 2

(B) 3 : 4

(C) 2 : 5

(D) 4 : 5

(E) 3 : 5

Answer: (D)

17. In a box there are 6 blue ball, red balls & 10 green balls. Probability of choosing one red ball from the given box is 1/3. Then find the sum of red and blue balls in the box?

(A) 20

(B) 12

(C) 14

(D) 18

(E) 16

Answer: (C)

18. Sum of A’s and B’s age 6 years ago is 88. A’s age 18 years ago is equal to B’s age 6 years ago. Find the age of A two y ear hence?

(A) 58 years

(B) 64 years

(C) 42 years

(D) 52 years

(E) 48 years

Answer: (A)

19. Train A of length 120 m can cross a platform of length 240 m in 18 second the ratio of speed of train A and Train B is 4 : 5. Then find the length of Train B if train B can cross a pole in 12 seconds-

(A) 280 m

(B) 300 m

(C) 320 m

(D) 350 m

(E) 240 m

Answer: (B)

20. What is the probability of forming word from the letters of word ‘IMPEACH’ such that all vowels come together?

(A) 8/35

(B) 1/7

(C) 3/35

(D) 17/35

(E) 2/7

Answer: (B)

Directions – (Q. 21 to 25) Find the value of (?) in following approximation questions-

21. 2? = 32.01 ÷01 × 1023.99 ÷ 7.99

(A) 7

(B) 3

(C) 4

(D) 5

(E) 8

Answer: (D)

22. 

(A) 17

(B) 20

(C) 10

(D) 34

(E) 40

Answer: (A)

23. 

(A) 14

(B) 21

(C) 8

(D) 27

(E) 17

Answer: (E)

24. (? ÷97) × 12.08 = 20.12% of 1319.97

(A) 220

(B) 240

(C) 260

(D) 280

(E) 200

Answer: (A)

25. 

(A) 80

(B) 60

(C) 40

(D) 10

(E) 20

Answer: (D)

Directions- (Q. 26 to 30) Pie chart given below shows total number of workers in three different companies. Table given below shows ratio between officers and workers working in these companies. Study the data carefully and answer the following questions-

26. Find the ratio between total number of workers in company A and C together to total number of officers in company A and C together?

(A) 16 : 1

(B) 12 : 1

(C) 14 : 1

(D) 18 : 1

(E) 20 : 1

Answer: (C)

27. Total number of employees in company ‘B’ is how much more than total number of employees in company ‘C’-

(A) 174

(B) 194

(C) 204

(D) 214

(E) 184

Answer: (E)

28. Total number of officers in company ‘A’ is how much less than total number of officers in company ‘B’?

(A) 4

(B) 2

(C) 0

(D) 6

(E) 8

Answer: (A)

29. Total number of officers and workers in company D is 50% and 25% more than total number of officers and workers in company ‘C’ respectively. Find total number of employees in company ‘D’?

(A) 279

(B) 297

(C) 342

(D) 324

(E) 306

Answer: (B)

30. Find the difference between total number of workers in company ‘A’ and total number of works in company ‘B’ and ‘C’ together?

(A) 432

(B) 396

(C) 360

(D) 324

(E) 288

Answer: (D)

Directions- (Q. 31 to 35) There are three persons A, B and C who each invested in two different scheme S1 and S2. A invested Rs 80,000 for 2 year in scheme S1 and Rs 30,000 for 4 years in scheme S2. B invested Rs 30,000 for 3 year in S1 and he did not invest in scheme S2. B also obtained a profit of Rs 10,000 by selling his car, C invested Rs 50,000 for 5 years in scheme S1 and Rs 10,000 for 3 year in scheme S2. Total profit obtained from scheme S1 is 2 lakh and scheme S2 is Rs 90,000.

31. What is the ratio of total profit obtained by B and profit obtained by C from scheme S1

(A) 23 : 47

(B) 54 : 47

(C) 36 : 43

(D) 23 : 50

(E) 27 : 50

Answer: (D)

32. Profit obtained by A from scheme S1 is what per cent of profit obtained by C from scheme S2?

(A)  

(B)   

(C)  

(D)   

(E)  

Answer: (E)

33. If sum of investment of A in both schemes and total profit obtained by A from both scheme is invested at compound interest at the rate of 20% p.a. then find the total compound interest obtained in 2 years?

(A) Rs 1,08,240

(B) Rs 1,04,206

(C) Rs 1,05,208

(D) Rs 1,09,280

(E) Rs 1,06,220

Answer: (A)

34. What is the average of profit attained by A from scheme S1 and profit of C obtained from scheme S2?

(A) Rs 41,000

(B) Rs 42,000

(C) Rs 44,000

(D) Rs 55,000

(E) Rs 40,000

Answer: (A)

35. If A had invested his sum at Simple interest for 3 year at the rate of R% p.a. instead in scheme S1 and B has invested his sum at compound interest at (R + 5%) p.a. for 1 year and difference in interest obtained is Rs 30,000 then find value of R%-

(A) 10%

(B) 9%

(C) 15%

(D) 18%

(E) 12%

Answer: (C)

SBI PO (Pre.) Examination 2nd Shifting Held on 29-04-2017 Quantitative Aptitude Question Paper With Answer Key

SBI PO (Pre.) Examination 2nd Shifting Held on 29-04-2017 Quantitative Aptitude
SBI PO (Pre.) Examination 2nd Shifting Held on 29-04-2017 Quantitative Aptitude Question Paper With Answer Key

SBI PO (Pre.) Examination 2nd Shifting Held on 29-04-2017

Quantitative Aptitude

Directions- (Q. 1-5) Refer to the pie chart and answer the given question.

1. The average number of jacket sold by store A in 2004 and 2005 is 345. The average number of jacket sold by store E in 2004 and 2005 is 452. What is the difference between number of jackets sold by store A and that by store E in 2005?

(A)  125

(B)  143

(C)  131

(D)  147

(E)  79

Answer: (E)

2. Store D sold only two types of jackets-leather and woolen. If the number of leather jackets sold by store D is 84, what is the respective ratio between the number of leather and woolen jackets sold by store D?

(A)  14 : 11

(B)  12 : 9

(C)  14 : 13

(D)  7 : 6

(E)  12 : 7

Answer: (A)

3. Number of jackets sold by store C increased by 50% from 2004 to 2005 and decreased by 25% from 2005 to 2006, what was the number of jackets sold by store C in 2006?

(A)  130

(B)  135

(C)  145

(D)  125

(E)  140

Answer: (B)

4. What is the total number of jackets sold by stores D and F together?

(A)  440

(B)  405

(C)  475

(D)  480

(E)  455

Answer: (B)

5. What is the central angle corresponding to the number of jackets sold by store F?

(A)  61.2°

(B)  58.6°

(C)  62.8°

(D)  54.4°

(E)  65.4°

Answer: (A)

6. A jar had a certain quantity (in litre) of water, to which the pure milk of seven times the quantity of water was added. 8 litre of the mixture was taken out and replaced with 10 litre of pure milk, as result of which milk constituted 90% of the resultant mixture. What was the initial quantity of water in the jar?

(A)  12 litres

(B)  3 litres

(C)  4 litres

(D)  6 litres

(E)  7 litres

Answer: (D)

Directions – (Q. 7-11) Study the following table and answer the given question.

Note- Total Employees : Total Gazetted employees + Total Non-Gazetted employees.

7. The number of Gazetted employees (both male and female) in Company D are what per cent more than those in Company C?

(A)  15

(B)  25

(C)  10

(D)  30

(E)  20

Answer: (E)

8. The number of male non-gazetted employees in Company A and those in Company C are equal. If the respective ratio between female Non-Gazetted employees in Company A and those in Company C is 11 : 17, what is the number of male non-Gazetted employees in Company A?

(A)  65

(B)  55

(C)  50

(D)  60

(E)  75

Answer: (A)

9. What is the difference number of male gazetted employees in Company A and those in Company D?

(A)  195

(B)  208

(C)  204

(D)  218

(E)  214

Answer: (B)

10. What is the average number of female gazetted employees in Company A and those in Company B?

(A)  380

(B)  385

(C)  370

(D)  390

(E)  350

Answer: (A)

11. If in Company B, 148 females are non-gazetted employees, approximately what per cent less than females are gazette employees?

(A)  59

(B)  73

(C)  65

(D)  64

(E)  78

Answer: (B)

12. The respective ratio between present ages of P and Q is 9 : 5. The respective ratio between P’s age 6 years hence and Q’s age 5 years ago is 12 : 5. What is the respective ratio between P’s age 12 years ago and Q’s age 6 years hence?

(A)  5 : 3

(B)  7 : 6

(C)  9 : 7

(D)  11 : 6

(E)  4 : 3

Answer: (B)

Directions – (Q. 13-17) In this question two equations numbered I and II are given. You have to solve both the equations and choose the appropriate option.

13. (I) x2 + 13x + 40 = 0

(II) y2 + 7y + 12 = 0

(A)  x ≥ y

(B)  x < y

(C)  x ≤ y

(D)  Relationship between x and y cannot be established

(E)  x > y

Answer: (B)

14. (I) 2x2 – 17x + 36 = 0

(II) 2y2 – 15y + 28 = 0

(A)  x ≥ y

(B)  x < y

(C)  x ≤ y

(D)  Relationship between x and y cannot be established

(E)  x > y

Answer: (A)

15. (I) 3x2 – 13x + 12 = 0

(II) 2y2 – 9y + 10 = 0

(A)  x ≥ y

(B)  x < y

(C)  x ≤ y

(D)  Relationship between x and y cannot be established

(E)  x > y

Answer: (D)

16. (I) x2 = 49

(II) y2 – 15y + 56 = 0

(A)  x ≥ y

(B)  x < y

(C)  x ≤ y

(D)  Relationship between x and y cannot be established

(E)  x > y

Answer: (C)

17. (I) 3x2 + 10x + 8 = 0

(II) 3y2 + 10y + 7 = 0

(A)  x ≥ y

(B)  x < y

(C)  x ≤ y

(D)  Relationship between x and y cannot be established

(E)  x > y

Answer: (D)

18. A rectangular plot, 26 m long and 18 m wide, has two concrete crossroads (of same width) running in the middle of the plot (one parallel to length and the other parallel to breadth). The rest of the plot is used for gardening. If the area used for gardening is 345 sq. m, what is the width of the roads?

(A)  2 m

(B)  3.5 m

(C)  4 m

(D)  3 m

(E)  2.5 m

Answer: (D)

19. The average runs scored by a batsman in forty-five matches were 42 runs. The difference between his highest and lowest runs scored was 114. If those two matches (on which he scored the highest and the lowest runs) are excluded, his average score will be 40. What will be his lowest score?

(A)  25

(B)  32

(C)  34

(D)  28

(E)  36

Answer: (D)

20. Sonu invested a certain sum (Rs x) in scheme A (offering compound interest compounded annually) @20% p.a. for two years and  in scheme B (offering simple interest p.a.) @16% p.a. for five years. If the difference between the amounts he earned from both schemes was Rs 3,510, how much did he invest in scheme A?

(A)  Rs 6,000

(B)  Rs 7,000

(C)  Rs 6,500

(D)  Rs 7,500

(E)  Rs 14,625

Answer: (E)

21. Cars M and N started from at the same time from place A and place B respectively. They travelled towards each other @35 kmph and @45 kmph respectively. When these cars met it was found that one of the cars had travelled 114 km more than the other. What is the distance between the two places (A and B) ? (In km)

(A)  570

(B)  760

(C)  820

(D)  855

(E)  912

Answer: (E)

22. Mallika purchased 30 kg of rice @ Rs24/- per kg and another 20 kg of rice at a certain rate. She mixed the two and sold the entire quantity @ Rs 38.12 per kg and made 40% overall profit. At what price kg did she purchased the lot of other 20 kg rice (Approx.)?

(A)  Rs 29.50

(B)  Rs 28.50

(C)  Rs 28

(D)  Rs 25.50

(E)  Rs 32

Answer: (E)

23. Kanika keeps aside 24% of his monthly salary for monthly expenses. From the remaining salary, he gave 1/4 to his mother and kept the remaining in his bank account. Difference between the amounts he gave to his mother and that he kept in h is bank account was Rs 11,400. What is Kanika’s monthly salary?

(A)  Rs 20,000

(B)  Rs 40,000

(C)  Rs 25,000

(D)  Rs 30,000

(E)  Rs 35,000

Answer: (D)

Directions – (Q. 24-28) What will come in place of question mark (?) in the given number series?

24. 103.1 100.8 96.2 87 ? 31.8

(A)  71.2

(B)  53.6

(C)  54.4

(D)  68.6

(E)  64.3

Answer: (D)

25. 126 64 34 20 14 ?

(A)  11

(B)  10

(C)  6

(D)  8

(E)  12

Answer: (E)

26. 3 5 12 38 154 ?

(A)  611

(B)  582

(C)  402

(D)  344

(E)  722

Answer: (E)

27. 18 8 6 8 24 ?

(A)  132

(B)  145

(C)  194

(D)  176       

(E)  182

Answer: (D)

28. 84 89 72 109 ? 145

(A)  35

(B)  56

(C)  71

(D)  53

(E)  44

Answer: (E)

29. A boat takes 7 hours 3 minutes to travel 90 km upstream. The respective ratio between the speed of the boat downstream and the speed of the boat upstream is 3 : 2. How much time will the boat take to cover 108 km downstream?

(A)  8 hours

(B)  5 hours

(C)  6 hours 25 minutes

(D)  4 hours 30 minutes

(E)  4 hours 25 minutes

Answer: (A)

30. A and B started a business by investing Rs 12,000 and Rs 18000 respectively. After 4 months C joined him with an amount equal to that invested by A. After 2 months of C’s joining. A and B withdraw 1/4 of their respective payments. If they earned an annual profit of Rs 91,500, what is C’s share of annual profit (Approx.)?

(A)  Rs 32,000

(B)  Rs 18,000

(C)  Rs 28,000

(D)  Rs 20,000

(E)  Rs 21,372

Answer: (E)

Directions – (Q. 31-35) What approximate value will come in place of question-mark (?) in the given question ? (You are not expected to calculate the exact value).

31. 79.99% of 799 + 35.01 × 1.99 = ?

(A)  790

(B)  690

(C)  710

(D)  750

(E)  650

Answer: (C)

32. 1649.99 – ? = 899 ÷ 2.01

(A)  1200

(B)  1280

(C)  1260

(D)  900

(E)  1410

Answer: (A)

33. 61 ÷ 4 + 29.99 ×01 = ?

(A)  125

(B)  75

(C)  90

(D)  60

(E)  105

Answer: (E)

34. 39.99% of 149.97 + 35.08% of 199 = ?

(A)  190

(B)  170

(C)  130

(D)  95

(E)  110

Answer: (C)

35. (2569.95 + 769.87) ÷61 = 749 + ?

(A)  610

(B)  760

(C)  780

(D)  920

(E)  940

Answer: (D)

SBI PO (Pre.) Examination Held on 30-04-2017 Quantitative Aptitude Question Paper With Answer key

SBI PO (Pre.) Examination Held on 30-04-2017 Quantitative Aptitude
SBI PO (Pre.) Examination Held on 30-04-2017 Quantitative Aptitude Question Paper With Answer key

SBI PO (Pre.) Examination Held on 30-04-2017

Quantitative Aptitude

1. ?2 – 137.99 ÷ 6 = 21.99 ×01

(A)  23

(B)  50

(C)  42

(D)  29

(E)  35

Answer: (A)

2. ?% of 400.02 + 12.932 = 285

(A)  18

(B)  15

(C)  24

(D)  34

(E)  29

Answer: (E)

3. (3327.99 – 27.93) ÷ ? = 110 ×99

(A)  9

(B)  1

(C)  19

(D)  15

(E)  5

Answer: (E)

4. 5520 ÷01 + √226 × 5.99 = ?

(A)  350

(B)  550

(C)  500

(D)  450

(E)  250

Answer: (B)

5. 160.01 + 40 ÷ (16.5 ÷ 33) = ?

(A)  310

(B)  290

(C)  250

(D)  350

(E)  240

Answer: (E)

Directions – (Q. 6-10) Refer to the graph and answer the given question.

6. Number of students who opted for Course B in 2013 was what per cent more than that who opted for course A in 2013?

(A) 

(B)   

(C)    

(D)    

(E)    

Answer: (C)

7. What is the difference between the total number of students who opted for courses A and B together in 2012 and that who opted for both the courses together in 2014?

(A)  50

(B)  30

(C)  60

(D)  40

(E)  20

Answer: (D)

8. In 2014, if ‘X’ students passed courses A and B each and the ratio of number of students that failed courses A and B respectively was 5 : 2, what is the value of ‘X’?

(A)  190

(B)  220

(C)  160

(D)  150

(E)  180

Answer: (B)

9. What is the average number of students who opted for course A in 2010, 2011 and 2012?

(A)  225

(B)  250

(C)  230

(D)  240

(E)  260

Answer: (D)

10. The number of students who opted for courses A and B in 2011n was respectively 25% more and 35% less than that in 2009. What was the total number of students who opted for courses A and B together in 2009?

(A)  600

(B)  540

(C)  575

(D)  560

(E)  584

Answer: (D)

11. The respective ratio between numerical values of curved surface area and volume of right circular cylinder is 1 : 7. If the respective ratio between the diameter and height of the cylinder is 7 : 5, what is the total surface area of the cylinder ? (in m2)

(A)  2992

(B)  3172

(C)  2882

(D)  3576

(E)  3992

Answer: (A)

12. The time taken by the boat to cover a distance of ‘D – 56’ km upstream is half of that taken by it to cover a distance of ‘D’ km downstream. The respective ratio between the speed of the boat downstream and that upstream is 5 : 3. If the time taken to cover ‘D-32’ km upstream is 4 hours, what is the speed of water current? (in km/h)

(A)  5

(B)  3

(C)  4

(D)  16

(E)  8

Answer: (C)

13. Poona invests Rs 4200 in scheme A which offers 12% p.a. simple interest. She also invests Rs (4200 –P) in scheme B offering 10% p.a. compound interest (compounded annually). The difference between the interests Poona earns from both the schemes at the ends of 2 years is Rs 294, what is the value of P?

(A)  1500

(B)  800

(C)  600

(D)  1000

(E)  Other than those given as options

Answer: (B)

14. Man sold two articles-A(at a profit of 40%) and B (at a profit of 20%). He incurred a total profit of Rs 169 in the whole deal. If article A costs Rs 140 less than article B, what is the price of article B?

(A)  Rs 380

(B)  Rs 280

(C)  Rs 340

(D)  Rs 375

(E)  Rs 300

Answer: (D)

15. A is eighteen years older to B. The respective ratio of B’s age six years hence and C’s present age is 3 : 2. If at present A’s age is twice the age of C, what was B’s age four years ago?

(A)  26 years

(B)  28 years

(C)  29 years

(D)  20 years

(E)  16 years

Answer: (A)

16. C is twice as efficient as B. If B and C together can complete the same price of work in  in how many days can C alone finish the same piece of work?

(A)  36

(B)  24

(C)  16

(D)  28

(E)  Other than those given as options

Answer: (E)

17. Out of her monthly salary, Ridhi spends 34% various expenses. From the remaining, she gives 1/6th to her brother, 2/3rd to her sister and the remaining she keeps as savings. If the difference between the amounts she gave to her sister and brother was Rs 10,560, what was Ridhi’s savings?

(A)  Rs 3,740

(B)  Rs 3,420

(C)  Rs 4,230

(D)  Rs 3,230

(E)  Other than those given as options

Answer: (E)

18. A bag contains 63 cards (numbered 1, 2, 3, ……, 63). Two cards are picked at random from the bag (one after another and without replacement), what is the probability that the sum of numbers of both the cards drawn is even?

(A)  11/21

(B)  34/63

(C)  7/11

(D)  11/63

(E)  Other than those given as options

Answer: (E)

19. 80 litre of milk is mixed with some water such that milk to water ratio become 8 : 3. ‘x’ litre of mixture is taken out and 5 litres of water is added. If in resultant mixture, water is 40 litre less than the milk, find the value of x?

(A)  16 litre

(B)  13 litre

(C)  11 litre

(D)  15 litre

(E)  None of the above

Answer: (C)

Directions – (Q. 20-24) In this question, two equations numbered I and II are given. You have to solve both the equations and mark the appropriate answer and answered as

(A) x > y       (B) x ≥ y

(C) x < y       (D) x ≤ y

(E) x = y or relationship cannot be established

20. (I) 3x2 – 4x + 1 = 0

(II) 15y2 – 8y + 1 = 0

Answer: (B)

21. (I) x2 + 14x + 45 = 0

(II) y2 + 19y + 88 = 0

Answer: (E)

22. (I) x2 – 2x – 8 = 0

(II) y2 + 15y + 54 = 0

Answer: (A)

23. (I) 2x2 – 9x + 9 = 0

(II) y2 – 7y + 12 = 0

Answer: (D)

24. (I) x2 = 121

(II) y2 – 23y + 132 = 0

Answer: (D)

25. A jar contains mixture of milk and water in the respective ratio of 3 : 1. 1/25th of the mixture was added to it. If the resultant ratio between milk and water in the jar was 2 : 1, what was the initial quantity of mixture in the jar ? (in litre)

(A)  160

(B)  180

(C)  200

(D)  250

(E)  192

Answer: (C)

26. 17 9 10 16.5 ? 90

(A)  44

(B)  35

(C)  48

(D)  38

(E)  33

Answer: (B)

27. 7 6 10 27 ? 515

(A)  112

(B)  104

(C)  114

(D)  96

(E)  108

Answer: (B)

28. 33 40 29 42 25 ?

(A)  40

(B)  44

(C)  52

(D)  48

(E)  46

Answer: (B)

29. 316 307 282 233 152 ?

(A)  35

(B)  25

(C)  31

(D)  41

(E)  47

Answer: (C)

30. 5 9 33 72 121 ?

(A)  169

(B)  163

(C)  171

(D)  184

(E)  Other than those given as options

Answer: (E)

Directions – (Q. 31-35) Study the table and answer the given option.

Data regarding the number of voters at 6 different centres – A, B, C, D, E.

31. What is the average number of people who voted at centres B, D and E?

(A)  1700

(B)  1880

(C)  1720

(D)  1740

(E)  1560

Answer: (A)

32. What per cent of the total number of registered voters cast invalid votes at Centre D, if the number of invalid voters cast at Centre D was 10% of the number of votes cast?

(A)  5.5

(B)  8.5

(C)  7.5

(D)  6.5

(E)  Other than those given as options

Answer: (E)

33. At centre F, the total number of registered voters was 25% less than that at Centre C. At Centre F, number of people who voted was 450 less than that at Centre C and 150 votes cast were declared invalid. What was the respective ratio between the number of valid votes cast and the total number of registered voters at centre F?

(A)  4 : 5

(B)  3 : 4

(C)  2 : 3

(D)  6 : 1

(E)  5 : 8

Answer: (C)

34. Number of people who did not vote at Centre D was what per cent more than that who did not vote at Centre A?

(A)   

(B)    

(C)   

(D)    

(E)  Other than those given as options

Answer: (E)

35. What is the difference between the total number of people who did not vote at Centres A and B together and that who did not vote at Centres D and E together?

(A)  80

(B)  62

(C)  50

(D)  70

(E)  118

Answer: (B)

SBI PO (Pre.) Examination Held on 30-04-2017 Reasoning Question Paper With Answer Key

SBI PO (Pre.) Examination Held on 30-04-2017 Reasoning
SBI PO (Pre.) Examination Held on 30-04-2017 Reasoning Question Paper With Answer Key

SBI PO (Pre.) Examination Held on 30-04-2017

Reasoning

1. Five movies – D, E, F G and H are released on five different days of the same week starting from Monday and ending on Friday, but not necessarily in the same order. F is released on one of the days before Thursday. Only two movies are released between F and G. H is released immediately before G. D is released on one of the days after H. Which movie was released on Wednesday?

(A)  E

(B)  Either G or D

(C)  G

(D)  H

(E)  F

Answer: (D)

2. Four cartons – A, B, Y and Z are placed above one another, but not necessarily in the same order. Each carton contains a different drink-Pepsi, Coffee, Frooti and Milkshake, but not necessarily in the same order. Only carton B is kept between the cartons of Pepsi and Frooti. Carton of Coffee is kept immediately below carton of Frooti. Carton of Coffee is kept at one of the positions below Z. What is the position of the carton of Milkshake in the stack?

(A)  Cannot be determined

(B)  Immediately below the carton of Frooti

(C)  First from the bottom

(D)  Immediately above the carton of coffee

(E)  Second from the top

Answer: (A)

Directions – (Q. 3 to 5) Study the following information and answer the given question.

  Each of the six stores P, Q, R, S, T and U sold different number of books in one day. Only three stores sold less books than U. P sold more books than R. T did not sell the highest number of books. S sold more books than R and P, but less than U. The store which sold the second highest number of books sold 72 books.

3. How many books did Q probably sell?

(A)  43

(B)  58

(C)  71

(D)  65

(E)  89

Answer: (E)

4. Which of the following stores sold the second lowest number of books?

(A)  T

(B)  P

(C)  S

(D)  R

(E)  Q

Answer: (B)

5. If the total number of books sold by P ad T is 125, then how many books did P sell?

(A)  51

(B)  76

(C)  68

(D)  45

(E)  53

Answer: (E)

6. In a vertical queue of 13 people, all facing north, K stands exactly at the centre of the queue. No one stands between K and W. Only five people stand between W and P. L stands at one of the positions before P, but not at the beginning of the queue. How many people stood after W?

(A)  Three

(B)  None

(C)  Five

(D)  Seven

(E)  Six

Answer: (D)

7. In which of the given expressions, does the expression ‘C < P’ definitely holds false?

(A)  P ≥ A ≥ L ≤ E ; C ≤ L ≥ O > N

(B)  P < A ≤ L ≥ E; C ≥ L ≤ O < N

(C)  P = A ≥ L = E; C = L > O < N

(D)  P > A > L > E; C < L < O < N

(E)  P = A ≥ A < E; C < L ≥ O ≥ N     

Answer: (B)

8. A person starts from Point A, walks 30 m towards south and reaches Point B. He then takes right walks 7 m followed by a right turn and walks for 6 m. He then takes a right turn and walks 7 m. He takes a final left turn, walks a certain distance and reaches Point R. Point R is 17 m to the north of Point B. What is the distance between Point A and Point R?

(A)  18 m

(B)  23 m

(C)  21 m

(D)  27 m

(E)  13 m

Answer: (E)

9. S is the only son of V. V is married to R. M is the daughter of R. R is the grandmother of A. How is S definitely related to A?

(A)  Uncle

(B)  Cannot be determined

(C)  Father

(D)  Father-in-law

(E)  Nephew

Answer: (B)

10. What should come in place of $ and # respectively in the expression-

P ≥ A $ R ≤ O ≤ T; S < L ≤ A # M, so that the expression T > M definitely holds true?

(A)  >, ≤

(B)  ≤, =

(C)  < , <

(D)  ≤, ≤

(E)  ≥, ≥

Answer: (C)

Directions – (Q. 11 to 15) Study the given information carefully to answer the given question.

  C, D, E, F W, X, Y and Z have to attend a wedding in January, April, September and December months of the same year. In each month the wedding is on either the 11th or the 24th of the month. Not more than two of the given people have to attend a wedding in the same month.

  W has to attend a wedding on the 11th of the month which has only 30 days. Only three people have to attend a wedding between W and Y. C and Y have to attend a wedding neither on the same date nor in the same month. C does not have to attend a wedding in April. Only two people have to attend a wedding between C and F. X and F have to attend a wedding on the same date. D has to attend a wedding on one of the days before X. Only one person has to attend a wedding between D and E. Less than four people have to attend a wedding between E and Z.

11. How many people have to attend a wedding between F and Z?

(A)  Two

(B)  Three

(C)  None

(D)  More than three

(E)  One

Answer: (D)

12. When does X have to attend a wedding?

(A)  24th April

(B)  Cannot be determined

(C)  11th January

(D)  24th September

(E)  11th December

Answer: (E)

13. If all the people are made to attend the wedding in alphabetical order starting from 11th January and ending on 24th December, the schedule of how many people will remain unchanged?

(A)  One

(B)  Two

(C)  Five

(D)  None

(E)  Three

Answer: (E)

14. Who among the following has to attend a wedding before y?

(A)  Both C and X

(B)  Only W

(C)  None

(D)  Both F and W

(E)  Only F

Answer: (C)

15. As per the given arrangement, four of the following five are alike in a certain way and so form a group. Which of the following does not belong to the group?

(A)  W

(B)  F

(C)  Z

(D)  Y

(E)  X

Answer: (C)

Directions – (Q. 16 to 18) In the questions, relationship between different elements is shown in the statements. The statements are followed by two conclusions. Study the conclusions based on the given statements and select the appropriate answer. And answered as-

(A) Only conclusions I follows

(B) Either conclusion I or II follows

(C) Both conclusion I and II follows

(D) Neither conclusion I nor II follows

(E) Only conclusion II follows

16. Statements:

M < O ≤ U ≤ R ≥ T;

P ≥ R ≤ I ≤ C < L

Conclusion: (I) L > M      (II) O ≤ C

Answer: (C)

17. Statements:

C < L = L ≤ N > G;

I < M ≥ O > R > T

Conclusion: (I) C < O      (II) G > T

Answer: (D)

18. Statements:

M < O ≤ U ≤ R ≥ T;

P ≥ R ≤ I ≤ C < L

Conclusion: (I) P > T       (II) P = T

Answer: (B)

Directions – (Q. 19 to 23)  Read the given information to answer the given questions-

  Eight people viz., X, G, T, C, P, J, A and M live on different floors of a building. The ground floor of the building is numbered one, the one above that is numbered two and so on till the topmost floor is numbered eight. All of t hem can perform a different form of dance viz., Kathak, Garba, Dandiya, Bhangra, Lavani, Odissi, Mohiniyattam and Sattriya.

(Note : None of the given information is necessarily in the same order.)

  T lives on an even numbered floor below floor number five. Only three people live between T and the one who performs Garba. As many people live below T same live above the one who performs Lavani. Number of people living between the one who performs Garba and Lavani is equal to the number of people living between C and P. C lives on an even numbered floor above P. Neither C nor P performs Garba or Lavani. The one who performs Kathak lives on an odd numbered floor below floor number four. P does not perform Kathak. The number of people between T and the one who performs Kathak is same as the number of people living between X and the one who performs Lavani. X lives on one of the floors above the one who performs Lavani. Number of people living between C and X is equal to the number of people living between T and M. The one who performs Odissi lives on an even numbered floor immediately above the one who performs Bhangra. Only three people live between G and J. G lives on one of the floors above J. The one who performs Bhangra lives immediately above the one who performs Sattriya. G does not perform daniya.

19. Which of the following statements is not true as per the given arrangement?

(A)  C performs Odissi.

(B)  The one who performs Garba lives on floor number six.

(C)  T lives immediately above J.

(D)  All the given statements is true.

(E)  Only two people live between M and G.

Answer: (E)

20. Four of the following five are alike in a certain way based on the given arrangement and thus form a group. Which one of the following does not belong to the group?

(A)  M-Bhangra

(B)  C-Lavani

(C)  A-Odissi

(D)  G-Floor number 8

(E)  J-Floor number 5

Answer: (E)

21. How many people live between the one who performs Lavani and Bhangra?

(A)  More than three

(B)  Three

(C)  None

(D)  One

(E)  Two

Answer: (B)

22. Who performs Sattriya?

(A)  G

(B)  A

(C)  P

(D)  T

(E)  C

Answer: (D)

23. Which dance M performs?

(A)  Garba

(B)  Lavani

(C)  Odissi

(D)  Mohiniyattam

(E)  Kathak

Answer: (A)

Directions-(Q. 24 to 28) Study the given information carefully to answer the given question.

  Eight people – A, B, C, D, E, F, G and H were born in different years, viz. 1961, 1970, 1974, 1980, 1983, 1987, 1996 and 2000, but not necessarily in the same order.

Note: (A) All calculation are done with respect to the present year, 2017 assuming the month and date to be same as that of the years of birth as mentioned above.

(B) Each person is assumed to born on the same date and same month of the respective years.

D was born after 1983, but not in the year 2000. The sum of the present ages of A and D is 64. The difference between the present ages of A and G is less than 5. B was born in an odd numbered year. B is older than G. The sum of present ages of F and C is 68. F is younger than C. H is not the youngest.

24. Four of the following five are alike in a certain way as per the given arrangement and hence form a group. Which of the following does not belong to the group?

(A)  E

(B)  B

(C)  C

(D)  F

(E)  G

Answer: (B)

25. How many persons is/are younger than E?

(A)  Two

(B)  More than three

(C)  None

(D)  Three

(E)  One

Answer: (C)

26. Who amongst the following was born in the year 1996?

(A)  E

(B)  C

(C)  A

(D)  G

(E)  F

Answer: (E)

27. If A’s uncle is 21 years older than B, then how old is A’s uncle at present (in years)?

(A)  77

(B)  68

(C)  89

(D)  64

(E)  85

Answer: (A)

28. Which of the following represents the difference between the present ages of B and H (in years)?

(A)  22

(B)  19

(C)  25

(D)  32

(E)  13

Answer: (E)

29. In the number 7853921, one is added to each digit exactly divisible by two. Two is subtracted from each digit exactly divisible by three. All other digits are kept unchanged. In the new number thus formed, which of the following digits will appear twice?

(A)  None

(B)  Only 5

(C)  Both 2 and 3

(D)  Only 3

(E)  Both 1 and 7

Answer: (E)

30. In a certain code language, ‘job requires expertise’ is written as ‘la nu si’. ‘expertise in area’ is written as ‘li bo la’ and ‘requires area inspection’ is written as ‘si dm bo’. How is ‘inspection’ written in that code language?

(All the given codes are two letter codes only)

(A)  si

(B)  Either ‘nu’ or ‘si’

(C)  Either ‘bo’ or ‘si’

(D)  dm

(E)  bo

Answer: (D)

Directions – (Q. 31 to 35) Study the following information to answer the given question.

  Eight persons F, G, H, I, O, P, Q and R are seated in a straight line facing north. Each of them works on different floors of an once building viz., 7th, 16th, 18th, 23rd, 31st, 35th, 44th, 47th. None of the given information is necessarily in the same order.

• O sits fourth to the right of the one who works on the 31st floor. The one who works on the 23rd floor sits second to the right of O.

• Q sits third to the left of I. I is not an immediate neighbour of O. Q does not sit at any of the extreme ends of the line.

• Only two people sit between Q and P. The one who works on the 44th floor sits to the immediate right of H. H is not an immediate neighbour of P.

• The difference between the numerical values of floor numbers in which P and the one to the immediate right of P work is 13.

• Only one person sits between F and the one who works on the 35th floor. F is not an immediate neighbour of I.

• More than two people sit between R and the one who works on the 16th floor. O does not work on the 16th floor.

• H works on a floor lower than O.

31. Which of the following pairs represents the persons seated at the two extreme ends of the line?

(A)  G and the one working on the 7th floor.

(B)  I, R

(C)  The ones working on the 18th and 44th floors.

(D)  The ones working on the 31st and 47th floor

(E)  None of these

Answer: (B)

32. What is the difference between the floor numbers in which P and R work?

(A)  31

(B)  4

(C)  3

(D)  16

(E)  15

Answer: (B)

33. F is related to the one working on the 47th floor following a certain pattern based on the given arrangement. In the same pattern, P is related to the one working on the 44th floor. To who amongst the following is H related to following the same pattern?

(A)  The one working on the 35th floor.

(B)  The one to the immediate left of R.

(C)  The one sitting second to the left of R.

(D)  The one working on the 16th floor.

(E)  O

Answer: (D)

34. Fill in the blanks (respectively in the same order) in order to make the statement correct based on the given arrangement-

G………… and O……….

(A)  works on the 47th floor, sits to the immediate left of O

(B)  works on one of the floors above H, works on one of the floors below F

(C)  sits to immediate left of I, works on the 44th floor

(D)  sits second to the right of Q, work three floors above Q

(E)  Other than those given as options

Answer: (D)

35. How many people sit to the left of the one working on the 35th floor?

(A)  One

(B)  Two

(C)  None

(D)  Four

(E)  Three

Answer: (C)

SBI PO (Pre.) Examination Held on 29-04-2017 English Language Question Paper With Answer Key

SBI PO (Pre.) Examination Held on 29-04-2017 English Language
SBI PO (Pre.) Examination Held on 29-04-2017 English Language Question Paper With Answer Key

SBI PO (Pre.) Examination Held on 29-04-2017

English Language

Directions- Read the following passage carefully and answer the questions given. Certain words/ phrases have been given in bold to help you locate them while answering some of the questions.

  Southern Patagonia, which stretches across Chile and Argentina, has long lured travelers to what is very nearly the end of the world with its legendary peaks carved by age-old glaciers and spellbinding landscapes. Here, in the country’s national parks, are snow-capped mountains, cobalt fjords and old-growth forests. At the southernmost tip of the Americas, icebergs rupture with a dramatic roar from ancient, massive glaciers.

  The Torres del Paine National Park in Chile and Argentina’s Los Glaciares National Park are the region’s top highlights, attracting hundreds of thousands of visitors per year. For a complete Patagonian journey, one must combine visits to both halves of the region. To avoid the crowds and still experience good weather, the best time to visit is during the spring when the flowers are in bloom, or during the fall when the leaves are a fiery mosaic of red orange and yellow. The summer months (December-January) have the mildest weather, but that temperatures rarely go above 70 degrees Fahrenheit and the winds are strong. However, one should be aware that weather in Patagonia is highly unpredictable, particularly in spring and early summer. Weather and temperatures can fluctuate without warning and violent storms can sweep in from the pacific.

  Since distances are quite large in Chile and Argentina, you will need to fly to Patagonia. Driving time is three hours between Puerto Natales and Punta Aenas, two hours between Puerto Natales and Torres del Paine National Park, and five hours between Punta Arenas and the park.

  Travelling between Chile and Argentina can be done easily by land or sea. For an overland trip, you’ll need to organize a private transfer of catch a bus. The drive takes approximately five hours to cross the border between Argentinian Cancha Carrera and Chilean Cerro Castillo pass, all while travelling along the Patagonian steppe. It’s the same route, either direction you’re travelling.

  Since temperatures can vary between 5 and 70 degrees Fahrenheit in the course of a single day, it’s crucial that travelers layer clothing. a waterproof jacket and trekking boots are highly essential, as is sunscreen and a pair of quality sunglasses (the sun is extremely bright).

  In order to pass time, one can stroll through the colourful fishing town of Perto Natales, or explore the region’s labyrinth of scenic fjords, where immense glaciers and unusual marine life can be admired from the deck of a boat. On the shores of Punta Arenas, there are penguin colonies at Seno Otway or Magdalena Island apart from sea lions and whales that populate the waters. One could even go kayaking along the Strait of Magellan, while the highly adventurous cold charter a private board to Clarence Island for skiing in isolation.

  Hotels, offer a staggering array of full and half-day excursion for every interest and fitness level that include mountain trekking, horseback riding, cycling, kayaking, sailing scenic fjords, and glacier cruises, among others Lastly, one of the area’s most popular hikes is the Mirador Las Torres, which crosses the beautiful Pehoe Lake, for view of the French Glacier calving from a natural atmosphere.

1. What does the author mean by ‘with a dramatic roar’ as used in the passage?

(A)  There are many wild animals in the region.

(B)  Glaciers are causing a threat to the region.

(C)  Icebergs separating from the glacier with a spectacular sound.

(D)  Glaciers are unpleasant and an unpleasant sight.

(E)  Natural ice-masses are breath-taking.

Answer: (C)

2. Which of the following is the central idea of the given passage?

(A)  The difficulties one may face while on a trip to Andes.

(B)  The charted boat trips across the boards of Norway.

(C)  Wildlife at the Torres del Paine National Park and its surroundings.

(D)  What all one may come across while touring Southern Ptagonia?

(E)  The importance of hotels in promoting tourism.

Answer: (D)

3. Which of the following is most nearly the opposite of the word ESSENTIAL as used in the passage?

(A)  basic

(B)  requirement

(C)  needless

(D)  abnormal

(E)  good

Answer: (C)

4. Which of the following is most nearly the same meaning to the word RUPTURE as used in the passage?

(A)  sink

(B)  attach

(C)  merge

(D)  explored

(E)  break away

Answer: (E)

5. What can be said about the fjords as mentioned in the given passage?

(A)  Most of their area is used for building houses.

(B)  These are the cause of maximum-cruise boat accidents.

(C)  These are a pleasing sight to watch while cruising on a boat.

(D)  These are not found anywhere else in the world

(E)  None of the given options can be said.

Answer: (C)

6. Which of the following is most nearly the OPPOSITE of the word LURED as used in the passage?

(A)  bellowed at

(B)  perplexed

(C)  constricted

(D)  enticed

(E)  repelled

Answer: (E)

7. Which of the following is most nearly the SAME in meaning as the word SWEEP as used in the passage?

(A)  poor

(B)  run

(C)  drought 

(D)  rush

(E)  clean

Answer: (D)

8. Which of the following is NOT TRUE about Southern Patagonia?

(a)   There is no air connectivity in the given region.

(b)   Kayaking in this region is prohibited by law.

(c)   Weather conditions are very unpredictable in the region.

(A)  Both (a) and (b)

(B)  Only (a)

(C)  Only (b)

(D)  Both (b) and (c)

(E)  All (a), (b) and (c) are false

Answer: (C)

9. Which of the following is a suitable title for the given passage?

(A)  The Beautiful Alps

(B)  The Amazing Amazon

(C)  A Journey to ‘The End of the World’

(D)  The Fluctuating Climate of Patagonia

(E)  Road Trip to Chile

Answer: (C)

10. Which aspect of travel has not been covered in the given passage?

(A)  Lodging

(B)  Trekking

(C)  Transport

(D)  Places of Interest

(E)  Expenses

Answer: (E)

Directions- Read this sentence to find out whether there is any grammatical error on inappropriate usage in it. The error, if any, will be in one part of sentence. Mark the part with the error as your answer. If there is no error, mark ‘No error’ as your answer. Ignore the errors of punctuation, if any.

11. Girul has been earned his (A)/ nickname for fighting (B) / a gang of robbers (C) / almost single-handedly. (D) / No Error (E)

Answer: (A)

12. Researchers have found that (A) / a person’s bones may act as (B) / one of the earliest indicators (C) / brain degeneration in Alzhemeir’s disease. (D) / No Error (E)

Answer: (E)

13. The latest global snapshot of (A) / student performance show decline with scores (B) / in the US and (C) / stagnant performance in science and reading. (D) / No Error (E)

Answer: (B)

14. Growing up in an environment (A) / where filmmaking was an essential aprt (B) / of the household, Ananya Kassarewali was acquainted in (C) / a number of dance forms from an early age. (D) / No Error (E)

Answer: (C)

15. Anti-virus protection kits (A) / were the most (B) / sought-after product (C) / across the country. (D) / No Error (E)

Answer: (B)

16. Inhaling through the nose enhances (A) / memory and response to fearful stimuli (B) / but such affects disappear (C) / when begin to inhale through the mouth. (D) / No Error (E)

Answer: (D)

17. The world’s most largest food company (A) / has developed a process (B) / to alter the structure of sugar that (C) / makes it taste sweetier in small amouns. (D) / No Error (E)

Answer: (A)

18. The drinking water in all (A) / major hospitals in (B) / the metropolis has been (C) / found to be contaminated. (D) / No Error (E)

Answer: (E)

19. Air quality with the city (A) / worsened as pollution levels (B) / in various parts of the city (C) / reached dangerously high levels. (D) / No Error (E)

Answer: (A)

20. The rate of abandonment (A) / for smartwacthes is very high (B) / among the fitness conscious because they (C) / do not seek them useful. (D) / No Error (E)

Answer: (D)

Directions – In the given passage, there are blanks, each of which has been numbered. Against each, five words are suggested, one of which fits the blank appropriately. Find out the appropriate word in each case.

  As hackers wreak havoc with regularity, the insurance industry is being forced to contemplate a whole new set of risks.

  These risks range from the theft of millions of credit card numbers from American retailers …(21)… disabling of a power grid as un Ukraine last December. The dedicated cyber insurance policies that insurance companies ….(22)…. against data braches have become relatively routine. However, it is important to note that the risks they ensure under other types of policies are …(23)…affected by cyber threats and they are still struggling to understand this so-called ‘silent cyber-exposure’ …(24)… that protects firms who suffer data breaches has been on offer for around fifteen  years. We must appreciate that it is much …(25)… to put a precise value on for example, stolen health records than on a property or a car insurer sidestep the problem by …(26)… only the direct costs, that a company incurs from a hack. Typically, these …(27)… hiring a specialized forensics firm to worn out exactly what was stolen and …(28)… affected customers, short-term business interruptions and fines. Data breaches, however, are for the most …(29)… a manageable nuisance rather than a disaster. Despite, the hundreds that take  place annually, only 30 since 2010 have been …(30)… by American companies to regulators as having had a material impact on their business.

21.

(A)  by

(B)  for

(C)  to

(D)  and

(E)  but

Answer: (C)

22.

(A)  fight

(B)  bargain

(C)  oppose

(D)  offer

(E)  extends

Answer: (D)

23.

(A)  too

(B)  similar

(C)  certainty

(D)  also

(E)  unlikely

Answer: (D)

24.

(A)  Whenever

(B)  Unless

(C)  Though

(D)  Policy

(E)  Insurance

Answer: (D)

25.

(A)  earlier

(B)  difficulty

(C)  worst

(D)  harder

(E)  necessary

Answer: (D)

26.

(A)  spending

(B)  safeguard

(C)  covering

(D)  dealing

(E)  conceal

Answer: (C)

27.

(A)  know

(B)  include

(C)  consists

(D)  involves

(E)  account

Answer: (B)

28.

(A)  notifying

(B)  having

(C)  inform

(D)  advice

(E)  acquaint

Answer: (B)

29.

(A)  part

(B)  people

(C)  of

(D)  commonly

(E)  existence

Answer: (B)

30.

(A)  testified

(B)  reported

(C)  conveying

(D)  arrived

(E)  turned

Answer: (B)

State Bank of India PO (Pre.) Examination 2nd Shifting Held on 03-07-2016 Reasoning Question Paper With Answer Key

State Bank of India PO (Pre.) Examination 2nd Shifting Held on 03-07-2016 Reasoning
State Bank of India PO (Pre.) Examination 2nd Shifting Held on 03-07-2016 Reasoning Question Paper With Answer Key

State Bank of India PO (Pre.) Examination 2nd Shifting Held on 03-07-2016

Reasoning

Direction- (Q. 1 to 5) In these questions, relationship between following different elements is shown in the statements. The statements are followed by conclusions. Study the conclusions based on the given statement and select the appropriate answer. Mark answer:

(A) If only conclusion I follows

(B) If only conclusion II follows

(C) If either conclusion I or II follows

(D) If neither conclusion I nor II follows

(E) If both conclusions I and II follow

1. Statements:

G ≥ R > O ≥ S; Y < O ≤ F

Conclusions: (I) G > Y    (II) S ≤ F

Answer: (E)

2. Statements:

W ≥ I = P > E ≥ S

Conclusions: (I) S < W    (II) S = W

Answer: (A)

3. Statement:

R ≤ C ≤ J = T ≤ P > Q ≥ H

Conclusions: (I) R < P     (II) P = R

Answer: (C)

4. Statements:

R ≤ O < A ≤ M; L = D ≥ A

Conclusion: (I) R < L       (II) D > O

Answer: (E)

5. Statements:

R ≤ C ≤ J = T ≤ P > Q ≥ H

Conclusions: (I) H > T     (II) C ≤ Q

Answer: (D)

Directions – (Q. 6 to 11) Study the following information and answer the questions given below it.

  Seven people namely C, D, E, F, G, H and I like different cities namely, Surat, Kolkata, Bangalore, Mumbai, Ranchi, Delhi and Pune. Each of them studies in either of three schools viz. DAS, RIS and VCS with atleast two of them in a school.

(Note: None of the information given is necessarily in the same order.)

  C studies with the one who likes Bangalore in RIS. The one who likes Delhi studies only with H. H does not like Bangalore. C studies with those who like Surat and Pune. C does not study with F. E studies only with the one who likes Mumbai. The one who like Mumbai does not study with the one who likes Delhi. More than one person studies with D. D does not like Pune. Both I and the one who likes Ranchi study in the same school but not in DAS. H does not like Ranchi.

6. Which of the following combinations represents the school in which E studies and the city the likes?

(A)  DAS-Bangalore

(B)  RIS-Bangalore

(C)  DAS-Kolkata

(D)  VCS-Delhi

(E)  PIS-Delhi

Answer: (B)

7. Which of the following cities does G like?

(A)  Pune

(B)  Ranchi

(C)  Mumbai

(D)  Delhi

(E)  Surat

Answer: (D)

8. Four of the following five are alike in a certain way as per the given arrangement and hence form a group. Which of the following does not belong to that group?

(A)  DF

(B)  CI

(C)  HG

(D)  FE

(E)  ID

Answer: (A)

9. Who amongst the following likes Kolkata?

(A)  F

(B)  H

(C)  I

(D)  C

(E)  D

Answer: (B)

10. Which of the following combinations represents the combination of people studying in VCS?

(A)  The one who likes Delhi and Bangalore

(B)  The one who likes Ranchi and Mumbai

(C)  The one who likes Delhi, Mumbai and Kolkata

(D)  The one who likes Pune, Surat and Kolkata

(E)  The one who likes Surat, Pune and Ranchi

Answer: (E)

11. Which of the following statements is not true?

(A)  F studies in the same school in which D studies

(B)  Both D and I do not study in the same school

(C)  G studies with only the one who likes Kolkata

(D)  C likes Ranchi

(E)  All of the given statements are true

Answer: (A)

Directions – (Q. 12 to 14) Study the following information and answer the given questions.

  D is the mother of S. S is the sister of T. T is the mother of R. R is the only son of J. J is the father of U. U is married to K.

12. How is J related to D?

(A)  Brother-in-law

(B)  Son-in-law

(C)  Grandson

(D)  Son

(E)  Nephew

Answer: (B)

13. If R is the father of X, then how is K related to X?

(A)  Father

(B)  Aunt

(C)  Uncle

(D)  Father-in-law

(E)  Mother

Answer: (C)

14. How is S related to R?

(A)  Sister-in-law

(B)  Mother-in-law

(C)  Grandmother

(D)  Aunt

(E)  Mother

Answer: (D)

Directions – (Q. 15 to 19) Study the given information carefully to answer the questions given below-

  Seven boxes – J, K, L, M, N, O and P are kept one above the other, but not necessarily in the same order. Each box contains different elements Cookies, Pencils, Spoons, Diaries, Colours, Jewellery and Watches, but not necessarily in the same order.

  Only two boxes are kept between M and N. The Pencil box is kept immediately below M. Only two boxes are kept between the Pencil box and the Watch box. N is kept above the Watch box. The Diary box is kept immediately below the Watch box. Only three boxes are kept between the Diary box and J. The Jewelley box is kept immediately above the J. O is kept immediately above K. O is not a Pencil box. P is kept immediately above the Cookie box. Only one box is kept between the P and the Spoon box.

15. Which of the following boxes is kept immediately above M?

(A)  P

(B)  O

(C)  L

(D)  The Diary box

(E)  The Jewellery box

Answer: (D)

16. What is the position of O in the given stock of boxes?

(A)  First from the top

(B)  Second from the top

(C)  Third from the bottom

(D)  Fifth from the bottom

(E)  Fourth from the top

Answer: (B)

17. Which of the following boxes contains Spoons?

(A)  Other than those given as options

(B)  K

(C)  M

(D)  N

(E)  L

Answer: (C)

18. Four of the following five are alike in a certain way and hence form a group. Which of the following does not belong to the group?

(A)  N-Diaries

(B)  P-Spoons

(C)  K-Colours

(D)  L-Pencils

(E)  O-Spoons

Answer: (D)

19. How many boxes are kept between K and the Watch box?

(A)  More than three

(B)  None

(C)  Three

(D)  One

(E)  Two

Answer: (B)

Directions – (Q. 20 to 24) Study the following information to answer the question given below-

  Eight friends A, B, C, D, L, M, N and O are seated in a straight line, but not necessarily in the same order. Some of t hem are facing north while some are facing south.

  Only three people sit to the left of N. B sits second to the right of N.

  C sits third to the left of O. O is not an immediate neighbour of B. O does not sit at any of the extreme ends of the line.

  C and O face same direction (i.e. if C faces north, then O also faces north and vice-versa.

  Both the neighbours of D face north, D does not sit at any of the extreme ends of the line.)

  Person sitting at extreme ends face opposite directions (i.e., if one person faces north then the other faces south and vice-versa)

  Both the immediate neighbours of N face same directions (i.e., if one neighbour faces north then the other also faces north and vice-versa.)

  A sits second to the left of L. D faces a direction opposite to L. (i.e., if L faces north then D faces south and vice-versa.)

20. How many people sit exactly between B and O?

(A)  More than four

(B)  One

(C)  Two

(D)  Four

(E)  Three

Answer: (E)

21. Who among the following represent the immediate neighbours of the persons sitting at extreme ends of the line?

(A)  D, O

(B)  C, D

(C)  D, M

(D)  A, L

(E)  A, M

Answer: (A)

22. Who amongst the following faces north?

(A)  A

(B)  L

(C)  O

(D)  N

(E)  C

Answer: (D)

23. Which of the following is true based on the given arrangement?

(A)  B and C face the same directions

(B)  None of the given options is true

(C)  B faces south

(D)  Only three people face south

(E)  M sits at one of the extreme ends of the line

Answer: (E)

24. Who amongst the following sits second to the right of C?

(A)  A

(B)  No one as only person sits to the right of C

(C)  M

(D)  B

(E)  L

Answer: (A)

Directions – (Q. 25 and 26) Read the following information and answer the question given below-

  Point J is 20 m to the north of Point G. Point G is 10 m to the west of Point K. Point K is 15 m to the south of Point L. Ashish is standing at point T which is 30 m to the east of Point L. He starts walking towards south and walks for 35 m. He takes a right turn and stops at Point M after walking for 40 m.

25. How far and in which direction is Point G with respect to point M?

(A)  15 m towards South

(B)  15 m towards West

(C)  20 m towards South

(D)  15 m towards North

(E)  20 m towards North

Answer: (E)

26. If Ashish walks for10 m towards north from his final position to reach Point D, how much distance will he have to cover in order to reach Point J?

(A)  10 m

(B)  35 m

(C)  25 m

(D)  20 m

(E)  30 m

Answer: (E)

Directions – (Q. 27 to 29) Read the given information carefully and answer the questions given below-

  Each of the six friends, I, J, K, L, M and N working in an office handles different number of projects in a month. I handled the second lowest number of projects. K handles more projects than L and M but less than J. J did not handle the maximum number of projects M did not handle the minimum number of projects. The one who handle the third highest number of projects handled 31 projects. L handled 12 projects.

27. How many projects did J possibly handle?

(A)  28

(B)  10

(C)  36

(D)  9

(E)  15

Answer: (C)

28. If the number of projects handled by I + L is only three more than number of projects handled by K how many projects were handled by I?

(A)  22

(B)  14

(C)  19

(D)  9

(E)  32

Answer: (A)

29. Which of the following is true regarding the number of projects handled by N?

(A)  No one handles more projects than N

(B)  Only J handled more number of projects than N

(C)  N possibly handled 24 projects

(D)  N handled more number of projects than only three people

(E)  The difference between projects handled by N and L is less than 18

Answer: (A)

Directions – (Q. 30 to 35) Study the following information and answer the questions given below –

  Seven people, namely K, L, M, N, O, P and Q have to attend a concert but not necessarily in the same order, on seven different months (of the same year) namely January, February, April, May, June, September and November. Each of them also likes a different movie namely (Men, Transformers, Frozen, Minions, Shrek, Tangled and Rio but not necessarily in the same order.

  M will attend a concert in a month which has 31 days. Only two people will attend a concert between M and the one who likes Frozen. The one who likes Frozen will attend a concert on one of the months after. Only two people will attend a concert between the one who likes Frozen and the one who likes Transformers. The one who like.

  Transformers will attend a concert a month which has 31 days K will attend a concert immediately after M. Only three people will attend a concert between K and the one who likes Tangled. Only one person will attend a concert between the one who likes Tangled and L. Only two people will attend a concert between L and the one who like Rio.

  The one who likes X-Men will attend a concert immediately before the one who likes Shrek. Only one person will attend a concert between the one who likes Shrek and P Only three people will attend a concert between Q and O. O will not attend a concert in a month which has 30 days.

30. Who amongst the following likes X-Men?

(A)  M

(B)  K

(C)  O

(D)  L

(E)  Q

Answer: (E)

31. How many people will attend a concert after M?

(A)  More than three

(B)  One

(C)  None

(D)  Two

(E)  Three

Answer: (E)

32. Which of the following represents the people who will attend a concert immediately before and immediately after Q?

(A)  M, N

(B)  P, L

(C)  L, N

(D)  P, N

(E)  N, M

Answer: (B)

33. As per the given arrangement January is related to Q and May is related to K following a certain pattern, which of the following is April related to following the same pattern?

(A)  P

(B)  O

(C)  M

(D)  L

(E)  N

Answer: (C)

34. Which of the following represents the month in which K will attend a concert?

(A)  September

(B)  June

(C)  November

(D)  April

(E)  Cannot be determined

Answer: (B)

35. Who amongst the following likes Minions?

(A)  M

(B)  L

(C)  P

(D)  Other than those given as options

(E)  K

Answer: (E)

SBI PO Main Online Examination Held on June 4, 2017 Question Paper With Answer Key

SBI PO Main Online Examination Held on June 4, 2017 Question Paper With Answer Key
SBI PO Main Online Examination Held on June 4, 2017 Question Paper With Answer Key

SBI PO Main Online Examination Held on June 4, 2017

Part 1 Reasoning and Computer Aptitude

Directions (Q. Nos. 1-3) Study the given information carefully and answer the questions given below :

An input-output is given in different steps. Some mathematical operations are done in each step. No mathematical operation is repeated in next step.

As per the rules followed in the steps given above, find out in each of the following questions the appropriate step for the given input.

1. Find the addition of two numbers obtained in step III?

(a)   1.5

(b)   3

(c)   7

(d)   3.5

(e)   Other than those given as options

Answer: (d)

2. Find the difference between sum to numbers which obtained in 1st step and sum of numbers obtained in all other steps?

(a)   232

(b)   185

(c)   188

(d)   183.5

(e)   Other than those given as options

Answer: (b)

3.Find the multiplication of the numbers obtained in Step II?

(a)   426

(b)   462

(c)   188

(d)   98

(e)   Other than those given as options

Answer: (b)

Directions (Q. Nos. 4-8) Study the following information carefully and answers the questions given below :

There are 10 shelves numbered 1, 2 …….10. They are arranged in two rows one above the other. The shelves 1, 2 ……5 are in row 1 and rest in row 2 which is above row 1. The shelves are arranged in increasing order of number given to them. Like the shelf number 1 is placed on extreme left of row 1, then shelf number 2 and so on. Similarly the shelf number 6 is placed on extreme left of row 2, and so on. Each shelf contains a certain number of glass slabs and photo frames. There is at least one glass slab in each shelf. The length of each glass slab is 15 cm and that of each photo frame is 6 cm.

The shelf 3 has length 33 cm. There is one shelf between shelf 3 and yellow shelf in increasing number of order. The yellow shelf contains 1 glass slab and 6 photo frames more than that in shelf 3. The silver is just above the yellow shelf. The silver shelf contains same number of glass slabs as yellow shelf and 1 photo frame. There are 2 shelves between silver and green shelf. The length of green shelf is 6 cm greater than the silver shelf. The blue shelf is immediate next in number to green shelf. The blue shelf contains 1 glass slab more than that in silver shelf and 1 photo frame less than that in green shelf. There is one shelf between blue and orange shelves. The white shelf is just below the orange shelf. There is one shelf between white and red shelf. Black shelf is in row 2. The pink shelf is just below the black shelf. The black shelf has total number of photo frames and glass slabs as in yellow and blue shelf. The orange shelf has 1 glass slab more than black shelf. The length of orange shelf is 24 cm more than the length of pink shelf. The length of violet shelf is 6 cm more than the length of yellow shelf. The white shelf has greater than or equal to four glass slabs. The length of pink shelf is 3 cm more than the shelf immediate next in number. The length of row 1 is 267 cm and that of row 2 is 249 cm.

4. How many more photo frames can the row 2 adjust?

(a)   1

(b)   2

(c)   3

(d)   4

(e)   Other than those given as options

Answer: (e)

5. The colour of shelf 2 is?

(a)   Violet

(b)   White

(c)   Red

(d)   Pink

(e)   Cannot be determined

Answer: (a)

6. How many total glass slabs do the silver, red and black shelves contain?

(a)   7

(b)   9

(c)   10

(d)   12

(e)   Other than those given as options

Answer: (e)

7. What is the total length of the pink, orange and blue shelves?

(a)   146 cm

(b)   168 cm

(c)   141 cm

(d)   133 cm

(e)   126 cm

Answer: (b)

8. If all photo frames of silver and white shelves are removed and added in black shelf then what will be the length of the black shelf?

(a)   67 cm

(b)   66 cm

(c)   105 cm

(d)   69 cm

(e)   62 cm

Answer: (c)

9. Study the given information carefully to answer the given question.

A few travelers were severely beaten up by people of country X found the movement of the travelers suspicious. The district authority has sent a police team to enquire about the same.

Which of the following inferences can be drawn from the above statement?

(An inference is something which is not directly stated but can be inferred from the given facts.)

(a)   None of those given as options

(b)   Country X is the most preferred tourist destination.

(c)   People of country X are generally suspicious in nature.

(d)   The government of country X generally provides protection to travelers across the country.

(e)   The people of country X dislike presence of strangers in their vicinity.

Answer: (d)

Directions (Q. Nos. 10-12) Study the following information and answer the questions.

A certain number of people are sitting in a straight, horizontal line, facing North. The 17 year old sits fourth from the left end of the line. Only two people sit between the 17 year old and the 40 year old. There are eight people between the 40 year old and the 19 year old. The 5 year old sits fourth to the left of the 19 year old. The 9 year old sits second to the left of the 5 year old. No one sits to the right of the 19 year old.

10. How many people are sitting in the line?

(a)   9

(b)   16

(c)   Cannot be determined

(d)   10

(e)   11

Answer: (b)

11. Which of the following correctly represents the position of the 9 years old in the line?

(a)   8th from the right end

(b)   2nd from the right end

(c)   7th from the left end

(d)   3rd to the right of the 40 year old

(e)   8th from the left end

Answer: (d)

12. If Zara sits exactly between the 5 year old and the 9 year old, how many people sit between Zara and the 40 year old?

(a)   Four

(b)   Five

(c)   Six

(d)   Three

(e)   Two

Answer: (d)

13. Study the given information carefully to answer the given question.

It has been reported in recent years that a very large number of seats in the architecture colleges in the country remain vacant at the end of the admission session.

Which of the following may be the probable cause of the above effect?

(a)   None of those given as options

(b)   There has always been a very poor success rate among the architecture students.

(c)   The government has recently decided to provide post qualification professional training to all architecture graduates at its own cost.

(d)   There has been a considerable decrease in job opening of architecture graduates in the recent years.

(e)   Students have always prepared to complete graduation in three years time instead of four years for architecture.

Answer: (d)

14. Study the given information carefully to answer the given question.

Statement The traffic policeman has been recommended for a suitable reward by his superior in recognition of his sincere duty and busting of several gangs of bikers actively involved in chain snatching.

Which of the following can be an assumption implicit in the above statement?

(a)   Rewards recognizing the sincerity and accomplishments of traffic policeman are given.

(b)   The traffic police desires to be monetarily compensated for his efforts.

(c)   The superior is certain that the recommendation would be denied.

(d)   The supporter wants to set an example for his other juniors by recommending the reward.

(e)   The number of criminals apprehended by this particular traffic policeman was highest ever.

Answer: (d)

Directions (Q. Nos. 15-16) In each question below is given a statement followed by two inferences numbered I and II. An inference is not directly given but can be inferred based on the facts presented. You have to consider the statement and the following inferences and decide which of those is/are implicit in the statement.

15. Statement A leading mobile manufacturer has reintroduced a new model of the existing mobile with modified and peppy looks, but with the same processor leading to increase in sales.

(I) Mobile’s processor is not important as its looks.

(II) Looks of the mobile are one of the important factor to the buyers.

(a)   Both I and II are implicit

(b)   Either I or II is implicit

(c)   Only I is implicit

(d)   Only II is implicit

(e)   Neither I nor II is implicit

Answer: (d)

16. Statement An advertisement in the newspaper Join our computer course to get a high paying job’.

(I) All those who join computer courses get highly paid jobs.

(II) Only a computer course can get someone a high paying job.

(a)   Both I and II are implicit

(b)   Only II is implicit

(c)   Only I is implicit

(d)   Either I or II is implicit

(e)   Neither I nor II is implicit

Answer: (e)

Directions (Q. Nos. 17-20) Study the given information carefully to answer the given questions.

Seven people A, B, C, D, P, Q and R were born in different years- 1988, 1991, 1993, 1996, 1999, 2001 and 2006 and also in different cities- Firozabad, Pune, Indore, Anand, Bhopal, Vellore and Delhi, but not necessarily in the same order.

A was born in an even numbered  year. Only three people were born between A and the one born in Vellore. Only one person was born between B and the one born in Vellore. Only four people were born between B and C. No one was born between C and the one born in Indore. C was born after the one born in Indore. As many people were born between D and the one born in Vellore as between D and the one born in Indore. Only two people were born between D and the one was born between P and Q. Q was born neither in Vellore in Firozabad. The one born in Anand was born in one of the years after R but not in 2006. Only one person was born between the one born in Delhi and the one born in Pune. D was not born in Pune.

17. How many people were born between P and the one born in Pune?

(a)   Three

(b)   More than three

(c)   None

(d)   One

(e)   Two

Answer: (c)

18. In which city was A born?

(a)   Indore

(b)   Bhopal

(c)   Anand

(d)   Other than those given as options

(e)   Pune

Answer: (b)

19. In which year was P born?

(a)   1991

(b)   1999

(c)   1988

(d)   2001

(e)   1993

Answer: (e)

20. As per the given arrangement, R is related to Bhopal in the same way as Q is related to Delhi. Following the same pattern, to which city is D related?

(a)   Indore

(b)   Vellore

(c)   Anand

(d)   None of those given as options

(e)   Pune

Answer: (c)

Directions (Q. Nos. 21-22) Study the information carefully and answer the questions given below.

There are two square fields of different size such that the larger one is surrounding smaller field. Four gates are there for each field in the middle of the sides. Eight people A, B, C, D, E, F, G and H are standing at different gates but not necessary in the same sequence. The persons who are on the sides of larger  park facing centre and the persons who are at side of smaller park facing outside such that inner sides person and outer sides persons are facing each other. There is one person standing between B and D. C faces B. A is to the immediate right of C. G is not the immediate neighbour of D. G faces neither D nor F. One person is standing between H and F. E is facing the centre.

21. Which of the following persons are facing each other?

(a)   BD

(b)   EB

(c)   FH

(d)   DE

(e)   AH

Answer: (d)

22. Four of the following five are alike in a certain way based on a certain arrangement. Which one of the following does not belong to the group?

(a)   EF

(b)   CH

(c)   DA

(d)   FC

(e)   BH

Answer: (e)

Directions (Q. Nos. 23-26) Study the given information carefully to answer the given questions.

Mohan is standing at Point G. He walks 12 m towards East, takes a left turn and walks for 9m to reach point S. He takes a right turn and walks for 5m to reach at a point which is to the South-West of Point J.

Varun, who is standing at Point J, starts walking towards North. He walks for 4m, takes a left turn and stops at Point D after walking for 14m. He turns left and stops at Point R after walking for 8m. Point Z is 9 m North of Point Q and 6m to the West of Point R.

23. If J is 4m North of Y, then what is the distance between Point Y and Mohan’s final position?

(a)   5 m

(b)   16 m

(c)   10 m

(d)   3 m

(e)   Cannot be determined

Answer: (e)

24. Rohit walks 4 m to the North of point Z, takes a right turn and stops at Point G after walking for 6m. Which of the following Point/s will be nearest to Point G?

(a)   Both D and R

(b)   Only S

(c)   Only D

(d)   Both S and J

(e)   Only Q

Answer: (e)

25. In which direction is Point D with respect to Point Q?

(a)   North-East

(b)   West

(c)   South-East

(d)   North

(e)   Cannot be determined

Answer: (a)

26. If Point L is 9 m South of Point R, what will be the distance between Point Q and Point L?

(a)   12 m

(b)   4 m

(c)   8 m

(d)   6 m

(e)   10 m

Answer: (d)

Directions (Q. Nos. 27-31) Study the given information carefully to answer the given questions.

Ten people are sitting in two parallel rows containing five people each. In such a way that there is equal distance between adjacent persons. In row-1 D, E, F, G and H are seated (not necessarily in the same order) and all of them are facing North. In row-2 K, L, M, N and O are seated (not necessarily in the same order) and all of them are facing South. Therefore, in the given seating arrangement each member seated in a row faces another member of the other row. People of each row consist one person of the given ages- 19, 22, 29, 35 and 56 (all ages are in years)j. L sits second to the right of K. Neither O nor K sit at any of the extreme ends of the row. The person sitting to the immediate right of the one who faces K is 29 years old. D sits third to the left of the 29 year old. One of the immediate neighbours of the one who faces D is 35 years old. O sits second to the left of the 35 year old. The one who sits to the immediate left of O faces the 19 year old. In row-1, only two people sit between the 19 year old and the 29 year old. E sits third to the right of F. E faces the 56 year old. G and K face each other. One of the immediate neighbours of the one who faces M is 56 years old. The 29 year old person of row 1 faces the 56 years old. N’s age is an even number.

27. Who sits second to the left of E?

(a)   G

(b)   The 56 year old

(c)   H

(d)   The one who faces K

(e)   The 19 year old

Answer: (a)

28. Which of the following pairs represent the people who are of same age?

(a)   M, D

(b)   L, E

(c)   O, H

(d)   K, F

(e)   N, G

Answer: (d)

29. One of the people whose age is 19 years of row-2 faces which of the following person?

(a)   G

(b)   K

(c)   E

(d)   N

(e)   L

Answer: (d)

30. What is the age of M (in years)?

(a)   35

(b)   19

(c)   56

(d)   22

(e)   29

Answer: (a)

31. Which of the following statement is TRUE with respect to the given information ?

(A) H is 29 years old.       (B) N faces the 22 year old.

(C) F sits to the immediate right of D.

(a)   Only B

(b)   A and B

(c)   All of these

(d)   Only A

(e)   A and C

Answer: (e)

Directions (Q. Nos. 32-35) Study the given information carefully to answer the given questions.

Six people A, B, C, D E and F like six different colours namely, Blue, Green, Silver, Yellow, Orange and Pink, but not necessarily in the same order. Each one of them also studies in one of the three Standards-I, IV and IX. Only two people study in each of the given standards. F studies in an odd numbered standard with the one who likes Silver.

The one who likes Blue is in a higher standard than D. D does not like Silver. F does not like Pink.

Only two people are in a  higher standard than A. A does not like Pink.

The one who likes Yellow and the one who likes blue study in the same standard. D does not like Yellow. C likes green. E does not study in the standard IX.

32. Four of the following five are alike in a certain way based on the given arrangement and hence form a group. Which one of the following does not belong to the group?

(a)   B-Green

(b)   E-Orange

(c)   A-Blue

(d)   A-Silver

(e)   D-Yellow

Answer: (c)

33. Which of the following statements is not true with respect to the given arrangement?

(a)   The one who likes Green studies in Standard I.

(b)   B is in a higher standard than C.

(c)   E likes Pink.

(d)   C and A study in the same standard.

(e)   None of the given information is true.

Answer: (c)

34. Who amongst the following likes Orange?

(a)   B

(b)   D

(c)   Other than those given as options

(d)   E

(e)   A

Answer: (c)

35. In which standard does C study?

(a)   The one in which E studies

(b)   IX

(c)   The one in which A studies

(d)   The one in which the one who likes Blue studies

(e)   I

Answer: (e)

36. Study the given information cerefjlly to answer the given question.

There has been a spatte of bus accidents in Country C in the recent months killing large numbers of passengers and injuring many. This has raised serious doubts about the road transport department of the country capability of providing safety to travelers.

Which of the following statements substantiates the views expressed in the above statement?

(a)   Local residents are always the first to provide a helping hand to the passengers in the event of such disasters.

(b)   None of those given as options

(c)   People have no option other than travelling by bus over long distances.

(d)   Most of the bus routes of Country C have been found to be stressed due to wear and tear in the recent times.

(e)   The road transportation of the country has recently come to be known as the safest mode by a national survey.

Answer: (e)

Directions (Q. Nos. 37-41) Read the following information carefully and answer the question given below.

There are 6 cars P, Q, R, S, T and U which are parked in a straight line. But adjacent cars cannot be alphabetically placed, like car cannot be parked adjacent to car Q. Car Q cannot be parked adjacent to car P and R and so on. Distance between each car is successive multiple of 4. Distance  between cars P and Q is 60 m. Car P is to the left of car Q. There is no car between car P and car Q. Distance between cars Q and T is 84 m. Distance between cars R and U is a multiple of 3. Car U is parked somewhere right of car R.

From a point, car V moves 16 m East, takes a right turn, moves 12 m and stops at Point Z. Pont Z is 15 m North of car P.

If car U goes 7 m in South direction, takes a left turn and moves 16 m, then it turns right and moves 5 m, next takes a left turn again and moves 22 m, then it reaches to point X.

37. How many cars are parked between cars P and S?

(a)   One

(b)   None

(c)   Three

(d)   Two

(e)   Four

Answer: (d)

38. What is the distance between car Q and Point X?

(a)   13 m

(b)   2√5 m

(c)   6√2 m

(d)   14 m

(e)   6√5 m

Answer: (e)

39. Car R will have to move how much distance and in which directions to reach to car V?

(a)   15 m North, 38 m East

(b)   24 m East, 17 m North

(c)   44 m East, 15 m North

(d)   17 m North, 38 m East

(e)   17 m North, 44 m East

Answer: (c)

40. Car S will have to move much distance and in which directions to reach to point X?

(a)   10 m West, 30 m South

(b)   12 m South, 30 m West

(c)   12 m South, 30 m East

(d)   32 m West, 10 m West

(e)   36 M West, 10 m South

Answer: (b)

41. If car V moves 28 m East from Point Z, takes a right turn and stops at point Y after moving 17 m, then car Q is in which direction with respect to point Y?

(a)   North-West

(b)   South-East

(c)   Cannot be determined

(d)   North-East

(e)   South-West

Answer: (d)

Directions (Q. Nos. 42-45) Study the given information carefully to answer the given questions.

Five boxes viz. E, F, G, H and I are placed above one another. Each box is of a different colour viz. Pink, Blue, Grey, White and Black. The grey box is kept immediately above H. Only two boxes are kept between the pink and the blue boxes. The pink box is kept at one of the positions above the blue box as well as H. More than one box is kept between the blue box and e. E is not pink in colour. As many boxes are kept between E and they grey box as between G and the white box. H is not white in colour. F is kept at one of the positions below the pink box but not immediately below it.

42. How many boxes are kept between I and the black box?

(a)   Cannot be determined

(b)   Three

(c)   None

(d)   One

(e)   Two

Answer: (d)

43. Which of the following is/are true with respect to the given arrangement?

(1) There are only two boxes between E and the blue box.

(2) F is placed at the bottom-most position.

(3) There is only one box between E and the grey box.

(a)   1 and 2

(b)   2 and 3

(c)   Only 1

(d)   Only 2

(e)   Only 3

Answer: (b)

44. Which of the following boxes is black in colour?

(a)   H

(b)   I

(c)   E

(d)   G

(e)   Cannot be determined

Answer: (a)

45. Which of the following represents the colour of i?

(a)   Grey

(b)   White

(c)   Black

(d)   Blue

(e)   Pink

Answer: (e)

Part 2 Data Analysis and Interpretation

46. PQR is a triangle and Qs is altitude.

Quantity I. Value of Side PS

II. Value of side SR

(a)   Quantity I > Quantity II

(b)   Quantity I < Quantity II

(c)   Quantity I ≤ Quantity II

(d)   Quantity I = Quantity II

(e)   Quantity I = Quantity II or No relation

Answer: (e)

47. Quantity I. Value of 

II. Value of 228

(a)   Quantity I > Quantity II

(b)   Quantity I < Quantity II

(c)   Quantity I ≤ Quantity II

(d)   Quantity I ≥ Quantity II

(e)   Quantity I = Quantity II or No relation

Answer: (e)

48. Quantity I. Product of odd integers from -5 to 13 inclusive.

II. Product of even integers from -12 to 6 inclusive.

(a)   Quantity I > Quantity II

(b)   Quantity I < Quantity II

(c)   Quantity I ≤ Quantity II

(d)   Quantity I ≥ Quantity II

(e)   Quantity I = Quantity II or No relation

Answer: (b)

49. RSTU is a parallelogram and SU is a diagonal of the parallelogram.

Quantity I. Value of < x

II. Value of < y

(a)   Quantity I > Quantity II

(b)   Quantity I < Quantity II

(c)   Quantity I ≤ Quantity II

(d)   Quantity I ≥ Quantity II

(e)   Quantity I = Quantity II or No relation

Answer: (e)

50. Five men and five women are to be arranged in a row while seating in a party.

Quantity I. Number of ways of arranging 5 men and 5 women such that no two men or women are adjacent to each other.

II. Number of ways of arranging 5 men and 5 women such that all men sit together.

(a)   Quantity I > Quantity II

(b)   Quantity I < Quantity II

(c)   Quantity I ≤ Quantity II

(d)   Quantity I ≥ Quantity II

(e)   Quantity I = Quantity II or No relation

Answer: (b)

Directions (Q. Nos. 51-60) Study the following pie-chart and table carefully and answer the questions that follow.

Some values are missing. Answer the questions on the basis of given table and information in question. Speed of stream is same for both upstream and downstream distance on respective days

51. Time taken to cover the upstream distance on Friday is same as time taken to cover the downstream distance on Thursday. Total speed of still water on Thursday and Friday is 10 km/h. Find the ratio of speed of still water on Thursday and Friday.

(a)   6 : 13

(b)   4 : 11

(c)   7 : 13

(d)   4 : 15

(e)   7 : 12

Answer: (e)

52. On Monday, the boat takes a total of 4 hours 30 minutes to cover both upstream and downstream distance. Ratio of speed of boat in still water in going upstream to downstream is 4 : 5. Find the speed of boat in still water while going downstream.

(a)   13 km/h

(b)   15 km/h

(c)   9 km/h

(d)   10 km/h

(e)   12 km/h

Answer: (b)

53. On Tuesday, ratio of speed of boat in still water in going upstream to downstream is 3 : 8. Also difference in speed of boat in still water in going upstream and downstream is 5 km/h. If the total time taken by boat to cove upstream and downstream distance is 14 hours on Tuesday, find the speed of stream.

(a)   2 km/h

(b)   1 km/h

(c)   3 km/h

(d)   1.5 km/h

(e)   2.5 km/h

Answer: (b)

54. On Wednesday, ratio of speed of boat in still water in going upstream to downstream is 4 : 5. The difference between time to cover upstream distance and downstream distance is 5 hours, find the total time taken to cover upstream distance and downstream distance.

(a)   5 h

(b)   4 h

(c)   7 h

(d)   8 h

(e)   6 h

Answer: (d)

55. Time taken to cover the upstream distance on Thursday is 12 hours more than time taken to cover the downstream distance on Friday. Total speed of still water on Thursday and Friday is 11 km/h. What will be the ratio of speed of still water on Thursday and Friday?

(a)   6 : 7

(b)   4 : 5

(c)   7 : 1

(d)   5 : 6

(e)   7 : 8

Answer: (c)

56. MLKJ is a trapezium. ABCD and PQRS are two rhombus. Diagonals of ABCD are 6 cm and 8 cm. One of the angle of PQRS is 120° and diagonal bisecting that angle measures 15 cm. Side of ABCD = ML, Side of PQRS = JK. Find ON (median of trapezoid).

(a)   10 cm

(b)   12 cm

(c)   8 cm

(d)   15 cm

(e)   9 cm

Answer: (a)

Directions (Q. Nos. 57-61) Study the information carefully to answer the question that follows.

A school consisting of 1560 students has boys and girls in the ratio of 7 : 5 respectively. All the students are enrolled for different country tours viz. Russia, Switzerland and Japan. One-fifth of the boys are enrolled for only Switzerland tour, 20% of the girls are enrolled for only Japan tour, 10% of the boys are enrolled for only Russia tour, 24% of the girls are enrolled for both Russia and Switzerland tour together. The number of girls enrolled for only Russia tour is 200% of the boys enrolled in the same. One-thirteenth of the boys are enrolled for all the three tour together. The respective ratio of boys enrolled for Switzerland and Japan tour together to the girls enrolled for the same is 2 : 1 respectively, 10% of the girls are enrolled for only Switzerland tour where as 8% of the girls are enrolled for both Switzerland and Japan tour together. The remaining girls are enrolled for all the three tour together. The number of boys enrolled for Russia and Switzerland tour together is 50% of the number of girls enrolled for the same. The remaining boys are enrolled for only Japan tour.

57. What is the respective ratio of the number of girls enrolled for only Japan tour and number of boys enrolled for Switzerland tour together to the number of boys enrolled for the Japan tour and number of girls enrolled for Switzerland tour together?

(a)   77 : 52

(b)   75 : 52

(c)   52 : 75

(d)   52 : 77

(e)   None of these

Answer: (c)

58. Number of girls enrolled for only Switzerland, Russia and Japan tour together is, what percent of the boys together enrolled for the same? (rounded off to two digits after decimal)

(a)   37.57%

(b)   57.29%

(c)   61.93%

(d)   55.22%

(e)   54.26%

Answer: (b)

59. What is the total number of students enrolled for all the three tour and number of girls enrolled for Switzerland and Russia tour together and number of boys enrolled for Switzerland and Japan tour together?

(a)   395

(b)   2415

(c)   385

(d)   390

(e)   393

Answer: (a)

60. What is the sum of the total number of boys who enrolled for Switzerland and the total number of girls who enrolled for same tour together?

(a)   872

(b)   752

(c)   802

(d)   772

(e)   None of these

Answer: (e)

61. Total number of girls enrolled for Russia is approximately what percent of the total number of students for the school?

(a)   37%

(b)   19%

(c)   32%

(d)   14%

(e)   26%

Answer: (d)

Directions (Q. Nos. 62-64) Study the given information and answer the following question.

There are three bags A, Band C. In each bag there are three types of coloured balls Yellow, green and Black. In bag A, number. of yellow coloured balls are y and number of green coloured balls are g. Number of green coloured balls are 4 more than the number of yellow coloured balls. When one ball is picked at random then the probability of getting black colour ball is 5/13. The value of y is 18 2/11% less than g.

In bag B, number of yellow coloured balls is 22 2/9% more than that of bag A. If two balls are picked at random from bag B then the probability of getting both green colour ball is 4/37. Total number of balls in bag B is 75.

In bag C, the ratio of number of green coloured balls and number of black coloured balls is 7 : 5. Total number of green and black coloured balls is 36. If one of the  balls is picked at random then the probability of getting one yellow ball is 7/13.

62. If x number of yellow balls from bag b are taken and p laced into bag a and 20% of black balls from bag A are taken and placed into in bag B. If we pick one ball from bag B then the probability that the ball is of black colour is 11/26. then find the value of x?

(a)   5

(b)   3

(c)   2

(d)   6

(e)   None of these

Answer: (b)

63. If one ball picked at random from each of the bag A and bag B then find the probability that both of the balls are of the same colour?

(a)   (21 × 47) / (65 × 75)

(b)   (22 × 43)/ (65 ×75)

(c)   (11 × 17) /(65 × 75)

(d)   (22 × 47) / (65 × 75)

(e)   Cannot be determined

Answer: (e)

64. Difference between the number of green balls in bag A and bag C is how much percent more/less than the sum of the number of black balls in bag A and bag C together?

(a)   100%

(b)   95%

(c)   97.5%

(d)   102.5%

(e)   Other than those given as options

Answer: (d)

Directions (Q. Nos. 65-69) Study the following table and answer the following questions.

The following table is related to profit and loss and some values are missing. All the discounts are on the Marked Price (MP) and the Profits are on Cost Price (CP).

65. If the ratio of the Cost price of the article M and article Q is 3 : 4, then what will be the discount on article Q?

(a)   12%

(b)   15%

(c)   8%

(d)   10%

(e)   20%

Answer: (d)

66. If the discount (in %) and profit (in %) of the article P is same, then what will be the marked price of article P?

(a)   Rs 2800

(b)   Rs 2200

(c)   Rs 3500

(d)   Rs 3000

(e)   Rs 4000

Answer: (b)

67. If the marked price of article M is Rs 130 more than the cost price of article N and the difference between marked price and selling price of article N is 684, then what will be the discount on article N?

(a)   18%

(b)   25%

(c)   30%

(d)   20%

(e)   19%

Answer: (e)

68. Cost price of article P is how much percent less than the marked price of article M?

(a)   26%

(b)   25.04%

(c)   29.68%

(d)   31%

(e)   28.02%

Answer: (c)

69. The ratio of discount (in %) and profit (in %) is 4 : 3 of the article O. Find cost price of article O.

(a)   Rs 1200

(b)   Rs 2800

(c)   Rs 3500

(d)   Rs 2000

(e)   Rs 2500

Answer: (d)

Directions (Q. Nos. 70-74) Study the following graph and answer the following questions.

A, B, C, D and E are five persons employed to complete a job X.

Note 1. All person worked on the job X for whole number days.

Note 2 Two jobs Y and Z are similar to job X and require same effort as required by job X.

70. A and C worked on job Y working alternatively for 10 days. B and D then worked together for x days. If 1/36 of the job was still remained, then find the value of x?

(a)   2 days

(b) 

(c) 

(d) 

(e)   1 day

Answer: (e)

71. E worked on job Z for 5 days and the remaining job was completed by A, B and D who worked on alternate days starting with A followed by B and D in that order. What will be the number. of days B has worked for?

(a)   2

(b)   4

(c)   9

(d)   3

(e)   None of these

Answer: (e)

72. If A, C and E worked on job Z for 2 days each and the remaining job was done by B and D. If the ratio of number of. days for which B and D worked is in ratio 20 : 21, then what is the number of days for which B worked?

(a)   50 days

(b) 

(c) 

(d)   4 days

(e)   None of these

Answer: (d)

73. If the ratio of number of days for which B and D worked on job X is 4 : 3, then find the difference between number of days for which B and D worked?

(a)   2 days

(b)   3 days

(c)   1 day

(d)   4 days

(e)   None of these

Answer: (a)

74. If C worked on job Y with 5/4 times his given efficiency and was assisted by B every 3rd day, then find the time taken by C to complete the job?

(a)   13 days

(b) 

(c) 

(d)   12 days

(e)   None of these

Answer: (d)

75. What is principal amount?

Statement

(I) The simple interest obtained on the principal after 2 years at 8% rate of interest is Rs 450 less than compound interest obtained on the principal after 2 years at 8% per annum.

(II) The sum becomes double in 10 years at 6% per annum rate of simple interest.

(III) The compound interest obtained on the principal amount is Rs 4540 after 2 years at the rate of 8% compounded annually.

(a)   Only I

(b)   All I, II and III together

(c)   II and III together

(d)   Only I or III

(e)   I or II or III

Answer: (d)

76. What is the speed of boat in still water ?

Statements

(I) The boat can cover 12 km downstream distance in 2 hours.

(II) Speed of the stream is three-fourth the speed of boat in still water.

(III) The boat can cover 12 km upstream distance in 4 hours.

(a)   I and III together

(b)   Only II

(c)   I and II together

(d)   I and either II or III

(e)   Any of the two statements

Answer: (a)

77. A person borrowed some money at compound interest for 2 years. What will be the amount required to return after 2 years?

Statements

(I) If the amount was borrowed at simple interest, then after 5 years Rs 600 was required to pay as simple interest.

(II) The rate of interest is 6% per annum.

(III) The sun of money borrowed is 10 times the amount required to be paid as simple interest after 2 years.

(a)   II and I or III

(b)   II and III together

(c)   I and III together

(d)   All statements are not sufficient to answer the question.

(e)   All of these

Answer: (a)

78. Shopkeeper gained how much by selling his products in November, 2016?

Statements

(I) He earned 40% more profit in December, 2016 as compared to October, 2016.

(II) In December, 2016 he earned 10% more profit than in November 2016.

(III) The total profit earned in November, 2016 and October, 2016 was Rs 55000.

(a)   I and III

(b)   II and III

(c)   I and II

(d)   All of these

(e)   All statements are not sufficient to answer the question.

Answer: (d)

79. In how many days 6 men and 5 women can done the work working together?

Statements

(I) The ratio of efficiency of man to woman is 2 : 1.

(II) 9 women can complete three-five of the work in 15 days.

(III) 6 men and a child together can complete 1/4th work in 12 days while 6 women and the child together can complete two-third of the work in 16 days.

(a)   I and either II or III

(b)   Any of the two statements

(c)   I and II

(d)   All of these

(e)   None of these

Answer: (c)

80. A vessel contains 2.5 L of water and 10 L of milk 20% of the contents of the vessel are removed. To the remaining contents,, x L of water is added to reverse the ratio of water and milk. Then y L of milk is added again to reverse the ratio of water and milk. What is the value of y?

(a)   100 L

(b)   110 L

(c)   120 L

(d)   130 L

(e)   None of these

Answer: (c)

Part 3 General Awareness

81. Kavinder Singh and Satish Kumar is related to which game?

(a)   Boxing

(b)   Cricket

(c)   Hockey

(d)   Football

(e)   Badminton

Answer: (a)

82. As per report of Stockholm International Peace Research Institute (SIPRI) India’s military expenditure in 2016 grew by

(a)   9.5%

(b)   8.5%

(c)   8.7%

(d)   9.3%

(e)   9.1%

Answer: (b)

83. What is the meaning of ‘C’ in PCA?

(a)   Prompt Collateral Action

(b)   Prompt Corrective Action

(c)   Prompt Credit Action

(d)   Prompt Cumulative Action

(e)   Prompt Competitive Action

Answer: (b)

84. India Licesing Expo (ILE), India’s first and most influential brand licensing show will take place of August 20-21 in

(a)   New Delhi

(b)   Mumbai

(c)   Kolkata

(d)   Chennai

(e)   Bengaluru

Answer: (b)

85. Who is highest wicket-taker in women’s ODIs?

(a)   Mithali Raj

(b)   Neetu David

(c)   Jhulan Goswami

(d)   Anum Chopra

(e)   Anju Jain

Answer: (c)

86. The Insolvency and Bankruptcy Board of India (IBBI) has constituted a Technical Committee in accordance with Regulation 14 of the IBBI (Information Utilities) Regulations 2017. The Technical Committee (Committee) will be headed by?

(a)   Mukulita Das

(b)   Navrang Saini

(c)   Dr. MS Sahoo

(d)   Dr. RB Barman

(e)   Shipra Shetty

Answer: (d)

87. India’s growth for 2018-19 as projected by IMF?

(a)   7.1%

(b)   7.7%

(c)   7.5%

(d)   7.4%

(e)   7.2%

Answer: (d)

88. National Financial Switch (NFS) is run by

(a)   NPCI

(b)   Reserve Bank of India

(c)   National Information Centre

(d)   Institute of Development and Research in Banking

(e)   None of the above

Answer: (a)

89. Who was elected as head of FIFA’s governance committee at the world football governing body’s 67th Congress in Baharian’s capital city of Manama?

(a)   Justice Dipak Misra

(b)   Justice Jasti Chelameswar

(c)   Justice Mukul Mudgal

(d)   Justice Madan Bhimarao Lokur

(e)   Justice Utpal Sahu

Answer: (c)

90. Foreign Direct Investment (FDI)in India grew 18% during 2016 to touch ……… data released by the Department of Industrial Policy and Promotion (DIPP).

(a)   $ 53 billion

(b)   $46 billion

(c)   $57 billion

(d)   $45 billion

(e)   $54 billion

Answer: (b)

91. Fiscal deficit for 2017-18 pegged at …… of GDP as per Union Budget 2017-18.

(a)   4.6%

(b)   4.2%

(c)   3.2%

(d)   3.8%

(e)   4.4%

Answer: (c)

92. The government has decided to make tax evasion of over ……. under the proposed Goods and Services Tax (GST) the regime as non-bailable offence.

(a)   Rs 2 crore

(b)   Rs 3 crore

(c)   Rs 5 crore

(d)   Rs 10 crore

(e)   Rs  1 crore

Answer: (c)

93. What is the meaning of ‘S’ in MSF?

(a)   Marginal Standing Facility

(b)   Marginal Stock Facility

(c)   Marginal Share Facility

(d)   Marginal Special Facility

(e)   Marginal Saving Facility

Answer: (a)

94. What is the full form of APBS?

(a)   Aadhaar Post Bridge System

(b)   Aadhaar Post Banking System

(c)   Aadhaar Payment Banking System

(d)   Aadhaar Payment Bridge System

(e)   Aadhaar Post Bureau System

Answer: (d)

95. The target of digital transactions through UPI, USSD, Aadhaar, IMPS for 2017-18?

(a)   Rs 2500 crore

(b)   Rs 5000 crore

(c)   Rs 7500 crore

(d)   Rs 10000 crore

(e)   Rs 6000 crore

Answer: (a)

96. Cheque written by drawer and dated at some point in past is called?

(a)   Ante-dated cheque

(b)   Post Dated Cheque

(c)   Stale Cheque

(d)   All of these

(e)   None of these

Answer: (a)

97. As per RBI norms on small banks, initial promoter stake for first 5 years should not be less than?

(a)   30%

(b)   15%

(c)   20%

(d)   35%

(e)   40%

Answer: (e)

98. Consider the following statements about Unified Payments Interface (UPI) :

(A) It seeks to promote cashless economy in the country

(B) It has been developed by National Payments Corporation of India (NPCI)

(C) It facilitates the user to have a single virtual address for multiple bank accounts

Which of the above statements is/are correct?

(a)   Only A

(b)   A and B

(c)   All of these

(d)   A and C

(e)   None of these

Answer: (c)

99. The base year for calculating key economic growth from 2004-05 changed to?

(a)   2010-11

(b)   2011-12

(c)   2014-15

(d)   2015-16

(e)   2013-14

Answer: (b)

100. Which one of the following is the rate at which the RBI lends money to commercial banks in the event of any shortfall of funds?

(a)   Benchmark Prime Lending Rate

(b)   Annual Percentage Rate

(c)   Bank Rate

(d)   Repo Rate

(e)   Interest Rate

Answer: (d)

101. Finland has become the first country in Europe to pay its unemployed citizens a basic monthly income, amounting to

(a)   $ 587

(b)   $ 636

(c)   $ 437

(d)   $ 735

(e)   $ 235

Answer: (a)

102. Total number of ATMs of SBI after the merger?

(a)   69000 ATMs

(b)   59000 ATMs

(c)   79000 ATMs

(d)   89000 ATMs

(e)   99000 ATMs

Answer: (b)

103. The Government of India (GOI) has approved Rs 681 crores as seed capital for building a venture for electronic devices with total corpus?

(a)   Rs 6535 crore

(b)   7137 crore

(c)   Rs 6831 crore

(d)   Rs 8323 crore

(e)   Rs 7931 crore

Answer: (c)

104. How much amount has allocated by the government to MGNREGA in this year’s (2017-18) Budget?

(a)   Rs 56000 crore

(b)   Rs 48000 crore

(c)   Rs 60000 crore

(d)   Rs 62000 crore

(e)   Rs 68000 crore

Answer: (b)

105. Risk of collapse of entire financial system on entire market is called

(a)   Systemic risk

(b)   Basis risk

(c)   Yield curve risk

(d)   Embedded option risk

(e)   Mismatch risk

Answer: (a)

106. Payments bank not permitted to

(a)   offer loans

(b)   offer interest on deposits

(c)   cash back offer

(d)   offer saving account

(e)   offer current account

Answer: (a)

107. The NBFCs are allowed to accept/renew public deposits for a minimum period of 12 months and the maximum period of …….. months.

(a)   18 months

(b)   24 months

(c)   36 months

(d)   48 months

(e)   60 months

Answer: (e)

108. The government of India has changed CII base year from 1981 to

(a)   1991

(b)   2001

(c)   2011

(d)   2010

(e)   2015

Answer: (b)

109. Metric which defines operating liquidity available to business is known as?

(a)   Gross Capital

(b)   Temporary Capital

(c)   Market Capital

(d)   Net Capital

(e)   Working Capital

Answer: (e)

110. An estimated $ ………… in black money entered India during 2005-2014, US-based think tank Global Financial Integrity(GFI).

(a)   770 billion

(b)   1215 billion

(c)   565 billion

(d)   931 billion

(e)   836 billion

Answer: (a)

111. Name the person, who will head a task force to assess the extent of employment creation at different points of time, based on ‘credible data’.

(a)   Ashok Rahane

(b)   Subhash Dogra

(c)   Vipin Ahlawat

(d)   Mohd. Nasim Zaidi

(e)   Arvind Panagriya

Answer: (e)

112. Institute for Development & Research in Banking Technology (IDRBT) is located in

(a)   Mumbai

(b)   Bengaluru

(c)   Chennai

(d)   Kolkata

(e)   Hyderabad

Answer: (e)

113. Class of derivative whose value is at least partly derived from one or more underlying stock exchange securities? (Trade, Commodity, Credit, Interest Rate, Equity)

(a)   Equal Derivative

(b)   Stock Derivative

(c)   Equity Derivative

(d)   Supply Derivative

(e)   Equal Value Derivative

Answer: (c)

114. The framework of Joint Lenders’ Forum (JLF) by RBI is to strengthen the system for revitalizing the

(a)   fixed assets in the economy

(b)   stressed assets in the economy

(c)   linked assets in the economy

(d)   basic assets in the economy

(e)   None of the above

Answer: (b)

115. Amendment to Section …….. of Negotiable Instruments Acts to include the electronic image of a truncated cheque and cheque in electronic form.

(a)   5

(b)   6

(c)   7

(d)   8

(e)   3

Answer: (b)

116. The target for agricultural credit in 2017-18 has been fixed at a record level of

(a)   Rs 10 lakh crore

(b)   Rs 2 lakh crore

(c)   Rs 5 lakh crore

(d)   Rs 15 lakh crore

(e)   Rs 20 lakh crore

Answer: (a)

117. 2017 Index of Economic Freedom is topped by which country?

(a)   Singapore

(b)   Germany

(c)   India

(d)   China

(e)   Hong Kong

Answer: (e)

118. As per RBI, ARCs must have the minimum net corpus of …….. crore by 2019.

(a)   Rs 100 crore

(b)   Rs 200 crore

(c)   Rs 300 crore

(d)   Rs 500 crore

(e)   Rs 1000 crore

Answer: (a)

119. The UN Day of Vesak Celebration is internationally a cultural and humanistic festival. 14th UN Vesak Day celebrated in which country?

(a)   Bhutan

(b)   Sri Lanka

(c)   India

(d)   Nepal

(e)   Mali

Answer: (b)

120. Using SBI Mingle, the bank’s customers can do a host of banking services on their …………accounts at their own convenience.

(a)   Facebook or Twitter

(b)   Facebook or Pinterest

(c)   Facebook or Linkedin

(d)   Facebook or Instagram

(e)   Facebook or Google+

Answer: (a)

Part 4 English Language

Directions (Q. Nos. 121-125) In each question given below a sentence is given and is divided into three parts I, II and III. For each part a correction statement is also given. You have to determine which part requires correction and mark it as your answer.

121. Today’s Britain find itself in a different era./ The vote for Brexit has committed it to leaving its biggest trading partner/ and snuggling closer to others, while including a less-welcoming America.

(I) Today Britain finds itself in a different era.

(II) The vote for Brexit has been committed it to leaving its biggest trading partner.

(III) and snuggling closer to others, including a less-welcoming America.

(a)   Only I

(b)   Both I and III

(c)   Both II and III

(d)   Both I and II

(e)   Only II

Answer: (b)

122. One reason Donald Trump is inviting acres of commentary / is that he keeps the world guessing what he means/ and where his foreign policy is heading.

(I) One reason Donald Trump invites acres of commentary

(II) is that he kept the world guessing what he means

(III) and that his foreign policy is heading.

(a)   Both I and II

(b)   Both II and III

(c)   Only II

(d)   Only I

(e)   All of these

Answer: (d)

123. In some rich countries ex-smokers now outnumber those who still puffed on./ But in many poor countries smoking/ is on the rise, ,particularly as per men.

(I) In some rich countries ex-smokers now outnumber those who still puff on.

(II) Likewise in many poor countries smoking

(III) is on the rise, particularly among men.

(a)   I and II

(b)   II and III

(c)   I and III

(d)   Only I

(e)   All of these

Answer: (c)

124. As the individual and group performance picture/ became clear to Paula, she began to pine for the/ simplicity of her life as an individual contributor.

(I) Hence the individual and group performance picture

(II) became clear to Paula, she assumed to pine for the

(III) Simpleness of her new life as an individual contributor.

(a)   Only I

(b)   Only II

(c)   I and II

(d)   II and III

(e)   None of these

Answer: (e)

125. When you have approached your colleagues and boss with the business case/ for making changes, be sure not to come across as indignant/ or self-righteous – after all, they may be part of the problem.

(I) When you approach your colleagues and boss with the business case

(II) for making changes, be sure as not come across as indignant

(III) or self-righteous- after all, it can be part of the problem.

(a)   Only II

(b)   Only I

(c)   I and II

(d)   II and III

(e)   None of these

Answer: (b)

126. (I) Today, health systems operate on a ……….. of how involved patients are in the delivery of their ……….. .

(II) The board members of the telecom company have disregarded the …….. issue because they ……… more about profit.

(a)   factor; service

(b)   spectrum ; care

(c)   guideline; factor

(d)   comprehension ; care

(e)   variety ; thought

Answer: (b)

127. (I) As other industries have become more ……. in detecting and blocking cyberattacks, criminals have had to find new ………. of data.

(II) Only ……… votes are able to differentiate between information ………. .

(a)   predominant ; outcome

(b)   developed ; change

(c)   sophisticated ; sources

(d)   advanced ; boom

(e)   reliable ; technology

Answer: (c)

128. (I) A new have of analytics and technology could help ……. cut costly and unnecessary hospitalizations while improving …….. for patients.

(II) The unified, streamlined approach ………… reduces the amount of work required to implement Learning ……….. through the intelligent reuse of assessment workflows in the grading process.

(a)   efficiently ; knowledge

(b)   effectively ; presence

(c)   dynamically ; cost

(d)   redundantly ; submission

(e)   dramatically ; outcomes

Answer: (e)

129. (I) Each year business leaders ………… to an overall revenue growth target, but the reality is that growth within a business is often very …….. .

(II) If you really want to grow a serious beard, you have to …….. to it or else you may end up getting an ……….. beard.

(a)   dedicate ; unseen

(b)   commit ; uneven

(c)   cooperate ; unevenly

(d)   predict ; unfriendly

(e)   decide ; reliable

Answer: (b)

130. (I) Research shows that even if reports about ……. turn out to be false, employee can suffer negative …….. .

(II) …….. always has ………….., but God forgives our sins.

(a)   performance, impact

(b)   misdeed, fallback

(c)   success, lessons

(d)   wrongdoing. consequences

(e)   report, impact

Answer: (d)

Directions (Q. Nos. 131-135) Read the following passage and answer the given questions.

Paragraph 1At a global financial services firm we worked with, a long time customer accidentally submitted the same application file to two offices. Though the employees who reviewed the file were supposed to follow the same guidelines-and thus arrive at similar outcomes-the separate offices returned very different quotes. Taken aback, the customer gave the business to a competitor. From the point of view of the firm, employees in the same role should have been interchangeable, but in this case they were not. Unfortunately, this is a common problem.

Paragraph 2 Professionals in many organizations are assigned arbitrarily to cases; appraisers in credit-rating agencies, physicians in emergency rooms, underwriters of loans and insurance, and others. Organizations expect consistency from the professionals: Identical cases should be treated similarly, if not identically. The problem is that humans are unreliable decision makers: their judgments are strongly influenced by irrelevant factors, such as their current mood, the time since their last meal, and the weather. We call the chance variability of judgments noise. It is an invisible tax on the bottom line of many companies.

Paragraph 3 Some jobs are noise-free. Clerks at a bank or a post office per complex tasks, but they must follow strict rules that limit subjective judgment and guarantee, by design, that identical cases will be treated identically. In contrast, medical professionals, loan officers, project managers, judges, and executives all make judgments calls, which are guided by informal experience and general principles rather than by rigid rules. And it they don’t reach precisely the same answer that every other person in their ole would, that’s acceptable; this is what we mean when we say that a decision is “a matter of judgment.” A firm hose employees exercise judgment does not expect decisions to be entirely free of noise. But often noise is far above the level that executive would consider tolerable- and they are completely unaware of it.

Paragraph 4 The prevalence of noise has been demonstrated in several studies. Academic researchers have repeatedly confirmed that professionals often contradict their own prior judgments when given the same data on different occasions. For instance, when software developers were asked on two separate days to estimate the completion time for a given task, the hours they projected differed by 71%, on average. When pathologists made two assessments of the severity of biopsy results, the correlation between their ratings was only 61 (out of a perfect 1.0), indicating that they made inconsistent diagnoses quite frequently. Judgments made by different people are even more likely to diverge. Research has confirmed that in many tasks, experts’ decisions are highly variable; valuing stocks, appraising real estate, sentencing criminals, evaluating job performance, auditing financial statements, and more. The unavoidable conclusion is that professionals often make decisions that deviate significantly from those of their peers, from their own prior decisions, and from rules that they themselves claim to follow.

Paragraph 5 Noise is often insidious: It causes even successful companies to lose substantial amounts of money without realizing it. How substantial? To get an estimate, we asked executives in one of the organizations we studied the following: “Suppose the optimal assessment of a case is $ 100000. What would be the cost to the organization if the professional in charge of the case assessed a value of $ 115000? What would be the cost of assessing it at $85000?” The cost estimates were high. Aggregated over the assessments made every year, the cost of noise was measured in billions- an unacceptable number even for a large global firm. The value of reducing noise even by a few percentage points would be in the tens of millions. Remarkably, the organization had completely ignored the question of consistency until then.

131. What is the opposite of the phrase “Unfortunately, this is a common problem” as mentioned in the Paragraph 1?

(a)   Employees often constitute variable decision-making capability even if they are assigned the same task to perform.

(b)   Even though employees are expected to come out with fair, positive and noise-free results, organizations find it almost an opposite and contradictory outcome to it.

(c)   A major problem is that the outcomes of decisions taken by different employees in the organization often aren’t known until far in the future, if at all.

(d)   Employees have to follow the strict norms and rules of the organization which often allow them to take rationale and conventional decisions in the best interest of the organization which hardly go unnoticed.

(e)   None of the above

Answer: (d)

132. What does the author mean by the term ‘noise’ as used in Paragraph 2?

(I) In an organization where work efficiency decides the potential of its employees, employees find it difficult to cope with their decisions and most of the time they land up in variable outcome to their motive which is termed as Noise.

(II) Noise is a problem which is effectively invisible in the business world; it can  be observed that audiences get quite surprised when the reliability of professional judgment is mentioned as an issue.

(III) Noise is a problem associated with the decision-making process of the employees involved in the corporate world which is invisible plausible alternatives to every judgment they make.

(a)   Only I is correct.

(b)   Only III is correct.

(c)   II and III are correct.

(d)   I and III are correct.

(e)   All are correct.

Answer: (d)

133. Why according to the author decision is considered as “a matter of judgment” as mentioned in Paragraph 3?

(I) In most of the cases, decisions are guided by informal experience and general principles rather than by rigid rules.

(II) In certain cases, medical professionals, judges and executives have to take decisions which are beyond the strict rules that control the instinctive judgment which might not be noise-free, yet acceptable to the system.

(III) Long experience on a job always increases people’s confidence in their judgments, but in the absence of rapid feedback, confidence is no guarantee of either accuracy or consensus.

(a)   Only I is correct.

(b)   Only III is correct.

(c)   I and III are correct.

(d)   I and II are correct.

(e)   All are correct.

Answer: (e)

134. Which of the following statements can follow paragraph 14 to form a connection with paragraph 5?

(a)   The surprising result of much research is that in many contexts reasoned rules are about as accurate as statistical models built with outcome data.

(b)   Uncomfortable as people may be with the idea, studies have shown that while humans can provide useful input to formulas, algorithms do better in the role of final decision maker.

(c)   Executives who are concerned with accuracy should also confront the prevalence of inconsistency in professional judgments.

(d)   Controlling noise is hard, but we expected that an organization that conducts an audit and evaluates the cost of noise in dollars will conclude that reducing random variability is worth the effort.

(e)   The major puzzle for us was the fact that neither organization had ever considered reliability to be an issue.

Answer: (c)

135. What is/are the author’s viewpoint (s) in accordance with Paragraph 5?

(I) The author is skeptical about the credibility of the research works on the measurement of cost of noise.

(II) The author is critical about the consequences of the problem of noise as it leads organizations to lose substantial amounts of money that often go unrealized.

(III) The author feels that the problems of noise is severe enough to require action.

(a)   Only I

(b)   I and III

(c)   II and III

(d)   All of these

(e)   Only II

Answer: (c)

Directions (Q. Nos. 136-140) In the following question part of the sentence is given in bold, it is then followed by three sentences which try to explain the meaning of the phrase given in bold. Choose the best set of alternatives from the five options given below each question which explains the meaning of the phrase correctly without altering the meaning of the sentence given as questions.

136. The problem of secularism in independent India lies in the fact that the Constitution was essentially a Western construct with the sidelining of the Gandhian Influence in the Constituent Assembly.

(I) Secularism in India is highly motivated by the Western pattern of Constitution with hardly any presence of Gandhian Influence.

(II) The Constitution was basically influenced by the culture followed in Western societies and it hardly considered the Gandhian Influence in the Constituent Assembly which depicts the problem of secularism in independent India.

(III) The developmental projects of western countries  had such an impact on Indian Constitution that it ignored the Gandhian Influence and today we are facing the problem of secularism in the country.

(a)   Only I is correct.

(b)   Only II is correct.

(c)   I and II are correct.

(d)   II and III are correct

(e)   All are correct.

Answer: (b)

137. The dichotomy is the striking feature. An estimated 25% of our population is economically advanced with access to the latest technology, while the rest wallows in inhuman conditions.

(I) An estimated  25% of our population has access to the latest technology forming economically advanced populace while the rest still lives a savage life; it make the difference more evident.

(II) What is more striking is that one-fourth of our population is economically and technologically advanced while the rest is deprived of any such facilities.

(III) It is a distinct feature that around 25% of our population is economically advanced with proficiency in technology and communication while the rest is still reeling under poverty.

(a)   Only I is correct.

(b)   Only III is correct.

(c)   II and III are correct.

(d)   I and II are correct.

(e)   All are correct.

Answer: (c)

138. Without casting aspersions on anyone, it must be noted that prolonged teething troubles of the Coast Guard resulted from the Navy considering the service a ‘poor relative’.

(I) There is a consistent extension of technical problems that the Coast Guard is facing as the Navy considers the service its poor correspondents.

(II) With no intention of criticism, it is to be noted that the sustained short-term problems of the Coast Guard resulted because Navy considered the service a poor correspondent.

(III) Without tossing any aspersions, the perpetual existence of long-term worries is a matter of concern for the Coast Guard as its is resulted from the navy considering the service its poor fellow.

(a)   Only I is correct.

(b)   Only II is correct.

(c)   I and III are correct.

(d)   II and III are correct.

(e)   All are correct.

Answer: (b)

139. The teacher tried his best explain the importance of the chapter to the students but soon realized that he was casting pearls before swine.

(I) The teacher soon realized that whatever he was explaining to the students about the chapter was grasped by them the way he expected.

(II) Despite the hard work done by the teacher to explain the importance of the chapter, the students were busy making mockery of him.

(III) After trying his best to make the students understand the importance of the chapter, the teacher soon realized that they are not recognizing its worth.

(a)   Only I is correct.

(b)   Only II is correct.

(c)   Only III is correct.

(d)   II and III are correct.

(e)   None is correct.

Answer: (c)

140. The Supreme Court has struck a blow for the rights of the disabled, with a direction to the Central State governments to provide full access to public facilities, such as buildings and transport, within stipulated deadlines.

(I) The Supreme Court has ordered all the States to provide full access to public facilities like building and transport to the disabled and they must not ignore the true spirit and purpose of the law made for these people.

(II) The Supreme Court has ordered the Central government to provide the basic amenities to the disabled within the stipulated time period.

(III) The Supreme Court has warned the Central and State governments for the rights of the disabled which they are deprived of and has ordered them to provide full access to public facilities, such as buildings and transport, within stipulated deadlines.

(a)   Only I is correct

(b)   Only II is correct.

(c)   Only III is correct.

(d)   All are correct.

(e)   None is correct.

Answer: (c)

Directions (Q. Nos. 141-145) In each of the given questions an inference is given in bold which is then followed by three paragraphs. You have to find the paragraphs(s) from where it is inferred. Choose the option with the best possible outcome as your choice.

141. The major concern is security.

(I) The major dilemma for may security professionals is whether the Brexit will make the UK more or less safe when it comes to cyber security. One poll found that most security professionals believed there would not be any major cyber security implications; however, another poll offered different conclusions, with most respondents believing that a Brexit would weaken cyber security because of additional bureaucratic hurdles to information sharing with EU.

(II) Most debate over Bexit has been about economics, trade and migration. But when David Cameron called the EU referendum in February the cited a new factor, asserting that membership made Britain safer. This week the Prime Minister went further, hinting that Brexit might increase the risk related to security- and adding that, every time Britain turned its back on Europe, it had come to regret it.

(III) Prime Minister Shinzo Abe touted the implementation of the legislation as an event of “historic importance that makes peace and security of our country even more secure” and “upgrades our deterrence ad enables the nation to proactively contribute more than ever to peace and stability of regional and international communities.”

(a)   Only I

(b)   II and III

(c)   Only III

(d)   I and III

(e)   All are correct

Answer: (d)

142. The government is trying hard to lure FD in India.

(I) In yet another significant move to attract Foreign Direct Investment (FDI), the government has opened the door wider in several major sectors of the Indian economy, through what is calls “path-breaking” amendments in the extant FDI policy.

(II) In less than a year, the government of India has announced yet another set of ‘radical changes’ in Foreign Direct Investment (FDI) policies. The earlier announcement in November, 2015 introduced changes in 15 major sectors, and the latest announcement covers nine sectors which seek to further simplify the regulations governing FDI in the country and make India an attractive destination for foreign investors.

(III) For India, the servicing burden of FDI in terms of repatriations, dividend payments and payments for use of intellectual property is now showing up prominently. About half of the inflows into India during the past six years were balanced by outflows.

(a)   Only I is correct.

(b)   Only II is correct.

(c)   Only III is correct.

(d)   I and II are correct.

(e)   All are correct.

Answer: (d)

143. Country’s economic standard can be best adjudged by per capital income.

(I) Exports and imports, a swelling favourable balance of trade, investments and bank-balances, are not an index or a balance sheet of national prosperity. Till the beginning of the Second World War, English exports were noticeably greater than what they are today. And yet England has greater national prosperity today than it ever had. Because the income of average Englishmen, working as field and factory labourers, clerks, policemen, petty shopkeepers and shop assistants, domestics workers and other low-paid workers, has gone up.

(II) It is possible that while per capita real income is increasing per capita consumption of goods and services might be falling. This happens when the government might itself be using up the increased income for massive military build up necessitating heavy production of arms and ammunitions.

(III) A rise in national income may occur as a result of increased spending on items such as defence. National income often rises in time of war, or the threat of war, because money is spent on weapons. This will push up GNP, but the people may be acutely short of goods to buy.

(a)   II and III

(b)   I and II

(c)   Only I

(d)   Only II

(e)   All are correct

Answer: (c)

144. The calorie count of foods that are high in carbohydrates is significant.

(I) Moderate use of non-nutritive sweeteners like aspartame for low calorie count could have  positive effect on insulin and blood sugar by aiding weight control. It is also important to keep in mind that many foods containing aspartame still provide calories and  carbohydrate from other ingredients, even though they may be labelled ‘sugarfree’.

(II) Ironically, people who use aspartame as a sweetener to reduce their calorie intake could wind up defeating their purpose, since studies show that high levels of aspartame may trigger a craving for carbohydrates by depleting the brain of a chemical that registers carbohydrates satiety.

(III) Forty-five to 65% of your total calories should come from carbs, recommends the Institute of Medicine. If you consume carbs on a regular basis, glycogen stores stay full and become a normal part of your total body weight. The rise in the popularity of Aspartame is due to its comparatively low calorie count.

(a)   Only II

(b)   I and II

(c)   II and III

(d)   All are correct

(e)   Only I

Answer: (e)

145. Pollution is adversely affecting the health of children.

(I) A study conducted on Delhi children and released recently in the Journal of Indian Paediatrics provides powerful evidence that shows children growing up in polluted environments like the Capital have reduced lung growth compared to children in developed countries like the United States.

(II) Bad food habits is taking a toll on our children, warn doctors. A group of doctors have published a multi-centric study to drive home the ill effects of moving away from healthy eating habits and opting for easy-to-use and widely accessible processed food.

(III) Children living in big cities such as Delhi, are likely to grow susceptible to allergic ailments, more than adults, due to urban pollution, especially air, health experts said. Infants and children living in metro cities are inhaling polluted air and therefore their resistance to allergic ailments is lowered at very young age, making them more susceptible to contract various allergies when they grow up, compared to adults.

(a)   Only I is correct.

(b)   Only II is correct.

(c)   Only III is correct.

(d)   Both I and III are correct.

(e)   All are correct.

Answer: (d)

Directions (Q. Nos. 146-150) Read the following passage divided into number of  paragraphs carefully and answer the questions.

Paragraph 1 Deutsche is more leveraged than its peers; it is unusual in lacking a crown jewel around which it can base a business model; and it has a stack of derivatives whose prices are hard to observe in the market. More positively, it is light on the non-performing loans that clog the balance-sheets of banks in places like Italy. But in other ways its problems have a very familiar ring. Deutsche is struggling to make a decent return. It has taken too long to face up to its problems. And the market it operates in is over banked. Years after American banks were forced to clean themselves up, too m any European lenders are still flailing as a result.

Paragraph 2 Europeans prefer to blame others for the turmoil. Deutsche has lashed out at “forces in the market” for its most recent bout of trouble. But its shares had already fallen by 42% this year before news broke last month of a proposed Department of Justice (DoJ) fine of $ 14 billion for mortgage-related misdeeds. German politicians insinuate that the mooted fine represents revenge for Europe’s recent tax case against Apple, an American champion. Yet the DoJ has slapped large fines on American banks, too. Deutsche’s vulnerability to shocks is the problem, not the shocks themselves.

Paragraph 3 Fingers also point at global regulators. The boos of Credit Suisse, Tidjane Thiam, says his sector is ‘not really investible’. It is true that the rules have got much stricter in the past few years, particularly for institutions, like Deutsche, that have big investment-banking arms. It is also true that ultra-loose monetary policy, and in particular the negative interest rates that now prevail in much of Europe, eat away at banks’ profitability. But some banks cope better than others in this painful environment. The IMF has compared returns on equity before and after the financial crisis. Those at large European  banks fell by 11.4% points, whereas those at American lenders dipped by only three points. Rather than blaming speculators, Americans and regulators, Europe’s bakers and policymakers need to put their own house in order.

Paragraph 4 Within institutions, that means cutting costs and raising capital. According to S $ P Global Market Intelligence, the average cost-to-income ratio at an American bank in 2015 was 59%; Italy’s figure stood at 67% and Germany’s at 72%. Scandinavian banks already operate with much lower costs than their peers elsewhere in Europe. The axe is now swinging: Commerzbank, another struggling German lender, and ING, a Dutch bank, have announced thousands of job cuts in the past few days.

Paragraph 5 But more can be done. Pay is one obvious lever. Deutsche’s bankers trousered roughly the same amount in annual compensation between 2011 and 2015, even as the bank’s share price dived. And before shareholders complain too loudly about that, recall that in 2007-15 the divided payments by 90 euro-zone banks amounted to € 223 billion ($250 billion). Their retained earnings would have been 64% higher at the end of that period if they had not paid out dividends.

146. Which of the following sentences justifies the statement, “Deutsche is more leveraged than its peers” as mentioned in paragraph 1?

(I) Deutsche is scuffling to generate adequate profits to stay afloat in the market.

(II) Though the market in which Deutsche operates is overbanked; it has a mound of derivatives whose prices are hard to observe in the market.

(III) Though Deutsche has taken a longer time to face up its problems, it is still airy on the non-performing loans that obstruct the balance-sheets of banks in places like Italy.

(a)   Only I is correct.

(b)   Only II is correct.

(c)   I and II are correct.

(d)   II and III are correct.

(e)   All are correct.

Answer: (d)

147. What does the author mean by the statement, “Deutsche’s vulnerability to shocks is the problem, not the shocks themselves” as mentioned in Paragraph 2?

(a)   Deutsche is m ore prone to consistent failures which may have indirect effects on the market in coming years.

(b)   Deutsche is losing its share values inadvertently due to certain forces in the market which is surprising to German politicians.

(c)   Despite performing better in certain specific fields than other banks, Deutsche is finding itself in the midst of turbulence which is itself a revelation.

(d)   The arguable fine of $14 billion for mortgage-related misdeeds by Department of Justice has completely shocked the entire European bureaucrats.

(e)   None of the above

Answer: (c)

148. Which of the following statements cannot be inferred from Paragraph 3?

(a)   Despite having bigger-investment banking arms, the consistent downfall of European banks is a matter of serious concern.

(b)   The prevalling negative interest rates in many parts of Europe indicate that there is ultra-loose monetary policy in Europe.

(c)   The IMF figures on return on equity show that American banks are performing for better than European banks in the existing critical market.

(d)   Without pruning, returns on equity of European banks are projected to fall towards zero as a result of ultra-low rates and regulation as compared to American counterpart.

(e)   European policymakers need to come out with strict and robust policies to safeguard its downgrading existence in the world market than pointing figures on speculators and regulators.

Answer: (e)

149. Which of the following sentences can be connected with Paragraph 4 to make a connection with Paragraph 5?

(a)   Some European markets have been clearing away excess capacity.

(b)   Like Deutsche, Credit Suisse is freer to make plans after a recent settlement with American authorities over mis-selling mortgage-backed securities before the financial crisis.

(c)   It expects to wind up a unit in which it has dumped unwanted assets by the end of 2018, a year ahead of schedule.

(d)   European banks could have done a lot more sooner.

(e)   The recapitalization of Europe’s banks has  been as gradual as that of America’s was swift, and in dribs and drabs of tens of billions a year rather than in one big splurge.

Answer: (a)

150. What could be the possible solutions for the rehabilitation of European banks in the existing system?

(I) Proper fiscal stimulus by European governments would cut the chances that central banks have to keep interest rates so low.

(II) Using public money to recapitalize the weakest banks in countries like Italy, and requiring them to slim down in return, is the fastest way to return them to health.

(III) Significant job cuts of their employees to compensate the losses occurred in last few years could bring everything back to normalcy.

(a)   Only I

(b)   Only II

(c)   I and III

(d)   I and II

(e)   All of these

Answer: (d)

Directions (Q. Nos. 151-152) In each question, the word at the top is used in four different ways, numbered (a) to (d). Choose the option in which the usage of the word in incorrect or inappropriate. If all the options are grammatically correct and usage of the given word is appropriate, then select option (e).

151. Turn

(a)   Your mother wants you to turn the TV down and come for dinner.

(b)   Just turn left at the end of the street and you will reach my house.

(c)   Can you turn the music up? This is my favourite song.

(d)   Our cat turned up after we put posters up all over the neighbourhood.

(e)   None of the above

Answer: (a)

152. Jack

(a)   We jacked the car up and changed the tyre.

(b)   Don’t listen to him –he always jacks people in.

(c)   I jacked my job in because my boss refused to give me a raise.

(d)   Once the tourists arrive, the restaurants jack up their prices.

(e)   None of the above

Answer: (b)

Directions (Q. Nos. 153-155) There are sets of four statements in question given below which when connected using the correct sentence structure forms a complete single sentence without altering the meaning of the sentences given in the question. There are four options given below the question, choose the sentence that forms the correct formation of single sentence which is both grammatically correct and contextually meaningful. If none follows, choose (e) as your answer.

153. Time will be required to implement badly needed judicial reforms; there are a few procedural steps that India can implement meanwhile; it should have the potential of transforming India’s justice system, work as force multipliers in clearing the backlog and improve India’s record in contract enforcement; it should give a boost to GDP and ensure that the rule of law prevails.

(a)   There are a few procedural steps that India can implement in the short-term to implement badly need judicial reforms by having the potential of transforming India’s justice system, work as force multipliers in cleaning the backlog and improve India’s record in contract enforcement and should give a boost to GDP to ensure that the rule of law prevails.

(b)   During the time required to implement badly needed judicial reforms, there are a few procedural steps that India can implement in the short-term which have the potential of transforming India’s justice system, work as force multipliers in clearing the backlog, improve India’s record in contract enforcement, give a boost to GDP and ensure that the rule of law prevails.

(c)   To implement badly need judicial reforms,, the time has come to take a few procedural steps that can be implemented by India in the short-term that should have the potential of transforming India’s justice system, working as force multipliers in clearing the backlog and improving India’s record in contract enforcement that can give a boost to GDP which ensure that the rule of law prevails.

(d)   With the motive of giving a boost to GDP and ensuring that the rule of law prevails, the time has come to implement badly needed judicial reforms by implementing a few procedural steps in the short run that should have the potential of transforming India’s justice system by working as force multipliers in cleaning the backlog and improve India’s record in contract enforcement.

(e)   None of the above is correct.

Answer: (b)

154. There is a statedly contemporary twist of a 360° review by peers, seniors and colleagues; it is an equally hush-hush affair; it does not really pass muster as an objective assessment tool for professional capabilities, it is in spite of our unique work environment.

(a)   The statedly contemporary twist of a 360° review by peers, seniors and colleagues, is an equally hush-hush affair and does not really pass muster as an objective assessment tool for professional capabilities, given our unique work environment.

(b)   Despite our unique work environment, there is a statedly contemporary twisting of a 360° review by peers, seniors, and colleagues which is an equally hush-hush affair but it has not really passed muster as an objective assessment tool for professional capabilities.

(c)   Twisting review to 360° by peers, seniors and colleagues and making it an equally hush-hush affair, it does not really pass muster as an objective assessment tool for professional capabilities in our unique work environment.

(d)   Although we have unique work environment,. there is a statedly contemporary twist of a 360° review by peers, seniors and colleagues, and an equally hush-hush affairs which do not pass muster as an objective assessment tool for professional capabilities.

(e)   None of the above is correct.

Answer: (a)

155. The major thrust of Marx’s political philosophy was aimed at human liberation; it is important to consider the significant shift to comprehend it; the shift occurred in the late 17th century; In that period traditional analysis of the political order based on scarcity was replaced by a philosophy of abundance.

(a)   Comprehending the major thrust of Marx’s political philosophy that was aimed at human liberation. It is important to consider the significant shift of late 17th century when the scarcity based on traditional analysis was replaced by a philosophy of abundance.

(b)   Marx’s major thrust was on political philosophy aiming human liberation while it is important consider the significant shift that occurred in the later 17th century, it was then that traditional analysis of the political order  based on scarcity was replaced by a philosophy of abundance.

(c)   It is important to consider the significant shift in the late 17th century when traditional analysis of the political order based on scarcity replaced philosophy of abundance to comprehend the major thrust of Marx’s political philosophy of human liberation.

(d)   To comprehend the major thrust of Marx’s political philosophy that aimed at human liberation, it is important to consider the significant shift that occurred in the late 17th century when traditional analysis of the political order based on scarcity was replaced by a philosophy of abundance.

(e)   None of the above is correct.

Answer: (d)

SBI PO (Pre.) Examination Held on 09-07-2016 Quantitative Aptitude Question Paper With Answwer Key

SBI PO (Pre.) Examination Held on 09-07-2016 Quantitative Aptitude
SBI PO (Pre.) Examination Held on 09-07-2016 Quantitative Aptitude Question Paper With Answwer Key

SBI PO (Pre.) Examination Held on 09-07-2016

Quantitative Aptitude

1. C is 20% more efficient than A. A and B together can finish a piece of work in 16 days. B and C together can do it in 15 days. In how many days A alone can finish the same piece of work?

(A)  42

(B)  48

(C)  54

(D)  36

(E)  45

Answer: (B)

2. A started a business with an investment of Rs 16,000. After 6 months from the start of the business, B and C joined with Rs 12,000 and Rs 18,000 respectively and A invested an additional amount of Rs 4000. If the difference between A’s share and B’s share in the annual profit is Rs 6000, what was the annual profit received?

(A)  Rs 17,600

(B)  Rs 13,200

(C)  Rs 14,300

(D)  Rs 16,500

(E)  Rs 11,000

Answer: (D)

3. Shiva gives 20% of her monthly salary to his mother, 50% of the remaining salary he invests in an insurance scheme and PPF in the respective ratio of 5 : 3 and the remaining he keeps in his bank account. If the sum of the amount he gives to his mother and that he invests in PPF is Rs 12,600, how much is Shiva’s monthly salary?

(A)  Rs 36,000

(B)  Rs 64,000

(C)  Rs 42,000

(D)  Rs 40,000

(E)  Rs 50,400

Answer: (A)

4. The respective ratio of radii of two right circular cylinders (A and B) is 4 : 7. The respective ratio of the heights of cylinders A and B is 2 : 1. What is the respective ratio of volumes of cylinders A and B?

(A)  25: 42

(B)  23 : 42

(C)  32 : 49

(D)  30 : 49

(E)  36 : 49

Answer: (C)

5. At present, Aanshi is five years younger to Binny. Binny’s age twenty-years hence will be equal to twice of Aanshi’s age five years ago. What will be Binny’s age eight year  hence?

(A)  42years

(B)  35 years

(C)  30 years

(D)  40 years

(E)  48 years

Answer: (E)

Directions – (Q. 6-10) In this equation two equations numbered I and II are given . You have to solve both the equations and mark the appropriate option.

6. (I) 9x2 – 37x + 30 = 0

(II) 3y2 – 19y + 30 =0

(A)  Relationship between X and Y cannot be established

(B)  X < Y

(C)  X > Y

(D)  X ≤ Y

(E)  X ≥ Y

Answer: (D)

7. (I) 2x2 – 17x + 36 = 0

(II) 5y2 – 33y + 40 = 0

(A)  Relationship between X and Y cannot be established

(B)  X < Y

(C)  X > Y

(D)  X ≤ Y

(E)  X ≥ Y

Answer: (A)

8. (I) 12x2 – 23x + 11 = 0

(II) 21y2 – 20y + 4 = 0

(A)  Relationship between X and Y cannot be established

(B)  X < Y

(C)  X > Y

(D)  X ≤ Y

(E)  X ≥ Y

Answer: (C)

9. (I) x2 + 12x + 35 = 0

(II) 7y2 + 32y + 16 = 0

(A)  Relationship between X and Y cannot be established

(B)  X < Y

(C)  X > Y

(D)  X ≤ Y

(E)  X ≥ Y

Answer: (B)

10. (I) 25x2+ 20x + 3 = 0

(II) 4y2 + 11y + 6 = 0

(A)  Relationship between X and Y cannot be established

(B)  X < Y

(C)  X > Y

(D)  X ≤ Y

(E)  X ≥ Y

Answer: (C)

11. A number is such that when it is multiplied by 6, it gives another number with is more than 168 as the original number itself is less than 168. What is 15% of the original number?

(A)  8.4

(B)  7.8

(C)  6.6

(D)  8.8

(E)  7.2

Answer: (E)

12. Dharma invested Rs P for 3 years in scheme A which offered 12% p.a. simple interest. She also invested Rs P + 400 in scheme B which offered 10% compound interest (compounded annually), for 2 years. If the amount received from scheme A was less than that received from scheme B, by Rs 304, what is the value of P?

(A)  Rs 1400

(B)  Rs 1000

(C)  Rs 1500

(D)  Rs 900

(E)  Rs 1200

Answer: (E)

Directions – (Q. 13-17) Study the following table carefully and answer the question.

13. The number of students (both male and female) who are learning freestyle in schools C and D together is what per cent less than the number of students (both male and female) who are learning the same in schools A and B together?

(A) 

(B) 

(C) 

(D) 

(E) 

Answer: (D)

14. What is the average number of students learning dance forms (other than freestyle) in schools B, C and D?

(A)  206

(B)  207

(C)  204

(D)  205

(E)  201

Answer: (B)

15. Out of the number of students (both male and female) learning freestyle in school B, 5/8 are 15 years or above. If out of the total students who are 15 years or above, 30% are females, what is the number of female students learning freestyle who are below 15 years?

(A)  35

(B)  42

(C)  32

(D)  46

(E)  40

Answer: (B)

16. What is the respective ratio between the total number of male students learning freestyle in schools A and C together and the total number of female students learning the same in the same schools together?

(A)  6 : 39

(B)  6 : 13

(C)  12 : 13

(D)  6 : 11

(E)  12 : 35

Answer: (B)

17. What is the difference between the number of male students studying freestyle in schools B and D together and the total number of female students studying the same in the same schools together?

(A)  22

(B)  18

(C)  26

(D)  16

(E)  12

Answer: (A)

18. Per cent profit earned when an article is sold for Rs 558 is double the per cent profit earned when the same article is sold for Rs 504. If the marked price of the article is 30% above the cost price, what is the marked price of the article?

(A)  Rs 585

(B)  Rs 595

(C)  Rs 624

(D)  Rs 590

(E)  Rs 546

Answer: (A)

Directions – What will come in place of question-mark (?) in the given number series?

19. 150  102  70  46  26  ?

(A)  7

(B)  13

(C)  8

(D)  2

(E)  12

Answer: (C)

20. 10  14  24  52  134  ?

(A)  351

(B)  302

(C)  368

(D)  341

(E)  378

Answer: (E)

21. 24  11  10  14  27  ?

(A)  67.5

(B)  60.5

(C)  66.5

(D)  61.5

(E)  62.25

Answer: (C)

22. 4500  900  90  6  ?  012

(A)  0.3

(B)  0.09

(C)  0.9

(D)  0.015

(E)  0.03

Answer: (A)

23. 8  7  12  33  128  ?

(A)  672

(B)  684

(C)  635

(D)  620

(E)  692

Answer: (C)

24. 3? × √170 = 183.998 ÷ 8.001 + 328.02

(A)  2

(B)  4

(C)  1

(D)  5

(E)  3

Answer: (E)

25. ?% of (230.02 ×89 – 559.85) = 960

(A)  20

(B)  80

(C)  50

(D)  70

(E)  75

Answer: (E)

26. 002 – 44.999 + 8.998 – 8.012 = ?

(A)  650

(B)  350

(C)  450

(D)  550

(E)  600

Answer: (C)

27. 98 ÷ 15.02 + 450.15 ÷ 8.9992 + 56.02 = ?

(A)  230

(B)  200

(C)  180

(D)  150

(E)  130

Answer: (E)

28. 25% of 459 + 65.01 ÷ 02 = ?

(A)  109

(B)  128

(C)  234

(D)  80

(E)  186

Answer: (B)

Directions- Refer to the bar graph and answer the given question.

29. In 2005, out of total number of application received for subjects A and B together, only 30% were accepted. What was the total number of applications accepted for subjects A and B together in 2005 ?

(A)  121

(B)  123

(C)  129

(D)  131

(E)  133

Answer: (B)

30. In 2004, 30% of applications received for subject A and 20% of applications received for subject B were from international students. What was the total number of International applicants for subjects A and B together in 2004 ?

(A)  91

(B)  97

(C)  89

(D)  93

(E)  87

Answer: (A)

31. If the respective ratio of total number of applications received for subjects A and B together in 2008 and 2009 is 3 : 4, what was the total number of applications received for subjects A and B together in 2009 ?

(A)  500

(B)  560

(C)  400

(D)  520

(E)  500

Answer: (B)

32. What is the average number of applications received for subject A in 2005, 2007 and 2008 ?

(A)  190

(B)  180

(C)  170

(D)  200

(E)  160

Answer: (A)

33. Number of applications received for subject B increase by what per cent from 2004 to 2006 ?

(A) 

(B) 

(C) 

(D) 

(E) 

Answer: (C)

34. In a bag, there are 8 red balls and 7 green balls. Three balls are picked at random. What is the probability that two balls are red and one ball is green in colour?

(A)  28/65

(B)  22/65

(C)  37/65

(D)  3/13

(E)  1/13

Answer: (A)

35. A vessel contains 120 litres of mixture of milk and water in the respective ratio 11 : 4. Forty-five l itres of this mixture was taken out and replaced with 5 litres of water. What is the percentage of water in the resultant mixture?

(A)  35

(B)  30

(C)  25

(D)  20

(E)  15

Answer: (*)

State Bank of India S.O. Examination Held on 17-6-2017 Reasoning and Data Interpretation Question Paper With Answer Key

State Bank of India S.O. Examination Held on 17-6-2017 Reasoning and Data Interpretation
State Bank of India S.O. Examination Held on 17-6-2017 Reasoning and Data Interpretation Question Paper With Answer Key

State Bank of India S.O. Examination Held on 17-6-2017

Reasoning and Data Interpretation

1. Read the given information and answer the given question-

College D has been experiencing frequent power cuts lately.

“We are in this situation only because of College S. Due to the new rule laid out by the government, the units consumed by an area calculated while deciding power cuts and those areas using more power face frequent cuts. College S regularly lend its ground to outsiders thus consuming much more power than us” – Statement by one of the staff members of college D.

Which of the following can be inferred from the statement of the staff member of college D?

(A)  The number of students studying in both colleges S or D is more or less equal

(B)  Both colleges S and D are located in the same area

(C)  College S is not facing as many power cuts as College D

(D)  College D consumes least units of electricity in its area

(E)  College D haasa not lent its ground even once to outsiders

Answer: (B)

Directions – (Q. 2-4) Study the following information carefully and answer the given question –

     There are ten family members A, B, C, D, E, F, G, H, I, J. There are three generations in the family. There are three married couple in the family. B is the only sister of G’s brother H. G. and J are the married couple. A is the mother of J. D is the brother of J. C is the father-in-law of G. I is the mother of B. F is the father-in-law of J. E is the son of G.

2. How is G related to D?

(A)  Brother

(B)  Brother-in-law

(C)  Cousin

(D)  Father

(E)  Husband

Answer: (B)

3. How is I related to E?

(A)  Grandfather

(B)  Sister

(C)  Grandmother

(D)  Mother

(E)  Brother

Answer: (C)

4. How is B related to E?

(A)  Aunt

(B)  Uncle

(C)  Spouse

(D)  Cousin

(E)  Niece

Answer: (A)

5. Read the given information to answer the given question-

A private sector bank of company P has been launching many new schemes on its credit cards from the past one year for its customers.

Which of the following may be an effect on its customers of the launch of new schemes by the bank ?

(1) The number of customers opting for transaction through credit card will increase.

(2) The said bank will make meager profits with t he launch of these new schemes.

(3) The internet rate offered by the bank on all its cards will decrease.

(4) Many new customers will be attracted to open an account with the said bank.

(A)  Only 1

(B)  None will be an effect

(C)  Only 1, 2 and 4

(D)  Only 2 and 3

(E)  All 1, 2, 3, and 4

Answer: (C)

6. Statement : P > R ≥ A = N ≤ K ≤ S

Conclusion : I. N < P

II. S ≥ A

(A)  Only conclusion I follows

(B)  Only conclusion II follows

(C)  Both conclusions I and II follow

(D)  Neither conclusion I nor II follows

(E)  Either conclusion I or II follows

Answer: (C)

Directions- (Q. 7-10) Study the following information to answer the given question –

      Six person, A, B, C, D, E and F are seated in a straight line facing North, with equal distance between each other but not necessarily in the same order. Each of them also likes a different colour viz. Red, blue, Pink, Yellow, White and Silver but not necessarily in the same order.

   C sits at an extreme end of the line. Only two people sit  between C and the one who likes White. A sits second to the left of the one who likes Blue. A is not an immediate neighbour of the one who likes White. As many people sit to the left of a as to the right of one who likes Silver. E is an immediate neighbour of the one who likes silver. F sits third to the right of the one who likes Pink. B sits at one of the positions to the right of F. F does not like Yellow.

7. What is the position of D with respect to B?

(A)  Second to the right

(B)  Third to the left

(C)  Fourth to the right

(D)  Second to the left

(E)  Third to the right

Answer: (B)

8. Who amongst the following likes Red colour?

(A)  A

(B)  C

(C)  D

(D)  F

(E)  E

Answer: (D)

9. Which of the following represents the immediate neighbour of the one who sits at the extreme ends of the line?

(A)  E, F

(B)  E, D

(C)  A, B

(D)  B, F

(E)  A, F

Answer: (A)

10. Which of the following is true with respect to the given arrangement?

(A)  The one who likes Pink sits to the immediate left of C

(B)  Only one persons sits between D and the one who likes Red

(C)  None of the given option is true

(D)  A likes Pink

(E)  C likes Silver

Answer: (B)

11. If it is possible to make only one meaningful English word with the first, fourth, seventh and ninth letters (when counted from left to right) of the word PRESIDENT using each letter only once, which will be the third letter of the word so formed from the left end. If no such word can be formed give X as your answer. If more than one such word can be formed, give Z as your answer.

(A)  Z

(B)  X

(C)  T

(D)  O

(E)  S

Answer: (A)

12. Read the following information and answer the question-

Municipal Corporation of City X has received a complaint about extending the premises of residential colony Dream Home without obtaining necessary permissions from the authorities. Select the most appropriate sequence of courses of action on behalf of Municipal Corporation of City X ?

(1) The builder of Dream Home should be given a month’s notice the rebuild a boundary within the permissible area.

(2) Follow up whether the premises are rebuild as per the norms of municipal corporation.

(3) Municipal Corporation of City X should order an enquiry to determine the authenticity of the complaint.

(4) Levy a plenty on the builder of DreamHome for extending premises without obtaining necessary permissions from the authorities.

(A)  3241

(B)  1243

(C)  2413

(D)  1324

(E)  3412

Answer: (A)

13. What should come next in place of question mark (?) in the given letter series according to the English alphabetical series?

AZ CX FV JT OR ?

(A)  UQ

(B)  SQ

(C)  UP

(D)  TP

(E)  TJ

Answer: (C)

14. How many such pairs of letters are there in the word FUMING, each of which has a many letters between them in the word (in both forward and backward directions) as they have in the English alphabetical series?

(A)  Four

(B)  One

(C)  Two

(D)  Three

(E)  More than four

Answer: (C)

15. The position of how many digits will remain same if digits in the number 613852747 are arranged in the ascending order within the numbers from left to right?

(A)  None

(B)  Three

(C)  Two

(D)  One

(E)  More than three

Answer: (B)

Directions – (Q.16-20) When a word and number arrangement machine is given an input line of words and numbers, it arranges them following a particular rule. The following is an illustration of input and rearrangement.

   (All the numbers are two-digit numbers)

Input : Apple 4216 Egg 34 Fan Grapes 25 91 Horse

Step 1 : Horse Apple 42 16 Egg 34 Fan Grapes 25 19

Step 2 : Horse Grapes Apple 16 Egg 34 Fan 25 19 24

Step 3 : Horse Grapes Fan Apple 16 Egg 25 19 24 43

Step 4 : Horse Grapes Fan Apple 16 Egg 19 24 43 52

Step 5 : Horse Grapes Fan Apple Egg 19 24 43 52 61

Step V  is the last step of above arrangement as the indended arrangement is obtained

As per the rules followed in the given step, find out the appropriate steps for the input.

Input : Ant 43 17 Eagle 24 Forest Gun 25 81 House.

16. Which element comes exactly between ‘17’ and ‘25’ in Step III of the given Input?

(A)  House

(B)  Gun

(C)  Eagle

(D)  18

(E)  Ant

Answer: (C)

17. If in the second step, ‘17’ interchanges its position with ‘25’ and ‘Gun’ also interchanges its position with ‘Forest’ then which element will be the second from the left end ?

(A)  House

(B)  Gun

(C)  18

(D)  Eagle

(E)  Forest

Answer: (E)

18. Which of the following combinations represent the first two and the last two elements in Step IV of the given input?

(A)  House Gun and 52 71

(B)  House Gun and 34 42

(C)  House Gun and 34 52

(D)  House Gun and 42 52

(E)  House Gun and 42 71

Answer: (D)

19. Which element in third t the left of the one which is eighth from the left in Step III of the given Input?

(A)  17

(B)  Forest

(C)  Gun

(D)  42

(E)  Eagle

Answer: (A)

20. In which step are the elements ‘Ant 17 Eagle 25’ found in the same order?

(A)  Third

(B)  Fourth

(C)  Second

(D)  Fifth

(E)  The given order of elements is not found in any step

Answer: (A)

Directions – (Q. 21-25) Study the given information carefully to answer the given question-

   Eight boxes –M, N, O, P, Q, R, S and T are kept above one another in a stack. Only two boxes are kept between M and T. P is kept immediately above T. As many boxes are kept between M and S as between S and P. Only three boxes are kept between S and O. S is kept at one of the positions above O. Only one box is kept between O and R. N is kept at one of the positions below O.

21. What is the position of Box Q in the stack?

(A)  Second from the top

(B)  Fifth from the bottom

(C)  Fifth from the top

(D)  Third from the top

(E)  Third from the bottom

Answer: (C)

22. What is the position of Box S in the stack?

(A)  Second from the top

(B)  Fifth from the bottom

(C)  Fifth from the top

(D)  Third from the top

(E)  Third from the bottom

Answer: (A)

23. Which of the following is at the bottom of the stack?

(A)  N

(B)  R

(C)  M

(D)  P

(E)  Q

Answer: (B)

24. Which of the following is true with respect to the given information ?

(A)  No one sits between O and Q

(B)  No one lives above M

(C)  T sits fourth from the top

(D)  N sits second from the bottom

(E)  All the given options are true

Answer: (E)

25. Which of the following is kept between Box Q and Box N?

(A)  R

(B)  S

(C)  M

(D)  P

(E)  O

Answer: (E)

Directions – (Q. 26 and 27) Study the given information carefully and answer the given question-

    Two different sum of money, Rs ‘X’ and Rs. ‘0.8X’ are invested in two schemes A and B respectively, for 2 years and 6 years respectively. Scheme A offers compound interest @ 10% p.a., compound annually while Scheme B offers simple interest @ 7.5% p.a. The interest earned from Scheme B is Rs 1050 more than that from Scheme A.

26. Interest earned from Scheme A was what per cent less than that earned from scheme B?

(A) 

(B) 

(C) 

(D) 

(E)   

Answer: (C)

27. What would have been the difference between the interest from both schemes, if Rs ‘1.2X’ had been invested in each of the Schemes for the given periods respectively ? (In Rs)

(A)  1996

(B)  1984

(C)  2004

(D)  2022

(E)  Other than those given as options

Answer: (E)

Directions – (Q. 28-31) The question consists of a question and two statements numbered I and II given below it. You have to decide whether the date provided in the statements are sufficient to answer the question. Read both the statements and choose the appropriate option and answered as-

(A) Only statement I is sufficient to answer the question

(B) Only statement II is sufficient to answer the question

(C) Both statements I and II together are necessary to answer the question

(D) Either statement I or statement II alone are sufficient to answer the question

(E) Statement I and Statement II together are not sufficient to answer the question

28. What is the value of A?

(I) 3A + B = 2B, 3B + C = 17 and 3C + A = 23

(II) 2A – B = 12, 2B – A = 0

Answer: (D)

29. What is the respective ratio of volume of right circular cylinder A and volume of right circular cylinder B?

(I) The radius of cylinder A is 3/5th of the radius of cylinder B and the height of cylinder of A is 1.2 times the height of cylinder B.

(II) The difference between the volumes of cylinder A and cylinder B is 8736 m3.

Answer: (A)

30. What is the respective ratio of initial investments of A and B?

(I) A started a business by investing a certain amount and he invested for the whole year. B joined A after 6 months from the start of the business and invested for the rest of the year. The profit earned by A and B is in the respective ratio of 8 : 3.

(II) The initial investment of A was Rs 9,200.

Answer: (C)

31. What is A’s mother’s present age ?

(I) The respective ratio between A’s father’s present age and A’s present age is 4 : 1.

(II) The combined present age of A’s father and A is 6 years more than the combined present age of A’s mother and A. 14 years ago, A’s father’s age that time was  times A’s mother’s age that time.

Answer: (B)

Directions – (Q. 32-35) Study the graph and answer the given questions :

32. The number of cats adopted in cities B in June is what per cent less than the number of cats adopted in city B in September?

(A)   

(B)  20

(C)  22

(D)  25

(E)  18

Answer: (A)

33. If the total number of cats adopted in cities A and B together in December was 150 more than that in July, what was the total number of cats adopted in cities A and B together in December?

(A)  650

(B)  730

(C)  720

(D)  660

(E)  680

Answer: (E)

34. What is the ratio between the total number of cats adopted in June in City A and July in City B respectively?

(A)  2 : 3

(B)  3 : 4

(C)  5 : 6

(D)  Other than those given as options

(E)  14/17

Answer: (E)

35. What is the difference between the total number of cat adopted in June and August together and that sold in July and September in both the cities?

(A)  210

(B)  70

(C)  90

(D)  110

(E)  Other than those given as options

Answer: (E)

Directions – (Q. 36-39) Study the table and answer the given question.

Data regarding number of students (boys + girls) in 5 schools-P, Q, R, S, T

36. What is the average number of girls schools P, Q and T?

(A)  160

(B)  170

(C)  150

(D)  165

(E)  140

Answer: (C)

37. What is the respective ratio between the total number of boys in Schools Q and R together that of girls in same schools together?

(A)  3 : 6

(B)  5 : 4

(C)  6 : 7

(D)  7 : 8

(E)  3 : 4

Answer: (E)

38. In an exam, a total of 120 girls from School R and S together failed. If the respective ratio between number of girls who passed from Schools R and S was 5 : 3, how m any girls from Schools S failed the exam?

(A)  95

(B)  35

(C)  60

(D)  25

(E)  30

Answer: (A)

39. What is the difference between the total number of boys in School R and S together and the total number of girls in School (P and Q) together?

(A)  60

(B)  70

(C)  80

(D)  50

(E)  90

Answer: (C)

Directions – (Q. 40-45) Read the graph and answer the question-

40. What is the central angle corresponding to the distance travelled on Monday?

(A)  41.8°

(B)  50.6°

(C)  43.2°

(D)  44.5°

(E)  49°

Answer: (C)

41. If the distance travelled on Sunday (X km) was the sum of 70% and 60% of the distances travelled on Tuesday and Thursday respectively, what was the value of X?

(A)  151

(B)  137

(C)  147

(D)  142

(E)  146

Answer: (C)

42. What is the average distance travelled on Monday, Wednesday and Friday? (in km).

(A)  184

(B)  178

(C)  175

(D)  182

(E)  172

Answer: (C)

43. What is the difference between the total distance travelled on Monday and Wednesday together and the travelled on Thursday and Friday together? (in km)

(A)  40

(B)  65

(C)  10

(D)  30

(E)  25

Answer: (D)

44. What is the total distance travelled on Thursday and Friday together ? (in km)

(A)  200

(B)  250

(C)  300

(D)  350

(E)  400

Answer: (C)

45. What is the ratio between the distance travelled on Monday and distance travelled on Wednesday?

(A)  3 : 4

(B)  5 : 8

(C)  4 : 7

(D)  3 : 8

(E)  5 : 2

Answer: (D)

Directions – (Q. 46-50) Study the following information carefully to answer the given question-

    There are 5 colleges in City ‘ABC’ namely, P, Q, R, S, T. The number of students in College P is 20% of the total number of students in City ‘ABC’. The number of students in College Q is 10% less than the total number of students in College P. The number of students in College R is 50% more than the total number of students in College Q. The number of students in College S is three-fifth of the total number of students in College P. The number of female students in College T is 414, which is one-third of the total number of students in College T.

   The respective ratio between male students and female students in city ‘ABC’ is 5 : 4.

46. What is the average number of students in College P, Q and R?

(A)  1230

(B)  1150

(C)  1120

(D)  1240

(E)  1170

Answer: (E)

47. What is the number of female students in city ABC?

(A)  2800

(B)  2500

(C)  2400

(D)  3000

(E)  2900     

Answer: (C)

48. What is the difference between the total number of students in College R and total number of students in college T?

(A)  150

(B)  216

(C)  170

(D)  180

(E)  168

Answer: (B)

49. Out of the total number of male students in City ‘ABC’ , 1/12 study in College S. What is the number of females studying in college S?

(A)  398

(B)  400

(C)  499

(D)  500

(E)  600

Answer: (A)

50. In College P, 30% of the total students study in Science Stream and the remaining study in Commerce Stream. What is the number of students studying in Commerce Stream?

(A)  724

(B)  756

(C)  748

(D)  750

(E)  759

Answer: (B)

SBI Clerical Cadre (Assistant and Junior Assistant) Preliminary Online Examination Held on May 29, 2016 Question Paper With Answer Key

SBI Clerical Cadre (Assistant and Junior Assistant) Preliminary Online Examination Held on May 29, 2016
SBI Clerical Cadre (Assistant and Junior Assistant) Preliminary Online Examination Held on May 29, 2016 Question Paper With Answer Key

SBI Clerical Cadre (Assistant and Junior Assistant) Preliminary Online Examination Held on May 29, 2016

Part I English Language

Directions (Q. Nos. 1-5) In each of the following questions, a sentence with four words in bold type is given. One of these four words may b e either wrongly spelt or inappropriate in the context of the sentence. Find out the word which is wrongly spelt or inappropriate, if any. That word is your answer. If all the words are correctly spelt and also appropriate in the context of the sentence, mark ‘All correct’ as your answer.

1. The Asian Development Bank has supported the government’s latest efforts to deliver quality health services to all sections of society.

(a)  supported

(b)  latest

(c)  deliver

(d)  sections

(e)  All correct

Answer: (e)

2. On the basis of the forecast by the Indian Meteorological Department the government ordered district collectors to visit districts, which are expected to receive heavy

(a)  ordered

(b)  district

(c)  expected

(d)  rainfal

(e)  All correct

Answer: (d)

3. The economy is on the part to recovery and slowly, but surely the banking sector will perform

(a)  part

(b)  recovery

(c)  surely

(d)  perform

(e)  All correct

Answer: (a)

4. During the course of the day many meetings were held but the agenda was not shared with us.

(a)  During

(b)  course

(c)  held

(d)  agenda

(e)  All correct

Answer: (e)

5. Meazures to increase both financial inclusion and investment in infrastructure will result in gains for the country.

(a)  Meazures

(b)  inclusion

(c)  investment

(d)  gains

(e)  All correct

Answer: (a)

Directions (Q. Nos. 6-15) Read the following story carefully and answer the questions given. Certain words/phrases have been given in bold to help locate them while answering some of the questions.

Once there was a king who had been trying really hard to capture an elephant, but that prize and proved elusive. Al the hunters in the kingdom had tried but their efforts were ‘in vain’. So the king announced that anyone who could capture the elephant would get half his kingdom. The tortoise heard about this and went to the king to accept the challenge. The king was very amused. ‘All my hunters have failed to capture the elephant and you think you can succeed?’, asked the king. The tortoise insisted that he was up to the task and promised to deliver that elephant to the king within forty eight hours. The tortoise then dug a big hole, big enough to hold the elephant along a path leading to the village. Then he covered the hole with sticks and leaves so that it was not visible unless inspected closely. When this was done, the tortoise went in search of the elephant. When the tortoise met the elephant, he told him, ‘You know you are the largest animal in the forest and you should be a king?’ The elephant had never considered this before but he thought it was not a bad idea. The tortoise told the elephant that the villagers had decided to sale the largest animal to their king and were all expecting the elephant to come to the village and be crowned as their king. The more the elephant heard, the more excited he became.

The tortoise adorned the elephant with colourful beads and beating a gong, he sang songs praising the elephant while he led the way to the village. Soon they approached the trap and the tortoise being lighter and smaller walked over the trap. The elephant who was following him fell through the sticks and leaves into the deep hole and thus helped the tortoise in achieving his goal well within the time limit.

6. As mentioned in the story, despite the king doubling his ability, the tortoise accepted the challenge

(A) he was forced to do so by the villagers.

(B) to prove to the king that only a tortoise and his family could complete the challenge.

(C) he wanted the entire kingdom to himself.

(a)  Only A

(b)  A and B

(c)  Only B

(d)  Only C

(e)  None of these

Answer: (e)

7. Which of the following correctly explains the meaning of the phrase, ‘in vain’ as used in the story?

(a)  Were painful

(b)  Were successful

(c)  Were in order

(d)  Were untruthful

(e)  Were simple

Answer: (e)

8. Which of the following could be a suitable title to the story?

(a)  The Elephant who Became the Most Popular King

(b)  The Wise King

(c)  The Clever Tortoise

(d)  The Villagers and their Love for the Elephant

(e)  The Selfish Villagers

Answer: (c)

9. Which of the following is most nearly the opposite in meaning as the word ‘Excited’ as used in the story?

(a)  Bored

(b)  Interested

(c)  Affected

(d)  Feeling

(e)  Merciless

Answer: (a)

10. Which of the following statements is true in the context of the story?

(a)  The king refused to share his kingdom with the tortoise.

(b)  The tortoise kept his word by providing the king with what he desired.

(c)  It was the king’s idea that helped the tortoise achieve his target.

(d)  The elephant was popular for his beautiful skin.

(e)  None of the given statements is true.

Answer: (b)

11. As mentioned in the story, the elephant followed tortoise’s advice because

(A) for him, the tortoise was the wisest animal in the village

(B) he really believed that the villagers wanted him as the king

(C) he had always wanted to become rich and powerful

(a)  Only A

(b)  A and C

(c)  A and B

(d)  Only B

(e)  All of these

Answer: (d)

12. Which of the following is most n early the same in meaning as the word ‘Insisted’ as used in the story?

(a)  Requested

(b)  Revived

(c)  Repeated

(d)  Invited

(e)  Showed

Answer: (c)

13. Which of the following is most nearly the opposite in meaning as the word ‘Praising’ as used in the story?

(a)  Insulting

(b)  Concealing

(c)  Hiding

(d)  Refusing

(e)  Removing

Answer: (a)

14. As mentioned in the story, which of the following can b e said about the elephant?

(a)  He had a good sense of humour

(b)  He was foolish

(c)  He was balanced.

(d)  He was logical.

(e)  He was short tempered.

Answer: (b)

15. Which of the following is most nearly the same in meaning as the word ‘Adorned’ as used in the story?

(a)  Touched

(b)  Adored

(c)  Charmed

(d)  Decorated

(e)  Stuffed

Answer: (d)

Directions (Q. Nos. 16-20) Rearrange the given six sentences/group of sentences A, B, C, D, E and F in a proper sequence so as to form a meaningful paragraph and then answer the given questions.

(A) The next day Shiva appeared at Raghu’s house and spotted a metal chest on the rooftop which had all the money and ornaments that Raghu owned.

(B) When Raghu arrived at Shiva’s house, Shiva asked him to guess what fruit it was, saying that if Raghu failed the (Shiva) would carry one thing out of Raghu’s house and if Raghu guessed correctly then Raghu could carry one thing out of Shiva’s house.

(C) Seeing him Raghu tanked him saying, ‘Thing you for sparing my wealth and carrying out only the ladder!’

(D) One day Shiva hatched a plan to outwit Raghu. He plucked a mango and wrapped it well in a piece of cloth

(E) ‘Oh ho!’ You thought that you’d hide all your wealth from me said Shiva going into Raghu’s house and bringing out a ladder began to climb up onto the roof.

(F) But Raghu’s guesses were incorrect.

16. Which of the following should be the FIFTH sentence after the rearrangement?

(a)  A

(b)  B

(c)  D

(d)  E

(e)  F

Answer: (d)

17. Which of the following should be the SECOND sentence after the rearrangement?

(a)  A

(b)  B

(c)  C

(d)  E

(e)  F

Answer: (b)

18. Which of the following should be the THIRD sentence after the rearrangement?

(a)  A

(b)  B

(c)  C

(d)  D

(e)  F

Answer: (e)

19. Which of the following should be the SIXTH(LAST) sentence after the rearrangement?

(a)  A

(b)  B

(c)  C

(d)  D

(e)  E

Answer: (c)

20. Which of the following should be the FIRST sentence after the rearrangement?

(a)  A

(b)  C

(c)  D

(d)  E

(e)  F

Answer: (d)

Directions (Q. Nos. 21-25) Read the following sentences to find out whether there is any grammatical error in them. The error, if any, will be in one part of the sentence. Mark the part with the errors as you answer. If there is no error, mark ‘No error’ as your answer. (Ignore errors of punctuation, if any)

21. One should always/ be happy from/ whatever one has/ and should not be greedy.

(a)  One should always

(b)  be happy from

(c)  whatever one has

(d)  and should not be greedy

(e)  No error

Answer: (b)

22. My friend lost/ his temper on the /shopkeeper and /slapped him.

(a)  My friend lost

(b)  his temper on the

(c)  shopkeeper and

(d)  slapped him.

(e)  No error

Answer: (e)

23. Luxurious and / expensive lights were / imported from/ various countries.

(a)  Luxurious and

(b)  expensive lights were

(c)  imported from

(d)  various countries

(e)  No error

Answer: (b)

24. They were unhappy because / the person who had a palace/ had no gold and the person / who had gold had no palace.

(a)  They were unhappy because

(b)  the person who had a palace

(c)  had no gold and the person

(d)  who had gold had no palace

(e)  No error

Answer: (e)

25. Despite his / poor financial condition, / the auto-driver remained/ honest and hard work.

(a)  Despite his

(b)  poor financial condition,

(c)  the auto-driver remained

(d)  honest and hard work

(e)  No error

Answer: (d)

Directions (Q. Nos. 26-30) In the following passage, there are blanks, each of the which has been numbered. Against each, five words are suggested, one of which fits the blank appropriately. Find out the appropriate word in each case.

The night fell heavy in the heights of the mountains and one could not see anything. All was black and the moon and the starts were covered by clouds.

As Tom was climbing only a few feet away from the top of the mountain, he (26) and fell in to the air, falling at great speed. he could only see black spots as he went down. He kept failing, when all of a (27) he felt the rope tied to his waist pull at him very hard. His body was hanging in the air. Only the rope was  holding him and in that moment of stillness, he had no other (28) but to scream, ‘Help me God’, All of a sudden a deep voice coming from the sky answered. ‘What did you want me to do?’ ‘Save me God,’ said Tom. ‘Do you really think I can save you?’ asked the voice. ‘Of course you can,’ (29) Tom. ‘Then cut the rope tied to your waist,’ said the voice. There was a moment of silence as Tom decided to hold on to the rope with ‘all his (30). The rescue team said that the next day a climber was found dead and frozen, his body hanging from a rope, only one foot away from the ground.

26.

(a)  stumbled

(b)  talked

(c)  tried

(d)  slapped

(e)  caused

Answer: (a)

27.

(a)  time

(b)  instant

(c)  sudden

(d)  shock

(e)  moment

Answer: (c)

28.

(a)  option

(b)  person

(c)  want

(d)  limitation

(e)  body

Answer: (a)

29.

(a)  asked

(b)  questioned

(c)  replied

(d)  denied

(e)  doubted

Answer: (c)

30.

(a)  strength

(b)  feet

(c)  family

(d)  reason

(e)  portions

Answer: (d)

Part II Reasoning Ability

31. Among five friends-O, P, Q, R and S, each scored different marks in an examination. P scored more than R and Q, but less than O. O did not score the highest marks and did not score the lowest marks. Who amongst the following scored the third highest?

(a)  Cannot be determined

(b)  S

(c)  O

(d)  Q

(e)  O

Answer: (c)

Directions (Q. Nos. 32-36) Study the following information carefully and answer the given questions.

E, F, G, H, I, J, K and L are sitting around a circular table facing the centre but not necessarily in the same order. Only three people sit between K and L. Only two people sit between L and H. Only three people sit between H and G. J sits second to the right of H. I is one of the immediate neighbours of H. F is neither an immediate neighbour of K nor G.

32. Which amongst the following statements is true regarding F, as per the given arrangement?

(a)  None of the given statements is true.

(b)  K is one of the immediate neighbours of F.

(c)  F sits second to the left of H

(d)  F sits third to the right of J.

(e)  Only three people sit between F and E.

Answer: (c)

33. If all the persons are made to sit in alphabetical order in clockwise direction, starting from E, the positions of how many, excluding E, would remain unchanged?

(a)  Three

(b)  Four

(c)  One

(d)  Two

(e)  None

Answer: (d)

34. Four of the following five are alike in a certain way based on their positions in the given arrangement and so form a group. Which is the one that does not belong to the group?

(a)  JHG

(b)  KIE

(c)  GJL

(d)  HFJ

(e)  ILK

Answer: (c)

35. Who among the following sits to the immediate left of L?

(a)  H

(b)  G

(c)  I

(d)  F

(e)  J

Answer: (b)

36. What is the position of H with respect to E?

(a)  Second to the left

(b)  Second to the right

(c)  To the immediate right

(d)  Third to the left

(e)  Third to the right

Answer: (d)

37. In a certain code language, CLUB is coded as ‘FKTE’ and THEN is codes as ‘WGDQ’. In the same code language, ARID will be coded as

(a)  DQHG

(b)  TQHG

(c)  YQKE

(d)  DULC

(e)  DQLC

Answer: (a)

38. In a certain code if ‘a fast train’ is coded as ‘1 5 8’ and ‘train to Mumbai’ is coded as ‘9 3 1’, then what is the code for ‘train’ in the given code language?

(a)  8

(b)  3

(c)  5

(d)  1

(e)  Either 3 or 9

Answer: (d)

Directions (Q. Nos. 39-43) Study the following information and answer the questions.

Seven people, namely J, K, L, M, N, O and P watch seven different channels on seven different days of the same week starting from Monday and ending on Sunday, not necessarily in the same order. K watches a channel to Saturday. More than two people watch channel between K and N. Only one person watches a channel between N and L. J watches a channel on one of the days before L but not on Wednesday. As many people watch a channel between L and P as between J and L. O watches a channel immediately after P.

39. On which of the following days does M watch a channel?

(a)  Friday

(b)  Monday

(c)  Sunday

(d)  Wednesday

(e)  Tuesday

Answer: (c)

40. Which of the following pairs represents the persons who watch a channel immediately before and immediately after K?

(a)  J, P

(b)  M, J

(c)  L, O

(d)  P, L

(e)  O, M

Answer: (e)

41. Which of the following statements is not true as per the given arrangement?

(a)  All the given statements are true.

(b)  Only two people watch a channel after O.

(c)  Only two people watch a channel between M and P.

(d)  N watches a channel immediately before J.

(e)  L watches a channel on Wednesday

Answer: (a)

42. Who amongst the following watches a channel on Thursday?

(a)  M

(b)  J

(c)  L

(d)  O

(e)  P

Answer: (e)

43. Four of the following five are alike in a certain way and thus form a group as per the given arrangement. Which of the following does not belong to the that group?

(a)  OP

(b)  NJ

(c)  LP

(d)  MK

(e)  JK

Answer: (e)

44. If three is added to each even digit and three is subtracted from each odd digit in the number 254673 and then in the new number thus formed all the digits are arranged in descending order, which of the following will be third from the right?

(a)  0

(b)  2

(c)  9

(d)  4

(e)  7

Answer: (d)

Directions (Q. Nos. 45-49) In these questions. relationship between different elements is shown in the statements. The statements are followed by conclusions. Study the conclusions based on the given statements and select the appropriate answer.

Give answer

(a) if only conclusion I is true

(b) if only conclusion II is true

(c) if either conclusion I or II is true

(d) if neither conclusion I nor II is true

(e) if both conclusions are true

45. Statement F ≤ L < U = K ≥ E

Conclusions I. U ≥ F       II. F < K

Answer: (b)

46. Statements B ≥ O ≥ N < K ≤ R; N ≥ F

Conclusions I. O < R       II. F ≤ B

Answer: (b)

47. Statement C ≤ D = E > F ≥G

Conclusions I. C ≤ F       II. G ≥ D

Answer: (d)

48. Statements L > A ≥ M > P; R ≤A ≤ N

Conclusions I. M ≤ N      II. P > R

Answer: (a)

49. Statement P ≤Q ≤R = S < T

Conclusions I. P = T        II. P < T

Answer: (b)

50. How many such p airs of letters are there in the word ‘TEACH’ each of which has as many letters between them in the word (in both forward and backward directions) as they have between them in the English alphabetical series?

(a)  More than three

(b)  None

(c)  Two

(d)  Three

(e)  One

Answer: (c)

Directions (Q. Nos. 51-55) Study the following arrangement carefully and answer the questions.

Y B @ 3 P ⋆ $ 5 T C Z J 4 L 7 R K 8 ⋁ ^ F X Q U π M 6 G N % 9

51. Four of the following five are alike in a certain way based on their positions in the given arrangement and hence form a group. Which one does not belong to that group?

(a)  $5⋆

(b)  6GM

(c)  ZJC

(d)  7RL

(e)  BY@

Answer: (e)

52. How many alphabets are there in the English alphabetical series between the sixth element from the left end and the sixth element from the right end of the given arrangement?

(a)  None

(b)  Two

(c)  One

(d)  Three

(e)  More than three

Answer: (b)

53. Which one of the following will come next in the given sequence?

YE3@, ⋆PE5$, ⋆CT, ?

(a)  CL4

(b)  T4J

(c)  54J

(d)  C4Z

(e)  5JZ

Answer: (e)

54. If all the symbols are deleted from the given arrangement then which of the following will be the ninth element from the right end?

(a)  F

(b)  K

(c)  V

(d)  8

(e)  7

Answer: (a)

55. How many such alphabets are there in the given arrangement each of which is immediately preceded by a number as well as immediately followed by a symbol?

(a)  None

(b)  Three

(c)  More than three

(d)  Two

(e)  One

Answer: (e)

Directions (Q. Nos. 56-57) Study the given information carefully to answer the given questions.

Marcus, who is standing at Point D, walks 11 m towards South and reaches Point E. He then takes a left turn and walks 7 m. He then takes a left turn. walks 5 m and reaches Point F. Point R is 18 m to the East of Point F. Point S is 6 m to the North of Point R.

56. In which direction is Point R with respect to Point D?

(a)  South-West

(b)  East

(c)  West

(d)  North

(e)  South-East

Answer: (e)

57. How far and in which direction is Point S with respect to Point D?

(a)  11 m towards East

(b)  11 m towards West

(c)  17 m towards West

(d)  17 m towards East

(e)  25 m towards East

Answer: (e)

Directions (Q. Nos. 58-60) Study the following information and answer the given questions. S is the father of R. R is the only son of U. U is the daughter of J. J has only two children. K is the son of J.

58. If X is the grand-daughter of J, then how is U related to X?

(a)  Sister

(b)  Mother-in-law

(c)  Grandfather

(d)  Sister-in-law

(e)  Cannot be determined

Answer: (e)

59. How is S related to K?

(a)  Brother

(b)  Brother-in-law

(c)  Cousin

(d)  Father

(e)  Uncle

Answer: (b)

60. How is R related to J?

(a)  Grandfather

(b)  Son

(c)  Nephew

(d)  Grandson

(e)  Son-in-law

Answer: (d)

Directions (Q. Nos. 61-65) The following questions are based on the given 3-digit numbers

  821    547    452    935    368

61. The product of the second and the third digit in which of the following numbers is second highest?

(a)  547

(b)  935

(c)  368

(d)  452

(e)  821

Answer: (a)

62. The difference of the first and the second digits in how many of the given numbers is greater than the third digit of the same number?

(a)  Two

(b)  Three

(c)  More than three

(d)  One

(e)  None

Answer: (a)

63. If all the digits of the given number are arranged in ascending order within the number, what will be the sum of the second and third digits of the second highest number thus formed?

(a)  12

(b)  10

(c)  15

(d)  9

(e)  14

Answer: (e)

64. If the first digit and the third digit of all the given numbers are interchanged, which of the following will become the second lowest number after performing the said operation?

(a)  452

(b)  547

(c)  935

(d)  368

(e)  821

Answer: (a)

65. If ‘2’ is added to the second digit of all even numbers and ‘3’ is subtracted from the first digit of all odd numbers, in how many numbers thus formed will a digit appear twice?

(a)  None

(b)  Three

(c)  More than three

(d)  Two

(e)  One

Answer: (e)

Part III Quantitative Aptitude

66. The circumference of Circle A is 75 m more than its diameter. If the radius of circle B is 3.5 m more than the radius of circle A, what is the circumference of circle B?

(a)  110 m

(b)  140 m

(c)  163 m

(d)  96 m

(e)  132 m

Answer: (e)

67. A shopkeeper sold an article at 20% discount and earned a profit of 4%. By what percent the marked price of the article more than the cost price of the article?

(a)  20%

(b)  15%

(c)  40%

(d)  25%

(e)  30%

Answer: (e)

Directions (Q. Nos. 68-72) What will come in place of question marks in the given number series?

68. 89 86 78 63 39 ?

(a)  19

(b)  4

(c)  9

(d)  13

(e)  28

Answer: (b)

69. 142 70 34 16 ? 2.5

(a)  5

(b)  7

(c)  3

(d)  12

(e)  8

Answer: (b)

70. 21 37 40.2 88.2 94.6 ?

(a)  211.6

(b)  224.5

(c)  192.3

(d)  174.6

(e)  182.8

Answer: (d)

71. 17 9 10 16.5 35 ?

(a)  84

(b)  102.5

(c)  90

(d)  92.5

(e)  72

Answer: (c)

72. 0.75 ? 6 36 288 2880

(a)  3

(b)  1.5

(c)  2

(d)  4.5

(e)  5

Answer: (b)

73. Pihu and Rani start a business together by investing Rs 9000 and Rs 6300 respectively. After certain period, Pihu withdrew from the business completely. If at the end of 2 years, the total profit earned was Rs 13050 and Pihu’s share was Rs 6750, after how many months did Pihu withdraw from business?

(a)  23 months

(b)  14 months

(c)  20 months

(d)  18 months

(e)  12 months

Answer: (d)

74. Jar A contains ‘X’ L of pure milk only. A 27 L mixture of milk and water in the respectively ratio of 4 : 5, is added to jar A. The new mixture thus formed in jar A contains 70% milk, what is the value of X?

(a)  23 L

(b)  30 L

(c)  27 L

(d)  48 L

(e)  28 L

Answer: (a)

75. The speed of a boat in still water is 500% more than the speed of the current. What is the respective ratio between the speed of the boat downstream and speed of the boat upstream?

(a)  9 : 2

(b)  7 : 3

(c)  7 : 5

(d)  9 : 4

(e)  4 : 3

Answer: (c)

Directions (Q. Nos. 76-80) Study the table and answer the given questions

76. Number of employees in organization E increased by, what percent from 2003 to 2006?

(a)  105%

(b)  110%

(c)  115%

(d)  102%

(e)  125%

Answer: (a)

77. Number of employees in organization B and C decreased by 5% and 10% respectively from 2007 to 2008. What was the total number of employees in organization B and together in 2008?

(a)  177

(b)  179

(c)  175

(d)  181

(e)  163

Answer: (a)

78. What is the difference between total number of employees in organization A in 2003 and 2004 together and that in organization D in the same years together?

(a)  77

(b)  79

(c)  73

(d)  83

(e)  67

Answer: (c)

79. What is the respective ratio between total number of employees in organizations B and C together in 2004 and total number of employees in organizations D and E together in 2007?

(a)  21 : 22

(b)  23 : 29

(c)  23 : 27

(d)  21 : 25

(e)  21 : 23

Answer: (e)

80. What is the average number of employees in organization A, B and E in 2005?

(a)  98

(b)  96

(c)  92

(d)  88

(e)  94

Answer: (e)

81. The average runs of a cricketer in tournament, in which he played 14 matches, are 47. His average runs in the first seven matches are 57 and that in the last five matches are 44. If the runs made by him in 8th match are15, how many runs did he make in 9th match?

(a)  24

(b)  32

(c)  26

(d)  22

(e)  28

Answer: (a)

82. An equal amount of sum, Rs P is invested in scheme A and Scheme B. Both the schemes A and B offer simply interest at the rate of 12% and 9% respectively. If at the end of two years total amount received from both the schemes together was Rs 21780, what is the value of P?

(a)  Rs 9000

(b)  Rs 9600

(c)  Rs 12000

(d)  Rs 8400

(e)  Rs 8000

Answer: (a)

Directions (Q. Nos. 83-92) What will come in place of question marks in the given questions?

83. 5/9 × (225.40 – 45.4) = ?2

(a)  15

(b)  5

(c)  100

(d)  25

(e)  10

Answer: (e)

84. 80 ÷4 × ? = 600

(a)  16

(b)  12

(c)  18

(d)  14

(e)  24

Answer: (c)

85. 57 – 1725 ÷ 69 = 4 × ?

(a)  9

(b)  11

(c)  13

(d)  7

(e)  8

Answer: (e)

86. 

(a)  26

(b)  24

(c)  22

(d)  28

(e)  25

Answer: (a)

87. 

(a)  112

(b)  164

(c)  128

(d)  116

(e)  136

Answer: (c)

88. 

(a)  15

(b)  225

(c)  16

(d)  256

(e)  289

Answer: (d)

89. 

(a)  6

(b)  6.4

(c)  8

(d)  4.2

(e)  4

Answer: (e)

90. 

(a)  3.1

(b)  2.53

(c)  2

(d)  2.48

(e)  3.6

Answer: (b)

91. 

(a)  18

(b)  324

(c)  14

(d)  144

(e)  196

Answer: (a)

92. 

(a)  2/3

(b)  2/5

(c)  1/3

(d)  3/5

(e)  4/5

Answer: (d)

93. The simple interest (@ 6% per annum) received on a principal of Rs X was Rs 482.40 when invested for 3 years in scheme A. If scheme B offered compound interest (compounded annually) at 10% per annum, what was the interest received by investing?

‘X-680’ for 2 years in scheme B?

(a)  Rs 420

(b)  Rs 490

(c)  Rs 530

(d)  Rs 540

(e)  Rs 650

Answer: (a)

94. Mary paid 15% of her monthly salary towards an EMI. From the remaining salary, she paid 10% as internet bill and 20% as rent. If after the mentioned expenses she was left with Rs 24990, what was Mary’s monthly salary?

(a)  Rs 45000

(b)  Rs 48000

(c)  Rs 42000

(d)  Rs 36000

(e)  Rs 40000

Answer: (c)

95. The sum of 4 consecutive even numbers is 107more than the sum of three consecutive odd numbers. If the sum of smallest odd number and the smallest even number is 55, what is the smallest even number?

(a)  36

(b)  40

(c)  32

(d)  38

(e)  34

Answer: (d)

96. A, B and C, each working alone, can finish a piece of work in 27, 33 and 45 days respectively. A starts by working alone for 12 days, then B takes over from A and works for 11 days. At this stage C takes over from B and completes the remaining work. In how many days whole work was completed?

(a)  33 days

(b)  31 days

(c)  39 days

(d)  35 days

(e)  37 days

Answer: (a)

97. A rectangular garden of length 12 m is surrounded by a 2 m wide path. If the area of the garden is 84 m2 and the cost of gravelling is Rs 8 per m2, what is the total cost of gravelling the path?

(a)  Rs 780

(b)  Rs 742

(c)  Rs 724

(d)  Rs 775

(e)  Rs 736

Answer: (e)

98. Train A, travelling a S m/sec, can cross a platform double its length in 21 seconds. The same train, travelling at S + 5 m/sec, can cross the same platform in 18 seconds. What is the value of S?

(a)  27.5 m/sec

(b)  32.5 m/sec

(c)  30 m/sec

(d)  35 m/sec

(e)  25 m/sec

Answer: (c)

99. The present age of Charu is 2.5 times the present age of Harsh. Had Harsh been two years younger and Charu been 13 years older, Charu’s age would have been 3.5 times Harsh’s age. What is Harsh’s present age?

(a)  48 years

(b)  25 years

(c)  20 years

(d)  28 years

(e)  36 years

Answer: (c)

100. Divide 27 into two parts so that 5 times the first and 11 times the second together equal to 195. Then ratio of the first and second part is

(a)  17 : 10

(b)  3 : 2

(c)  2 : 7

(d)  5 : 4

(e)  5 : 2

Answer: (a)

SBI Clerical Cadre (Assistant and Junior Assistant) Preliminary Online Examination Held on May 22, 2016 Question Paper With Answer Key

SBI Clerical Cadre (Assistant and Junior Assistant) Preliminary Online Examination Held on May 22, 2016
SBI Clerical Cadre (Assistant and Junior Assistant) Preliminary Online Examination Held on May 22, 2016 Question Paper With Answer Key

SBI Clerical Cadre (Assistant and Junior Assistant) Preliminary Online Examination Held on May 22, 2016

Part I English Language

 

Directions (Q. Nos. 1-10) Read the passage carefully and answer the questions given  below it Certain words/phrases have been given in bold to help you locate them while answering some of the questions.

Once upon a time, there lived a washer man in a village. He had a donkey by the name of Udhata. He used to carry loads of clothes to the river bank and back home everyday. The donkey was not satisfied with the food that was stealthily and at the crops growing there. Once, the donkey, while wandering around, happened to meet a fox. Soon, both of them became friends and began in to wander together in search of delicious food.

One night, the donkey and the fox were eating water-melons in a field. The water-melons were so tasty, that the donkey ate in a large quantity. Having eaten to h is appetite, the donkey became so happy that he was compelled by an intense desire to sing. he told the fox that he was in such a good mood that he had to express his happiness in a melodious tone.

‘Don’t be a fool. If you sing, the people sleeping in and around this field will wake up and beat us black and blue with sticks:’ said the fox worriedly. ‘You are a dull fellow’, the donkey said hearing the words of fox. ‘Singing makes one happy and healthy. No matter what comes.

I’ll definitely sing a song. “The fox became worried to see the donkey adamant to sing a song in the midst of the field, while the owner was still sleeping only a little distance away. Seeing his adamance, he said to the donkey. ‘Friend, wait a minute before you start First, let me Jump over to the other side of the fence for my safety. ‘Saying so the fox jumped over to the other side of the fence without losing a moment. The donkey began in his so-called melodious tone. Hearing, suddenly, a donkey braying in the field, the owner woke up from his sleep. He picked up his stick lying by his side and ran towards the donkey who was still braying happily. The owner of the field looked around and saw the loss caused by the donkey. He became very angry and beat him so ruthlessly that the donkey was physically incapacitated temporarily.

He, somehow, managed to drag himself out of the field with great difficulty. The fox looked at the donkey and said in a sympathetic tone, ‘I’m sorry to see you in this pitiable condition. I had already warned you but didn’t listen to my advice.’ The  donkey too realized his folly and hung his head in shame.

1. Why did donkey want to sing?

(a)  Donkey was so happy that he was compelled by an intense desire to sing.

(b)  He wanted to wake up the people sleeping around.

(c)  Fox fold the donkey to sing a song.

(d)  He wanted to prove that he was a good singer.

(e)  None of the above

Answer: (a)

2. What was donkey’s reaction when fox suggested him not to sing a song?

(a)  Donkey agreed with fox and did not sing a song.

(b)  Donkey chose to dance instead of singing a song.

(c)  Donkey became angry and did not talk to fox after that

(d)  Donkey didn’t pay attention to fox words and sang a song.

(e)  None of the above

Answer: (d)

3. What happened when donkey sang a song?

(a)  The fox clapped for the donkey.

(b)  The villagers praised donkey and gave him a gift.

(c)  The fox started dancing

(d)  The owner woke up and beat the donkey ruthlessly

(e)  None of the above

Answer: (d)

4. How did donkey came to know the fox?

(a)  Both were childhood friends.

(b)  Donkey happened to meet the fox in the field while wandering around in the search of delicious food.

(c)  They used to work together for the washerman from the starting.

(d)  Fox once save donkey life.

(e)  None of the above

Answer: (b)

5. Which of the following statements is false according to the passage?

(a)  The donkey realized his mistake in the end.

(b)  The donkey was not satisfied with the food given to him by his master.

(c)  Donkey didn’t want to sing but he was compelled to do that

(d)  The owner of the field woke after hearing the donkey’s voice

(e)  None of the above

Answer: (c)


Directions (Q. Nos. 6-8) Choose the word/group of words which is most similar in meaning to the word/group of words as used in the passage.

6. Stealthily

(a)  Superstitiously

(b)  Secretly

(c)  Honestly

(d)  Openly

(e)  Overtly

Answer: (b)

7. Compelled

(a)  Discouraged

(b)  Delayed

(c)  Suspended

(d)  Forced

(e)  Demanded

Answer: (d)

8. Adamance

(a)  Easy going

(b)  Yielding

(c)  Flexible

(d)  Mischievous

(e)  Rigid

Answer: (e)

Directions (Q. Nos. 9-10) Choose the word/group of words which is most opposite in meaning of the word/group of words as used in the passage.

9. Folly

(a)  Stupidity

(b)  Madness

(c)  Advanceness

(d)  Sageness

(e)  Sharpness

Answer: (c)

10. Pitiable

(a)  Miserable

(b)  Misfortunate

(c)  Pathetic

(d)  Honourable

(e)  Commendable

Answer: (e)

Directions (Q. Nos. 11-15) Rearrange the following six sentence A, B, C, D, E and F in the proper sequence to form a meaningful paragraph and then answer the questions given below.

(A) Anything that you touch will turn into gold. The king was delighted with his good fortune.

(B) Even though he was very rich he always craved for more and more.

(C) Everything he touched turned into gold. He turned trees, grass, tables, chairs, flowers, and vases into gold.

(D) One day, he called his court magician and commanded, find me a spell that can get me more treasures than I already have.

(E) King Midas was very greedy king.

(F) The magician said, Your majesty, I can give you a power that no one else in this world has.

11. Which of the following should be the FIRST sentence after rearrangement?

(a)  F

(b)  E

(c)  B

(d)  A

(e)  C

Answer: (b)

12. Which of the following should be the SECOND sentence after rearrangement?

(a)  A

(b)  B

(c)  C

(d)  E

(e)  D

Answer: (e)

13. Which of the following should be the THIRD sentence after rearrangement?

(a)  E

(b)  F

(c)  D

(d)  C

(e)  B

Answer: (b)

14. Which of the following should be the FOURTH sentence after rearrangement?

(a)  A

(b)  B

(c)  F

(d)  E

(e)  C

Answer: (a)

15. Which of the following should be the LAST (SIXTH) sentence after rearrangement?

(a)  C

(b)  A

(c)  B

(d)  D

(e)  F

Answer: (c)

Directions (Q. Nos. 16-20) Read each sentence to find out whether there is any grammatical or idiomatic error in it. The error, if any, will be in one part of the sentence. The number of that part is the answer. If there is ‘No error’, the answer is e. (Ignore errors of punctuation, if any.)

16. (a) Harsh Vardhan said he was looking forward / (b) to serve his roots, as he had spend/ (c) the first 15 years of his life/ (d) in the streets of old Delhi. (e) No error

Answer: (b)

17. (a) A family office has to / (b) execute all tasks related to/ (c) manage the wealth of/ (d) the upper-rich family. (e) No error

Answer: (c)

18. (a) Service providers in the financial industry, specially private banks/ (b) and wealth management firms, are increasingly/(c) forced to standardize their/ (d) products to optimize profit. (e) No error

Answer: (e)

19. (a) Social or impact funds are private equity-like funds/ (b) that pool money from investors/ (c) and put it work in a portfolio of ventures/ (d) that meet the funds objective. (e) No error

Answer: (a)

20. (a) While residential land purchases/ (b) have benefitted many investors/ (c) buying land can be risky/ (d) especially for a NRI. (e) No error

Answer: (c)

Directions (Q. Nos. 21-25) In each question below, four words printed in bold type are given. These are indicated as a., b., c. and d. One of these words printed in bold may either be wrongly spelt or inappropriate in the context of the sentence. Find out the word that is inappropriate or wrongly spelt, if any. The number of that word is your answer. If all the words printed in bold are correctly spelt and appropriate In the context of the sentence then mark e. i.e. ‘All correct’, as your answer.

21. All the competitors (a)/ completed (b)/ be race (c)/ with just one (d) All correct (e)

Answer: (d)

22. Poor posture (a)/ can lead (b)/ to muscular (c)/ problems (d)/ in later life. All correct (e)

Answer: (e)

23. The pump (a)/ shut off (b)/ as a result (c)/ of a mechanikal (d)/ failure. All correct (e)

Answer: (d)

24. The Principal (a)/ gave a very pompous (b)/ speach (c)/ about ‘The portals of learning’. (d) All correct (e)

Answer: (c)

25. Copeing (a)/ with her mother’s long illness (b)/ was a heavy load (c)/ to (d) All correct (e)

Answer: (a)

Directions (Q. Nos. 26-30) In the following passage, some of the words have been left out. each of which is indicated by a number. Find the suitable word from the options given against each number and fill up the blanks with appropriate words to make the paragraph meaningfully complete.

A camel and a jackal were friends. One day the jackal (26) his friend to a big sugarcane farm. It was on the opposite side of a river. After a (27) meal, the Jackal began to howl loudly. The frightened camel pleaded (28) the Jackal not to do so. The jackal said, “Friend, I have this habit after every meal. I cannot help it. “Soon the farmers arrived and gave a sound thrashing to the camel.

When the camel crossed the river the jackal joined him on his back. In the midstream, the camel took a deliberate dip (29) the water. When the jackal cried out in terror, the camel said casually “I have the habit of rolling in the water after every meal.” The poor jackal was (30)

26.

(a)  brought

(b)  bought

(c)  took

(d)  taken

(e)  invites

Answer: (c)

27.

(a)  dally

(b)  desirous

(c)  delicacy

(d)  dinner

(e)  sumptuous

Answer: (e)

28.

(a)  to

(b)  for

(c)  on

(d)  with

(e)  before

Answer: (e)

29.

(a)  onto

(b)  in

(c)  within

(d)  down

(e)  on

Answer: (b)

30.

(a)  sank

(b)  sinking

(c)  drowning

(d)  drowned

(e)  wetting

Answer: (d)

Part 2 Reasoning Ability

Directions (Q. Nos. 31-35) To answer these questions study carefully the following arrangement of letters, digits and symbols.

M 7 ∑ 8 L P @ ? 6 N B T Y 3 2 = E $ 4 9 © G H 5

31. How many such letters are there in the arrangement each of which is immediately followed by a number?

(a)  Three

(b)  Four

(c)  One

(d)  Two

(e)  None of these

Answer: (a)

32. How many such symbols are there in the arrangement each of which is immediately preceded by a number?

(a)  Two

(b)  Three

(c)  Four

(d)  Nil

(e)  None of these

Answer: (b)

33. If all the symbols are deleted from t he arrangement, then which of the following will be fourth to the left of the 17th element from the left end?

(a)  9

(b)  E

(c)  2

(d)  Y

(e)  None of these

Answer: (b)

34. ‘78’ is to ‘P?6’ and ‘?N’ is to ‘T32’ in the same way as ‘2E’ is to ………. in the arrangement.

(a)  4©H

(b)  49G

(c)  4©G

(d)  9GH

(e)  None of these

Answer: (c)

35. If all the numbers are deleted from the arrangement then which of the following will be fifth to the right of the 13th element from the right end?

(a)  B

(b)  N

(c)  Y

(d)  T

(e)  None of these

Answer: (d)

Directions (Q. Nos. 36-40) In these questions, a relationship between different elements is shown in the statements. The statements are followed by two conclusions.

Give answer

(a) if only conclusion I is true

(b) if only conclusion II is true

(c) if either conclusion I or II is true

(d) if neither conclusion I nor II is true

(e) if both conclusions are true

36. Statements A > B > C < D C = E > G

Conclusions I. D > E       II. B > E

Answer: (e)

37. Statements P > Q > M > N, Q = S

Conclusions I. S > P        II. N < S

Answer: (b)

38. Statements S > M = Z > T < Q > V

Conclusions I. V = S        II. Q > M

Answer: (d)

39. Statements T < U = V < S > P > Q

Conclusions I. S > T        II. V < Q

Answer: (a)

40. Statements M < N > R > W, E = J > L > W

Conclusions I. E > W       II. M > L

Answer: (a)

Directions (Q. Nos. 41-45) The following questions are based on the five three-digit numbers given below:

684      512    437    385    296

41. If 2 is added to the first digit of each of the numbers how many numbers thus formed will be divisible by three?

(a)  None

(b)  One

(c)  Two

(d)  Three

(e)  None of these

Answer: (b)

42. If all the digits in each of the numbers are arranged in descending order within the number, which of the following will be the highest number in the new arrangement of numbers?

(a)  684

(b)  385

(c)  296

(d)  437

(e)  None of these

Answer: (c)

43. What will be the resultant number if the second digit of the second lowest number is divided by the third digit of the highest number?

(a)  2

(b)  3

(c)  0

(d)  1

(e)  4

Answer: (a)

44. If 1 is added to the first digit and 2 is added to the last digit of each of the numbers then which of the following numbers will be the second highest number?

(a)  385

(b)  684

(c)  437

(d)  296

(e)  512

Answer: (e)

45. If in each number the first and second digits are interchanged then which will be the highest number?

(a)  296

(b)  512

(c)  437

(d)  684

(e)  385

Answer: (a)

Directions (Q. Nos. 46-47) Study the following information carefully and answer the questions given below :

P is to the North of Q and S is to the East of P, who is to the South of W. T is to the West of P.

46. Who among the following is towards South of W and North of Q?

(a)  P

(b)  T

(c)  S

(d)  Q

(e)  None of these

Answer: (a)

47. W is in which direction with respect to T?

(a)  North

(b)  North-East

(c)  South-West

(d)  West

(e)  None of these

Answer: (b)

Directions (Q. Nos. 48-52) Study the following information carefully and answer the questions given  below :

Dhondu, Chintu, Titu, Chiku, Sonu, Monu, Bittu and Sonty are sitting around a circular table facing the centre. Sonty is third to the right of Titu and second to the left of Sonu. Chintu is not an immediate neighbour of Sonty or Titu. Monu is second to the right of Chiku and is in immediate neighbour of Titu. Bittu is not the neighbour of Sonu.

48. Who among the following is second to the right of Titu?

(a)  Sonty

(b)  Bittu

(c)  Monu

(d)  Sonu

(e)  None of these

Answer: (b)

49. Who among the following is an immediate neighbour of Sonty and Sonu?

(a)  Dhondu

(b)  Chintu

(c)  Titu

(d)  Bittu

(e)  None of these

Answer: (a)

50. In which of the following pairs the second person is sitting on the immediate right of the first person?

(a)  Dhondu, Sonty

(b)  Titu, Chiku

(c)  Bittu, Sonty

(d)  Sonu, Sonty

(e)  Monu, Titu

Answer: (c)

51. Who among the following is second to the left of Chintu?

(a)  Titu

(b)  Sonty

(c)  Monu

(d)  Dhondu

(e)  None of these

Answer: (d)

52. Who among the following is opposite to Chiku?

(a)  Dhondu

(b)  Bittu

(c)  Sonty

(d)  Sonu

(e)  None of these

Answer: (c)

Directions (Q. Nos. 53-57) Study the following information carefully to answer the given questions. Seven neighbours S, P, L, Q, R, M and I live on different floors in the same building having seven floors numbered one to seven. (The first floor is numbered one, the floor above it is numbered two and so on and the topmost floor is numbered as seven.) Three persons live between I and M. M lives on the floor above S, who lives on an odd-numbered floor. P is neither live on odd number nor on topmost floor. L lives on the first floor. Two persons live between I and S. Q lives neither on the first floor nor on the third floor.

53. Who lives on the floor just above M?

(a)  I

(b)  P

(c)  Q

(d)  R

(e)  Noone of these

Answer: (c)

54. How many persons live between L and P?

(a)  None

(b)  One

(c)  Two

(d)  Three

(e)  Can’t be determined

Answer: (c)

55. Which of the following pairs live on the first floor and the topmost floor respectively?

(a)  L Q

(b)  Q, P

(c)  I, Q

(d)  L, I

(e)  Can’t be determined

Answer: (a)

56. Who among the following lives on the topmost floor?

(a)  I

(b)  Q

(c)  P

(d)  L

(e)  None of these

Answer: (b)

57. Which of the following combinations is true?

(a)  First floor-S

(b)  Fourth floor-R

(c)  Third floor-M

(d)  Sixth floor-I

(e)  None of the above

Answer: (e)

58. How many pairs of letters are there in the word (in forward direction) APPLICATION, each of which have many letters between them in the word as they have between them in the English alphabet?

(a)  One

(b)  Two

(c)  Three

(d)  Four

(e)  None of these

Answer: (a)

59. In a certain coding system, PAPER is written as PERPA and SUBJECT is written as JECTSUB, what should be the code for COUNCIL?

(a)  NCILCOU

(b)  LICNOUC

(c)  NCOUCIL

(d)  NLICUOC

(e)  None of these

Answer: (a)

60. In certain code language ‘lu ja ka hu’ means ‘will you meet us’, ‘lu ka hu pa’ means ‘will you sold us’ Then what is the code of ‘meet’ in this code language?

(a)  ja

(b)  lu

(c)  ka

(d)  hu

(e)  cannot be determined

Answer: (a)

61. In a certain code language, COMBINE is written as XLNYRMV. How will TOWARDS be written in that code language?

(a)  FLDZIWJ

(b)  GLDZIWH

(c)  GLEZJWH

(d)  FLEZIWH

(e)  None of these

Answer: (b)

62. 37 girls are standing in a row facing the school building. Ayesha is fifteenth from the left end. If she is shifted six places to the right what is her position from the right end?

(a)  16th

(b)  21st

(c)  20th

(d)  18th

(e)  None of these

Answer: (e)

63. X is mother is the mother-in-law of the father of Z. Z is the brother of Y while X Is the father of M. How is X related to Z?

(a)  Paternal uncle

(b)  Maternal uncle

(c)  Cousin

(d)  Grandfather

(e)  Brother-in-law

Answer: (b)

64. If A is the brother Of B, C is the sister of A, D is the brother of E, E is the daughter of B, F is the father of C. Who is the uncle of D?

(a)  A

(b)  C

(c)  B

(d)  None of these

(e)  Can’t be determined

Answer: (a)

65. A said to B that B’s mother was the mother-in-law of A’s mother. How is A’s mother related to B’s mother?

(a)  Daughter-in-law

(b)  Mother-in-law

(c)  Sister

(d)  Aunt

(e)  None of these

Answer: (a)

Part 3 Quantitative Aptitude

Directions (Q. Nos. 66-70) What should come in the place of question marks in the following questions?

66. 

(a)  133.4

(b)  137.2

(c)  127.8

(d)  131.6

(e)  None of these

Answer: (a)

67. 12% of 555 + 15% of 666 = ?

(a)  166.5

(b)  167.5

(c)  168.5

(d)  169.5

(e)  None of these

Answer: (a)

68. 84368 + 65466 – 72009 – 13964 = ?

(a)  61481

(b)  62921

(c)  63861

(d)  64241

(e)  None of these

Answer: (c)

69. 338 × 331.2 ÷ 335 = 33 × 33?

(a)  2.8

(b)  3

(c)  3.2

(d)  4

(e)  6

Answer: (b)

70. 

(a)  404

(b)  408

(c)  410

(d)  414

(e)  416

Answer: (e)

Directions (Q. Nos. 71-75) Study the following table carefully and answer the given questions.

71. The total number of various crimes in Himachal Pradesh is

(a)  37803

(b)  38903

(c)  37903

(d)  36903

(e)  37003

Answer: (d)

72. Find the ratio of Stalking and Assault in Uttar Pradesh to Theft and Criminal Trespass in Haryana.

(a)  28 : 51

(b)  21 : 52

(c)  52 : 21

(d)  14 : 55

(e)  55 : 14

Answer: (b)

73. Find the the approximate average of Murder and Theft all the eight States together.

(a)  1141

(b)  1132

(c)  1311

(d)  941

(e)  1021

Answer: (e)

74. The total number of Assaults and Murders together in Bihar is, what percent of the total number of crimes in that State?

(a)  29.82%

(b)  39.82%

(c)  25%

(d)  21.82%

(e)  25.5%

Answer: (b)

75. Find the difference between the number various crimes committed in Bihar and that in Rajasthan.

(a)  105

(b)  98

(c)  145

(d)  139

(e)  104

Answer: (c)

Directions (Q. Nos. 76-80) What should come in the place of question marks in the following questions?

76. 4376 + 3209 – 1784 + 97 = 3125 + ?

(a)  2713

(b)  2743

(c)  2773

(d)  2793

(e)  2737

Answer: (c)

77. 

(a)  1521

(b)  1369

(c)  1225

(d)  961

(e)  1296

Answer: (b)

78. 85% of 420 + ?% of 1080 = 735

(a)  25

(b)  30

(c)  35

(d)  40

(e)  45

Answer: (c)

79. 

(a)  8920

(b)  8940

(c)  8960

(d)  8820

(e)  8640

Answer: (d)

80. 30% of 1225 – 64% of 555 = ?

(a)  10.7

(b)  12.3

(c)  13.4

(d)  17.5

(e)  None of these

Answer: (b)

81. How many litres of water should be added to a 30 L mixture milk and water containing milk and water in the ratio of 7 : 3 such that the resultant mixture has 40% water in it?

(a)  5 L

(b)  2 L

(c)  3 L

(d)  8 L

(e)  7 L

Answer: (a)

82. The SI on certain sum of money for 15 months at rate of 7.5% per annum exceed the SI on same sum at 12.5% per annum for 8 months by Rs 3250 find sum?

(a)  Rs 160000

(b)  Rs 20000

(c)  Rs 170000

(d)  Rs 18000

(e)  Rs 3120000

Answer: (e)

83. 4 men and 3 women finish a job in 6 days, and 5 me and 7 women can do the same job in 4 days. How long will 1 man and 1 woman to do the work?

(a) 

(b) 

(c) 

(d) 

(e)  None of these

Answer: (a)

84. A and B started a business with initial investments in the ratio 5 : 7. If after one year their profits were in the ratio 1 : 2 and the period for A’s investment was 7 months, B invested the money for

(a)  6 months

(b) 

(c)  10 months

(d)  4 months

(e)  7 months

Answer: (c)

85. An army lost 10% its men in war, 10% of the remaining due to diseases and 10% of the rest were disabled. Thus, the strength was reduced to 729000 active men. Find the original strength.

(a)  1000000

(b)  1200000

(c)  1500000

(d)  1800000

(e)  None of these

Answer: (a)

86. What is the difference between the compound interests on Rs 5000 for 1 years at 4% per annum compounded yearly and half-yearly?

(a)  Rs 2

(b)  Rs 3

(c)  Rs 4

(d)  Rs 8

(e)  None of these

Answer: (a)

87. The speeds of John and Max are 30 km/h and 40 km/h. Initially, Max is the place L and John is at a place M. The distance between L and M is 650 km. John started his journey 3 hours earlier than Max to meet each other. If they meet each other at a place P somewhere between L and M, then the distance between P and M is

(a)  220 km

(b)  250 km

(c)  330 km

(d)  320 km

(e)  None of these

Answer: (c)

88. The average weight of boys in a class is 30 kg and the average weight of girls in the same class is 20 kg. If the average weight of the whole class is 23.25 kg, what could be the possible strength of boys and girls respectively in the same class?

(a)  14 and 26

(b)  13 and 27

(c)  17 and 27

(d)  19 and 21

(e)  None of these

Answer: (b)

89. A profit of 8% is made by selling a shirt after offering a discount of 12%. If the marked price of the shirt is Rs 1080, find its cost price.

(a)  Rs 890

(b)  Rs 780

(c)  Rs 880

(d)  Rs 900

(e)  None of these

Answer: (c)

90. The difference between 4/5 of a number and 45% of the number is 56. What is 65% of the number?

(a)  96

(b)  104

(c)  112

(d)  120

(e)  None of these

Answer: (b)

91. A man can row 24 km upstream and 54 km downstream in 6 hours. He can also row 36 km upstream and 48 km downstream is 8 hours. What is the speed of the man in still water?

(a)  18.75 km/h

(b)  19.25 km/h

(c)  17.65 km/h

(d)  15.55 km/h

(e)  22.75 km/h

Answer: (b)

92. The numerator of a fraction is decreased by 25% and the denominator is increased by 250%. If the resultant fraction is 6/5, what is the original fraction?

(a)  22/5

(b)  24/5

(c)  27/6

(d)  28/5

(e)  30/11

Answer: (d)

93. What would be the area of a rectangle whose area is equal to the area of a circle of radius 7 cm?

(a)  77 cm2

(b)  154 cm2

(c)  184 cm2

(d)  180 cm2

(e)  150 cm2

Answer: (b)

94. In a village, three people contested for post of village Pradhan. Due to their own interest, all the voters voted and no one vote was invalid. The losing candidate got 30% votes. What could be the minimum absolute margin of votes by, which the winning candidate led by the nearest rival, if each candidate got an integral percent of votes?

(a)  4

(b)  2

(c)  1

(d)  3

(e)  None of these

Answer: (b)

95. The price of an article is first increased by 20% and later on the price were decreased by 25% due to reduction in sales. Find the net percentage change in final price of article.

(a)  20%

(b)  18%

(c)  38%

(d)  10%

(e)  None of these

Answer: (d)

Directions (Q. Nos. 96-100) What will come in the place of the question marks in the following number series?

96. 48, 23, ?, 4.25, 1.125

(a)  10.5

(b)  10

(c)  2.5

(d)  11

(e)  None of these

Answer: (d)

97. 2, 15, 41, 80, 132, ?

(a)  197

(b)  150

(c)  178

(d)  180

(e)  None of these

Answer: (a)

98. ?, 15, 75, 525, 4725, 51975

(a)  5

(b)  10

(c)  8

(d)  6

(e)  None of these

Answer: (a)

99. 4, 19, 49, ?, 229

(a)  75

(b)  109

(c)  65

(d)  169

(e)  None of these

Answer: (b)

100. 840, ?, 420, 140, 35, 7

(a)  408

(b)  840

(c)  480

(d)  804

(e)  None of these

Answer: (b)

SBI P.O. (Pre.) Examination Held on 06-05-2017 Reasoning Question Paper With Answer Key

SBI P.O. (Pre.) Examination Held on 06-05-2017 Reasoning
SBI P.O. (Pre.) Examination Held on 06-05-2017 Reasoning Question Paper With Answer Key

SBI P.O. (Pre.) Examination Held on 06-05-2017

Reasoning

1. C is the mother-in-law of Z. Z is the only child of P. P is the grandfather of S. S is the niece of T. If C has only one daughter, how is T definitely related to Z?

(A)  Daughter-in-law

(B)  Brother-in-law

(C)  Sister

(D)  Cannot be determined

(E)  None of the above

Answer: (B)

2. In vertical queue of 15 people all facing north with equal distance between each other, P stands third from one of the extreme ends of the queue. Only six people stand between P and A. The number of people standing before A is same as that standing after G. Q stands exactly between A and G. How many people stand after Q?

(A)  Seven

(B)  Four

(C)  Twelve

(D)  Fourteen

(E)  None of the above

Answer: (A)

Directions – (Q. 3-7) Study the given information carefully to answer the given questions-

Eight people – J, K. L, M, N, O, P and Q were born in different years viz. 1955, 1963, 1977, 1982, 1988, 1993, 2000 and 2006, but not necessarily in the same order.

Note- (A) All calculations are done with respect to the present year, 2017 assuming the month and date to be the same as that of the years of birth as mentioned above.

  (B) Each person is assumed to be born on the same date and same month of respective years.

M was born after 1993. The sum of the present ages of M and Q is 46. The difference between the present ages of Q and P is 5. L’s age is exactly divisible by 9. The difference between the present ages of N and J is less than 6. J is older than N. O was born in an even numbered year. O is not 11 year old.

3. How many person/s is/are younger than M?

(A)  Three

(B)  One

(C)  Two

(D)  None

(E)  More than three

Answer: (D)

4. Which of the following statements is true regarding K as per the given arrangement?

(A)  The difference between the present ages of K and P is less than 12

(B)  K was born in an even numbered year

(C)  None of the given statements is true

(D)  K is younger than L.

(E)  K is 11 years old

Answer: (C)

5. If A is older than O, but younger than J, then which of the following can possibly be the present age of A (in years)?

(A)  31

(B)  13

(C)  15

(D)  34

(E)  28

Answer: (E)

6. Who amongst the following was born in the year 1977?

(A)  J

(B)  K

(C)  L

(D)  P

(E)  N

Answer: (D)

7. Which of the following represents the presents ages of J and L (in years) respectively?

(A)  35, 54

(B)  24, 40

(C)  24, 62

(D)  29, 54

(E)  29, 40

Answer: (D)

Directions – (Q. 8-10) Study the following information and answer the given questions-

    Each of the six stores – D, E, F, G, H and I sold different number of fruits in one day. Only two stores sold less fruits than I. F sold less fruits than D but more than G and I. G sold less fruits than I. H sold more fruits than D. The store which sold the third lowest number of fruits sold 62 fruits.

(Note – The number of fruits sold is in whole numbers).

8. If the total number of fruits sold by E is 60, with reference to the total number of fruits sold, If E + D = 135, then D + F = …….?

(A)  122

(B)  111

(C)  129

(D)  139

(E)  150

Answer: (D)

9. If the total number o fruits sold by E is 60, then who amongst the following sold the lowest number of fruits?

(A)  G

(B)  D

(C)  F

(D)  Cannot be determined

(E)  E

Answer: (D)

10. How many fruits did H probably sell?

(A)  61

(B)  54

(C)  81

(D)  49

(E)  30

Answer: (C)

11. In a certain code language, ‘out of order’ is written as ja bi gf’, ‘order of court’ is written as ‘st gf ja’ and ‘out of court’ is written as ‘dc st bi’. How is ‘order’ written in that code language (All the given codes are two letter codes only)?

(A)  Either ‘dc’ or ‘st’

(B)  dc

(C)  st

(D)  Either ‘gf’ or ‘ja’

(E)  bi

Answer: (D)

12. What should come in place of $ and # respectively in the expression V > U > L ≥ T $ R; M ≥ O # V, so that the expression R < M definitely holds true?

(A)  >, >

(B)  <, <

(C)  ≤, <

(D)  None of those given as options

(E)  Relationship cannot  be possible

Answer: (A)

Directions – (Q. 13-15) In this questions, relationship between different elements is shown in the statements. The statements are followed by two conclusions. Study the conclusions based on the given statements and select the appropriate answer-

(A) Only conclusion I is true

(B) Only conclusion II is true

(C) Either conclusion I or II is true

(D) Neither conclusion I nor II is true

(E) Both conclusion I and II are true

13. Statements : S ≤ P ≤ A ≥ R ≥ O; E ≥ L ≥ A ≤ W ≤ Y

Conclusions I. E ≥ S

II. O ≤ Y

Answer: (E)

14. Statements : C ≥ R ≥ E < A < P; G > I ≥ C ≤ O ≤ N

Conclusions : I. N ≥ P

II. G > A

Answer: (D)

15. Statements : C ≥ R ≥ E < A < P; G > I ≥ C ≤ O ≤ N

Conclusions : I. I > R

II. I = R

Answer: (C)

Directions – (Q. 16-20) Study the given information carefully to answer the given questions –

   A, B, C, D, P, Q, R and S have a party in either April or December of the same year which will be organized only on the four days viz. 4th, 17th, 20th and 29th of each month. No two people have a party on the same day.

A has a party in December on an odd numbered date. Only three people have a party between A and C. The number of people having a party after C is same as those having a party before B. Only one person has a party between P and B. P does not have to arrange a party in April. Both S and R have to arrange a party on the same month. S has to party on one of the days after R. Only two people have to arrange a party between S and Q. D have to arrange a party on 4th April.

16. How many people have to attend a party between Q and D?

(A)  None

(B)  One

(C)  Three

(D)  Two

(E)  More than three

Answer: (E)

17. When does Q have to arrange party?

(A)  4th April

(B)  29th December

(C)  17th December

(D)  Cannot be determined

(E)  20th April

Answer: (C)

18. Who among the following have to arrange a party on 4th April and 4th December?

(A)  B, C

(B)  R, S

(C)  P, B

(D)  B, D

(E)  P, C

Answer: (D)

19. Which of the following statements is true with respect to the given information?

(i) S has to arrange a party on 4th December.

(ii) Only two people have to arrange a party between A and Q.

(iii) C has to arrange a party on one of the days after R.

(A)  None of the given statements is true

(B)  Both (i) and (ii)

(C)  Both (ii) and (iii)

(D)  Only (i)

(E)  Only (iii)

Answer: (E)

20. In the given arrangement, If Q and A interchange their position and so do C and P, then how many people will have to arrange a party between A and C?

(A)  Three

(B)  Two

(C)  One

(D)  More than three

(E)  None

Answer: (E)

Directions – (Q. 21-25) Eight persons – A, B, C, D, P, Q, R and S all of them facing north in a straight line. Each person is having different age 14, 16, 21, 26, 28, 17, 29 and 31.

   B sits one of the extreme end of the line. Q’s age is not a prime number. There are three persons sits between one whose age is 16 and B. One whose age is 29 sits second to the left of C. C sits to the right of one whose age is 16 but not immediately. D sits to the third position from the left end. The one who is eldest among all is not an immediate neighbour of D, who is not eldest among all. P sits to the immediate right of one who is eldest. C’s age is neither 14 nor 17. Difference of D’s immediate neighbour’s age is 5. There are two persons sits between one whose age is 26 and S, who sits to the right of one whose age  is 26. R is not an immediate left of S. Q sits third to the right of A.

21. Who among the following persons sits second to the right of S?

(A)  B

(B)  C

(C)  Q

(D)  A

(E)  None of these

Answer: (C)

22. What is the age of one who sits to the immediate right of P?

(A)  17

(B)  28

(C)  14

(D)  26

(E)  Cannot be determined

Answer: (A)

23. Four of the following five are alike certain way based on the arrangement and hence form a group. Which one does not belong to that group?

(A)  14, R

(B)  P, 21

(C)  26, B

(D)  Q, 29

(E)  S, 21

Answer: (B)

24. Who among the following person are immediate neighbours of one whose age is 14?

(A)  R and Q

(B)  C and S

(C)  S and B

(D)  R and C

(E)  None of the above

Answer: (D)

25. Which of the following statement is/are definitely true?

(A)  One who sits third to the right of S is Q, whose age is 14

(B)  A’s immediate neighbours age is consecutive numbers

(C)  R is an immediate neighbour of S and C

(D)  A’s age is 21 and R’s age is not a prime number

(E)  Both B and D

Answer: (B)

26. A person starts from point A, walks 7m towards north and reaches Point B. He then takes right turn, followed by a right turn and walks for 10 m. He then takes a right turn and walks 14 m. He takes a final left turn, walks 3m and reaches Point C. Point Z is 10 m to the north of Point C. Point Z is in which direction with respect to Point B?

(A)  North-West

(B)  South-East

(C)  South-West

(D)  East

(E)  None of the above

Answer: (C)

27. Five movies- D, E, F, G and H are to be release on five different days of the same week starting from Monday and ending on Friday, but not necessarily in the same order. E will be released on one of the days after Tuesday but not on Wednesday. Only two movies will be released between E and F. D will be released immediately before F. Only one movie will be released between D and H, which move is released immediately after G?

(A)  Cannot be determined

(B)  H

(C)  E

(D)  D

(E)  None of the above

Answer: (C)

Directions – (Q. 28-32) Read the given information to answer the given questions-

   Eight people viz-Z, V, N, E, L, Q, H and U live on eight different floors of a building. The ground floor of the building is numbered one, the one above that is numbered two and so till the topmost floor is numbered eight. All of them play a different video games viz. Batman, Darkstone, Gothic, Magicka, Jurassic Park, Rayman, Payday and Victoria.

(Note– None of the given information is necessarily in the same order.)

E lives on floor number two. No one lives between E and the one who plays Batman. Only four people live between U and the one who plays Batman. Only one person lives between U and the one who plays Magicka. The one who plays Magicka does not live on the topmost floor. As many people live between the one who plays Batman and Magicka as between L and Q. L lives on an odd numbered floor above Q. As many people live between the one who plays Batman and Q as between the one who plays Darkstone and Q. The one who plays Payday lives on an even numbered floor immediately above the one who plays Victoria but not floor be low two. Only two people live between H and Z. H lives on one of the floors above Z. The one who plays rayman lives immediately below the one who plays Gothic. Only one people live between V and N. V lives on one of the floors above N.

28. Four of the following five are alike in a certain way based on the given arrangement and thus form a group. Which one of the following does not belong to the group?

(A)  V-Floor number six

(B)  Q-Victoria

(C)  Z-Magicka

(D)  N-Victoria

(E)  E-Dark Stone

Answer: (C)

29. Who plays Jurassic Park?

(A)  E

(B)  Z

(C)  L

(D)  H

(E)  N

Answer: (D)

30. Which of the following statements is true as per the given arrangement?

(A)  Z lives on floor number five

(B)  Q plays Magicka

(C)  Only one person lives between U and H

(D)  V lives immediately above E

(E)  All the given statements are true

Answer: (E)

31. Which game does U play?

(A)  Victoria

(B)  Payday

(C)  Rayman

(D)  Darkstone

(E)  Batman

Answer: (B)

32. How many people live between the one who plays Gothic and U?

(A)  Three

(B)  Two

(C)  One

(D)  Four

(E)  None

Answer: (B)

33. In the number 2698345, three is subtracted from each digit greater than five. Five is added to each digit less than four. All other digits are kept unchanged. In the new number thus formed, which of the following digits will appear more than once?

(A)  None

(B)  Only 3

(C)  Both 3 and 6

(D)  Both 3 and 4

(E)  Only 5

Answer: (E)

34. Leena placed 4 envelops-A, B, C and D above one another but not necessarily in the same order. She kept on of the items in each of the envelopes-Tickets, Passport, Marksheets and Stationary but not necessarily in the same order. The envelope containing Passport is kept immediately below A. No envelope is kept between A and C. The envelope containing Tickets is kept immediately above the envelope containing Marksheets. A does not contain Marksheets. What is the position of the envelope containing Stationary from the top of the stack?

(A)  Fourth

(B)  Cannot be determined

(C)  Second

(D)  First

(E)  Third

Answer: (B)

35. In which of the following expression does the expression K > N definitely holds false?

(A)  Q ≤ U = B ≤ K; N < T < Q< R

(B)  Q = U = B < K; N < T < Q > R

(C)  Q < U < B > K: N = T < Q > R

(D)  Q = U ≤ B < K; N ≤ T ≤ Q > R

(E)  None of the above

Answer: (C)

SBI P.O. (Pre.) Examination Held on 09-07-2016 1st-Shift Reasoning Question Paper With Answer Key

SBI P.O. (Pre.) Examination Held on 09-07-2016 1st-Shift Reasoning
SBI P.O. (Pre.) Examination Held on 09-07-2016 1st-Shift Reasoning Question Paper With Answer Key

SBI P.O. (Pre.) Examination Held on 09-07-2016 1st-Shift

Reasoning

 

Directions- (Q. 1 to 3) Study the following information and answer the given question.

    S is the mother of D. K is the brother of D. K is the only son of M. M is the son of of U. U is the husband of T. T is the mother of Y.

1. If D is married to X, then how is X related to M?

(A)  Son-in-law

(B)  Daughter-in-law

(C)  Son

(D)  Niece

(E)  Daughter

Answer: (A)

2. How is T related to K?

(A)  Mother

(B)  Aunt

(C)  Grand-daughter

(D)  Grandmother

(E)  Mother-in law

Answer: (D)

3. How is S related to Y?

(A)  Sister

(B)  Cousin

(C)  Mother-in-law

(D)  Niece

(E)  Sister-in-law

Answer: (E)

Directions – (Q. 4 to 8) In this question, relationship between different elements is shown in the statements. The statements are followed by conclusions. Study the conclusions based on the given statement and select the appropriate answer.

4. Statement :

R > S = T ≥ U; S ≥ A > V

Conclusions : I. A < R

II. V ≤ U

(A)  Both conclusion I and II are true

(B)  Only conclusion I is true

(C)  Neither conclusion I nor II is true

(D)  Only conclusion II is true

(E)  Either conclusion I or conclusion II is true

Answer: (B)

5. Statements :

B > E ≥ A ≥ T = H < I ≤ M

Conclusions : I. H ≤ E

II. B > T

(A)  Both conclusion I and II are true

(B)  Only conclusion I is true

(C)  Neither conclusion I nor II is true

(D)  Only conclusion II is true

(E)  Either conclusion I or conclusion II is true

Answer: (A)

6. Statements :

S < M < I < T ; R ≥ J > I

Conclusions : I. R = S

II. S < R

(A)  Both conclusion I and II are true

(B)  Only conclusion I is true

(C)  Neither conclusion I nor II is true

(D)  Only conclusion II is true

(E)  Either conclusion I or conclusion II is true

Answer: (D)

7. Statements :

B > E ≥ A ≥ T = H < I ≤ M

Conclusions : I. E < I

II. M ≤ A

(A)  Both conclusion I and II are true

(B)  Only conclusion I is true

(C)  Neither conclusion I nor II is true

(D)  Only conclusion II is true

(E)  Either conclusion I or conclusion II is true

Answer: (C)

8. Statements :

D ≤ O ≤ L > C ≥ E

Conclusions : I. O < E

II. L ≥ D

(A)  Both conclusion I and II are true

(B)  Only conclusion I is true

(C)  Neither conclusion I nor II is true

(D)  Only conclusion II is true

(E)  Either conclusion I or conclusion II is true

Answer: (D)

Directions – (Q. 9 and 10) Read the following information and answer the given question.

   Vansh starts walking from point E and walks 25 m towards the north. He then takes a right turn and walks for 15 m. He makes a left turn and stops at Point M after walking for 20 m.

     Point K is 30 m to the west of Point M. Point K is 45 m to the north of Point J. Point J is 10 m to the east of Point L.

9. How far and in which direction is Point E with respect to Point L?

(A)  30 m towards West

(B)  25 m towards East

(C)  20 m towards East

(D)  20 m towards West

(E)  25 m towards West

Answer: (B)

10. If Neha is standing at Point D which is 20 m to the north of Point L, in which direction will she have to walk to order to reach Point M?

(A)  North-west

(B)  East

(C)  South-East

(D)  North-East

(E)  North

Answer: (D)

Directions – (Q. 11 to 13) Read the given information carefully and answer the question.

   Each of the six sections, U, V, W, X, Y and Z of the same class has different number of students. Only Z has more number of students than X. V has more number of students than Y but less than U. W has more number of students than both the both Y and U. The section having the third highest number of students has 39 students. Y has 24 students.

11. If the number of students in sections W + Z is sixty-six more than the number of students in section Y, how many of students are there in section Z?

(A)  31

(B)  46

(C)  51

(D)  55

(E)  45

Answer: (C)

12. How many students dos section V possibly have?

(A)  39

(B)  43

(C)  55

(D)  31

(E)  14

Answer: (D)

13. Which of the following is true regarding the number of students is section U?

(A)  No other section has less students than U

(B)  X has more number of students than U

(C)  U possibly has 45 students

(D)  U has more number of students than only three sections

Answer: (B)

Directions – (Q. 14 to 19) Study the following information and answer the question.

   Seven people, namely, A, B, C, D, E, F and G have a an appointment but not necessarily in the same order, on seven different months (of the same year) namely January, February, April, June, August, October and December. Each of them also likes a different activity namely Drawing, Singing, Painting, Boxing, Karate, Craft and Running but not necessarily in the same order.

   The one who likes Craft has an appointment on one of the months before April. Only two people have an appointment between the one who likes craft and the one who likes painting. Only one person has an appointment between the one who likes painting and the one who likes running. The one who likes running has an appointment in a month which has 31 days. Only three people have an appointment between the one who likes running and E. G has an appointment on one of the months before E. G does not have an appointment in the month which has the least number of days. Only three people have an appointment between G and C. Only one person has an appointment between C and the one who likes Karate. The one who likes Karate has an appointment before C. The one who likes singing has an appointment immediately before B. B has an appointment in a month which has less than 31 days. Only one person has an appointment between A and F. A has an appointment before F. Only one person has an appointment between F and the one who likes drawing.

14. Who amongst the following has an appointment before the one who has an appointment in December?

(A)  B

(B)  E

(C)  C

(D)  The one who likes Running

(E)  The one who likes Boxing

Answer: (E)

15. In which of the following pairs, both the persons have an appointment in months which have less than 31 days?

(A)  A, F

(B)  F, D

(C)  B, F

(D)  E, D

(E)  B, A

Answer: (E)

16. Which of the following combinations is correct?

(A)  G-Singing

(B)  A-Painting

(C)  F-Running

(D)  B-Karate

(E)  D-Painting

Answer: (A)

17. As per the given arrangement Craft is related to April and Karate is related to June following a certain pattern, which of the following is Drawing related to following the same pattern?

(A)  February

(B)  October

(C)  December

(D)  August

(E)  January

Answer: (D)

18. Who amongst the following likes Running?

(A)  F

(B)  D

(C)  A

(D)  B

(E)  Other than those given as options

Answer: (B)

19. How many people have an appointment between the one who has an appointment in February and A?

(A)  Three

(B)  None

(C)  More than three

(D)  One

(E)  Two

Answer: (D)

Directions – (Q. 20 to 24) Study the given information carefully to answer the given question.

   Seven boxes-A, B, C, D, E, F and G are kept one above the other, but not necessarily in the same order. Each box contains different items-Shoes, Papers, Bands, Medicines, Ribbons, Creams and Phones, but not necessarily in the same order.

    Only three boxes are kept between D and G. The Ribbon box is kept immediately above G. Only one box is kept between the Ribbon box and A. The Ribbon box is not the second from the bottom of the stack. Only one box is kept between E and A. E is kept above A. The medicine box is kept immediately above E Only three boxes are kept between the Medicine box and the Shoe box. The Paper box is immediately above the Phone box is immediately above the Phone box. G is not the Paper box. F is kept immediately below the Cream box. Only one box is kept between B and the Cream box.

20. Four of the following five are alike in a certain way and hence form a group. Which of the following does not belong to the group?

(A)  G-Creams

(B)  E-Bands

(C)  C-Shoes

(D)  D-Papers

(E)  A-Bands

Answer: (C)

21. Which of the following boxes contains bands?

(A)  D

(B)  C

(C)  G

(D)  A

(E)  Other than those given as options

Answer: (C)

22. What is the position of F in the given stack of boxes?

(A)  Second from the top

(B)  Third from the bottom

(C)  First from the top

(D)  Fifth from the bottom

(E)  Fourth from the top

Answer: (B)

23. Which of the following boxes is kept immediately below G?

(A)  A

(B)  B

(C)  C

(D)  The Shoe box

(E)  The Paper box

Answer: (A)

24. How many boxes are kept between B and the Ribbon box?

(A)  Two

(B)  One

(C)  None

(D)  Three

(E)

Answer: (D)

Directions – (Q. 25 to 29) Study the following information to answer the given question.

    Eight friends, C, D, E, F, L, M, N and O are seated in a straight line, but not necessarily in the same order. Some of them are facing north while some face south.

   Only three people sit to the right of M. E sits second to the left of M.

    F sits third to the right of O. O is not an immediate neighbour of M. O does not sit at any of the extreme ends of the line.

    Both the immediate neighbours of O face south.

    D sits second to the right of N.

    As many people sit between M and D as between M and L.

    Immediate neighbours of F face opposite direction (i.e., if one person faces north then the other person faces south and vice-versa.)

25. Which of the following is true, based on the given arrangement?

(A)  D faces north

(B)  Only three people face south

(C)  L sits at one of the extreme ends of the line

(D)  O and E face the same directions

(E)  None of the given options is true

Answer: (D)

26. How many people sit to the left of O?

(A)  Three

(B)  More than four

(C)  One

(D)  Four

(E)  Two

Answer: (C)

27. Who amongst the following faces north?

(A)  E

(B)  M

(C)  F

(D)  N

(E)  D

Answer: (A)

28. Who amongst the following sits second to the left of L?

(A)  O

(B)  F

(C)  D

(D)  No one as less than two people sit to the left of L

(E)  N

Answer: (D)

29. Who among the following represent the persons sitting at extreme ends of the line?

(A)  D, N,

(B)  C, D

(C)  L, N

(D)  D, L

(E)  C, N

Answer: (B)

Directions- (Q. 30 to 35) Study the following information and answer the given question.

    Seven people, namely A, B, C, D, E, F and G teach seven different subjects namely, Mathematics, English, Chemistry, History, Accountancy, Physics and Biology. Each of them works in either of the three institutes viz., Paramount, Brilliant and Embibe with atleast two of them in a institute.

(Note : Of the information given is necessarily in the same order.)

   G teaches in Brilliant with the one who teaches Accountancy. The one who teaches Biology works in an institute with only the one who teaches Chemistry. B teaches English. B does not work with G. D works with F but not in Embibe. Neither D nor F teaches Accountancy. F does not teach Chemistry. C works with only the one who teaches Mathematics. E works with the one who teaches History.

30. Who amongst the following teaches Physics?

(A)  F

(B)  D

(C)  C

(D)  E

(E)  G

Answer: (D)

31. Which of the following combinations represents the institute in which C works and the subject he teaches?

(A)  Brilliant-Accountancy

(B)  Embibe-Accountancy

(C)  Paramount-Biology

(D)  Briliant-History

(E)  Paramount-Chemistry

Answer: (A)

32. Which of the following subjects does A teach?

(A)  Biology

(B)  Chemistry

(C)  Mathematics

(D)  Accountancy

(E)  History

Answer: (E)

33. Which of the following statements is true?

(A)  D teaches Biology

(B)  None of the given statements is true

(C)  Only two people teach in brilliant

(D)  Both E and G work in the same institute

(E)  A teaches Mathematics

Answer: (C)

34. Which of the following combination represents the combination of people working in Paramount?

(A)  D and the one who teaches English

(B)  E, A

(C)  G, C, E

(D)  F and the one who teaches Chemistry

(E)  A, B

Answer: (D)

35. Four of the following five are alike in a certain way based on the given arrangement and hence form a group. Which of the following does not belong to that group?

(A)  BE

(B)  GC

(C)  EA

(D)  FG

(E)  AB

Answer: (D)

Associate Banks of SBI Clerical Cadre Recruitment 2014-15 Online Examination Held on February 15, 2015 Question Paper With Answer Key

Associate Banks of SBI Clerical Cadre Recruitment 2014-15 Online Examination Held on February 15, 2015
Associate Banks of SBI Clerical Cadre Recruitment 2014-15 Online Examination Held on February 15, 2015 Question Paper With Answer Key

Associate Banks of SBI Clerical Cadre Recruitment 2014-15 Online Examination Held on February 15, 2015

Part A Reasoning

Directions (Q. Nos. 1-5) Study the given information and answer the given questions.

In a certain code language

          ‘dress code for meeting’ is written as ‘dk pd jn te’

          ‘ wear black formal dress’ is written as ‘pd ro ld le’

          ‘formal meeting this weekend’ is written as ‘yi te le vr’

          ‘black code this weekend’ is written as ‘jn vr ld yi’

          (All the codes are two-letter codes)

1. In t he given code language, what does ‘le’ stand for?

(a)  this

(b)  formal

(c)  dress

(d)  black

(e)  meeting

Answer: (b)

2. In the given code language, what s the code for ‘dress’?

(a)  jn

(b)  ro

(c)  ld

(d)  pd

(e)  td

Answer: (d)

3. What does ‘ld’ stand for in the given code language?

(a)  meeting

(b)  weekend

(c)  formal

(d)  Other than those given as options

(e)  for

Answer: (d)

4. Which of the following possibly means ‘security code’ in the given code language?

(a)  ux vr

(b)  vr tc

(c)  pd ux

(d)  jn pd

(e)  jn ux

Answer: (e)

5. What is the code for ‘weekend’ in the given code language?

(a)  Either yi or vr

(b)  le

(c)  te

(d)  jn

(e)  ld

Answer: (a)

6. If it is possible to make only one meaningful word with the first, fourth, sixth and eighth letters of the word SENTENCE, which could be the second letter of the word from the right end? If more than one such word can be formed given ‘X’ as the answer. If no such word can be formed given ‘Z’ as your answer.

(a)  N

(b)  T

(c)  Z

(d)  X

(e)  S

Answer: (d)

Directions (Q. Nos. 7-11) In the following questions, two statements followed by two conclusions numbered I and II have been given. You have to take the given statements to be true even if they seems to be at variance from commonly known facts and then decide which of the given conclusions logically follows from the given statements disregarding commonly known facts.

7. Statements All drums are flutes.

All guitars are drums.

Conclusions

I. All guitars are flutes. II. All drums are guitars.

(a)  Both conclusions I and II are true.

(b)  Only conclusion I is true.

(c)  Neither conclusion I nor II is true.

(d)  Only conclusion II is true

(e)  Either conclusion I or II is true.

Answer: (b)

8. Statements Some speakers are guests.

No guest is a dignitary.

Conclusions

I. No dignitary is a speaker.

II. Some guests are definitely not speakers.

(a)  Only conclusion II is true

(b)  Both conclusions I and II are true.

(c)  Either conclusion I or II is true.

(d)  Neither conclusion I nor II is true.

(e)  Only conclusion I is true.

Answer: (a)

9. Statements All spades are tool.

No spade is a vessel.

Conclusions

I. At least some tools are vessel.

II. No tool is a vessel.

(a)  Either conclusion I or II true.

(b)  Neither conclusion I nor II is true.

(c)  Only conclusion I is true.

(d)  Both conclusions I and II are true.

(e)  Only conclusion II is true.

Answer: (a)

10. Statements All users are senders.

Some users are machines.

Conclusions

I. Some senders are machines.

II. All senders are machines.

(a)  Only conclusion I is true.

(b)  Only conclusion II is true.

(c)  Either conclusion I or II is true.

(d)  Neither conclusion I nor II is true.

(e)  Both conclusions I and II are true.

Answer: (a)

11. Statements Some cooks are employees.

Some employees are workers.

Conclusions

I. All cooks are workers. II. No cook is a worker.

(a)  Either conclusion I or II is true.

(b)  Neither conclusion I nor II is true.

(c)  Only conclusion I is true.

(d)  Both conclusions I and II are true.

(e)  Only conclusion II is true.

Answer: (b)

12. The positions of how many digits will remain the same if the digits in the number 297345618 are rearranged in the descending order within the number, from left to right?

(a)  None

(b)  Two

(c)  More than three

(d)  Three

(e)  One

Answer: (e)

Directions (Q. Nos. 13-17) In these questions, relationship between different elements is shown in the statements. The statements are followed by conclusions. Study the conclusions based on the given statement and select the appropriate answer.

13. Statement S = A ≤ P > R ≤ T < U

Conclusion   I. U > A      II. S ≤ T

(a)  Both conclusions I and II are true.

(b)  Only conclusion I is true.

(c)  Only conclusion II is true.

(d)  Neither conclusion I nor II is true.

(e)  Either conclusion I or II is true.

Answer: (d)

14. Statements P ≥Q < R = M, R ≥ F

Conclusion I. M ≥ F        II. F > P

(a)  Both conclusions I and II are true.

(b)  Only conclusion I is true.

(c)  Neither conclusion I nor II is true.

(d)  Either conclusion I or II is true.

(e)  Only conclusion II is true.

Answer: (b)

15. Statements P ≥ Q < R = M, R ≥ F

Conclusions I. F < Q        II. P ≥ M

(a)  Only conclusion I is true.

(b)  Both conclusions I and II are true.

(c)  Neither conclusion I nor II is true.

(d)  Either conclusion I or II is true.

(e)  Only conclusion II  is true.

Answer: (c)

16. Statement T ≥ O = I ≥ L = D

Conclusions I. D < T       II. D = T

(a)  Both conclusions I and II are true.

(b)  Neither conclusion I nor II is true.

(c)  Only conclusion I is true.

(d)  Only conclusion II is true.

(e)  Either conclusion I or II is true.

Answer: (c)

17. Statement J < C = T ≥ U ≥ R = N

Conclusions I. N < C       II. U > J

(a)  Both conclusions I and II are true.

(b)  Neither conclusion I nor II is true.

(c)  Only conclusion I  is true.

(d)  Only conclusion II is true.

(e)  Either conclusion I or II is true.

Answer: (b)

18. How many such pairs of letters are there in the word CLAYED, each of which has as many letters, between them in the word (in both forward and backward directions) as they have between them in the English alphabetical series?

(a)  None

(b)  One

(c)  Two

(d)  More than three

(e)  Three

Answer: (e)

Directions (Q. Nos. 19-24) Study the following information carefully and answer the questions.

C, D, E, F, G, H, I and J are seated in the straight line facing North, but not necessarily in the same order. H sits fifth to the left of D. D does not sits at any of the extreme ends of the line. Only two people sit between H and I. E sits third to the right of J. J is not an immediate neighbour of H. C is an immediate neighbour of G. G is not an immediate neighbour of I.

19. Which of the following represents persons seated at the two extreme ends of the line?

(a)  H, E

(b)  I, F

(c)  C, E

(d)  G, J

(e)  F, G

Answer: (a)

20. Which of the following is true with respect to F as per the given arrangement?

(a)  Only one person sits between F and J.

(b)  Both I and D are immediate neighbours of F.

(c)  None of the given options is true.

(d)  Only two people sit to the right of F.

(e)  F sits second to the left of E.

Answer: (a)

21. If all the given people are made to sit in alphabetical order from left to right, the positions of how many of them will remain unchanged?

(a)  Two

(b)  Three

(c)  Four

(d)  None

(e)  One

Answer: (d)

22. What is the position of J with respect to H?

(a)  Fourth to the right

(b)  Third to the right

(c)  Second to the right

(d)  Second to the left

(e)  Fifth to the right

Answer: (a)

23. How many persons are seated between G and F?

(a)  One

(b)  Five

(c)  Four

(d)  Three

(e)  Two

Answer: (c)

24. Who amongst the following sits exactly in the middle of the person who sits fourth from the left and third from the right?

(a)  D

(b)  G

(c)  F

(d)  I

(e)  J

Answer: (e)

Directions (Q. Nos. 25-27)The following questions are based on the given five three-letter words :

ACT    SHY  EON  ACE  WEB

25. If the letter ‘F’ is added before each of the given words, how many of them will form meaningful English words?

(a)  One

(b)  More than three

(c)  None

(d)  Two

(e)  Three

Answer: (d)

26. If the first and second letters of each of the given words are interchanged, then which of the following will become a meaningful English word?

(a)  EON

(b)  ACE

(c)  SHY

(d)  ACT

(e)  WEB

Answer: (d)

27. If the first and third letters of each of the given words is changed to the next letter in the English alphabetical series, how many of them will have at least two vowels (same or different vowels)?

(a)  None

(b)  Two

(c)  Three

(d)  One

(e)  Four

Answer: (d)

Directions (Q. Nos. 28-33) Study the given information and answer the given questions.

Eight people (M, N, O, P, W, X, Y and Z) are sitting around a circular table facing the Centre, with equal distances between each other but not necessarily in the same order. M sits second to the left of O. P is an immediate neighbour of O. Only three people sits between O and Y. X sits to the immediate right of W. W is not an immediate neighbour of O. Z is neither an immediate neighbour of W nor Y.

28. What will come in the place of question mark in the following series based on the given arrangement?

MZ      PW    YN    ZO     ?

(a)  WP

(b)  XN

(c)  WX

(d)  XY

(e)  WY

Answer: (c)

29. Which of the following pairs represents the immediate neighbours of M ?

(a)  X, W

(b)  P, Y

(c)  Y, Z

(d)  Z, W

(e)  Z, N

Answer: (e)

30. Which of the following statements is true as per the given arrangement?

(a)  X is an immediate neighbour of Y.

(b)  Z sits second to the right of W.

(c)  Y sits to the immediate left of M.

(d)  P sits second to the right of X.

(e)  None of the given options is true.

Answer: (a)

31. How many people sit between M and W as per the given arrangement?

(a)  None

(b)  Four

(c)  Three

(d)  Two

(e)  One

Answer: (c)

32. Who amongst the following sits third to the left of P?

(a)  Y

(b)  N

(c)  O

(d)  Z

(e)  M

Answer: (e)

33. Four of the following five are alike in a certain way and so form a group. Which of the following does not belong to the group?

(a)  O, M

(b)  W, O

(c)  Y, X

(d)  Z, N

(e)  N, X

Answer: (c)

Directions (Q. Nos. 34-38) Study the following arrangement carefully and answer the questions :

3 9 2 4 7 5 9 2 8 1 4 9 5 3 1 6 5 7 3 4 2 9 8 1 3 6 2 8 1 7 5 4 5

34. How many such 2s are there in the given arrangement, each of which is immediately followed by a perfect square?

(a)  More than three

(b)  Three

(c)  One

(d)  Two

(e)  None

Answer: (d)

35. If all the even digits are deleted from the given arrangement, which of the following will be tenth from the right end of the arrangement?

(a)  3

(b)  1

(c)  5

(d)  7

(e)  9

Answer: (c)

36. How many such 4s are there in the given arrangement, each of which is immediately followed by a digit which has a numerical value of more than four?

(a)  More than three

(b)  One

(c)  None

(d)  Two

(e)  Three

Answer: (e)

37. Which of the following is seventh to the left of the seventeenth digit from the left end of the given arrangement?

(a)  5

(b)  9

(c)  4

(d)  8

(e)  1

Answer: (e)

38. How many such 3s there in the given arrangement, each of which immediately preceded by an odd digit and also immediately followed by an even digit?

(a)  Three

(b)  None

(c)  More than three

(d)  Two

(e)  One

Answer: (d)

39. In a certain code, the word DOUBT is written as ‘$#35&’ and the word BONES is written as 5#@67’. How is the word STONE written in this code?

(a)  7&#@6

(b)  7@53#

(c)  7&53#

(d)  $673@

(e)  &753#

Answer: (a)

40. Among five friends A, B, C, D and E each running at a different speed in a race. E ran faster than only two friends. A ran faster than D but slower than B. B did not run the fastest. Who ran the second slowest?

(a)  A

(b)  B

(c)  C

(d)  Cannot be determined

(e)  D

Answer: (a)

Part B English

Directions (Q. Nos. 41-45) The sentences have a blank indicating that something has been omitted. Choose the word that bests suits the meaning of the sentence as a whole.

41. Ideally you should set aside funds to help you …. over situations such as a sudden medical emergency or unexpected expenses.

(a)  come

(b)  turn

(c)  reach

(d)  pass

(e)  tide

Answer: (e)

42. Organizations should implement some practices that t he army follows …… younger staff with important conclusions.

(a)  experiencing

(b)  nominating

(c)  responsible

(d)  entrusting

(e)  supportive

Answer: (d)

43. …… from teaching young students about entrepreneurship, we also provide funds to start and sustain their business for the short time.

(a)  While

(b)  Whether

(c)  Apart

(d)  Further

(e)  Perhaps

Answer: (c)

44. Cost of fuel is the second most crucial ……… in a person’s decision to buy a car.

(a)  event

(b)  advice

(c)  factor

(d)  opportunity

(e)  aspects

Answer: (e)

45. The green revolution which started in the 1960s helps to reduce food scarcity and ………. improved the health of the economy.

(a)  still

(b)  rapidly

(c)  expected

(d)  growing

(e)  extremely

Answer: (b)

Directions (Q. Nos. 46-50) In the following question sentence with four bold words were given. One of them from four words may be either wrongly spelt or inappropriate in the context of the sentence. Find out the word which is wrongly spelt or inappropriate. If any, that word will be your answer. If all words given in the bold are correctly spelt and also appropriate in the context of the sentence, then mark ‘All correct’ as your answer.

46. According to the management the bank is likely to perform better in the next

(a)  management

(b)  likely

(c)  perform

(d)  quarter

(e)  All correct

Answer: (e)

47. It is difficult to gain access to the hospital because there is no permanent

(a)  difficult

(b)  gain

(c)  access

(d)  permanent

(e)  All correct

Answer: (c)

48. One of the most important tasks of the morden government is to run as efficient welfare system.

(a)  most

(b)  tasks

(c)  morden

(d)  efficient

(e)  All correct

Answer: (c)

49. Even experienced bankers were surprised by RDFs decision to cut rates prior to the budget.

(a)  Even

(b)  experienced

(c)  surprised

(d)  prior

(e)  All correct

Answer: (b)

50. The new generation is confident, tech-savvy and is not fearful of trying new

(a)  generation

(b)  fearful

(c)  trying

(d)  things

(e)  All correct

Answer: (c)

Directions (Q. Nos. 51-60) Read the following passage carefully and answer the given questions. Certain words/phrases have been given in bold to help you locate them while answering some of the questions.

    A group of employees was working in a software company, it was a team of thirty employees. They was young, energetic and dynamic team with keen enthusiasm and a desire to learn and grow. The management decided to teach the employees how to find real solutions to problems. The team was called to play a game in banquet hall. The group was quite surprised that they were called to play a game. Everyone reached the venue with various thoughts on their minds. As they entered the hall, they found the hall decorated beautifully with colourful decorated papers and ballons all over the place. It was more like a children’s  party area than a corporate meeting hall.

Everyone was surprised and gazed at each other. Also there was a huge box of balloons was placed at the centre of the hall. The team leader asked everyone to pick a balloon from the box and asked them to blow it up.

Everyone picked up the balloon and blow it up. Then the team asked them to write their names on their balloons carefully so that the balloons did not burst. Everyone tried to write their names on their balloons, but not everyone was successful. A few balloons burst due to pressure and the people whose balloons were burst, were given another chance to  pick a balloon. Those, who failed to write their names even after the second chance, were asked to leave the game.

After the second chance, twenty-five employees qualified for the next level. All the balloons were collected and then put into a room. Then team leader asked the employee to go to the room and pick the same balloon that had his/her name on it. Also he told them that no balloon should burst and warned them to be very careful. All the employees reached the room, where the balloons carrying their names were thrown here and there. They were searching for their respective balloons. While they were in a rush to find the respective balloons. They tried not to burst the balloons. Fifteen minutes passed away and no one was able to find the balloons carrying their own name.

The team was told that the second level of the game was over. Now as the time for third and final level. The employees were asked to pick any balloon in the room and given it to the person named on the balloon. Within a couple of minutes alb balloons reached the hands of the respective employees and everyone reached the hall. The team leader then said “This is called as the new solution to problem. Everyone is frantically haunting for solutions to problem without understanding the ideal way. Many times sharing and helping others gives you real solutions to all problems.”

51. Which of the following cannot be said about the employees?

(A)  At least one employee found the balloon when they were asked to pick up a balloon with their name on it.

(B) The participated whole-heartedly in the task.

(C) They all failed to follow the instructions given by the team leader in the end.

(a)  Only A

(b)  A and B

(c)  All of these

(d)  Only B

(e)  A and C

Answer: (b)

52. Why was everyone surprised reaching on their corporate meeting hall?

(a)  Their team leader was dressed as a kid.

(b)  Their seniors were missing from the hall.

(c)  The hall was equipped with the latest and high technological gadgets.

(d)  The hall was too small to accommodate all of them.

(e)  Other than those given as options

Answer: (e)

53. Which of the following can be said about the management’s idea of trying to teach their employees something?

(A) The idea did not indeed to teach anything for the people who did not finish the task.

(B) The idea was ill received by all.

(C) The idea was creative.

(a)  Only A

(b)  B and C

(c)  Only C

(d)  All of these

(e)  A and B

Answer: (b)

54. Which one of the following aspects come across very strongly in the story?

(a)  One should not worry about goals.

(b)  One should be honest and sincere regarding one’s work.

(c)  One should be helpful towards others regardless what the end results in

(d)  One should look beyond personal goals to achieve success in a task.

(e)  One should share one’s problem with others to feel better.

Answer: (e)

55. Choose the word which is most opposite in meaning to the word ‘Qualified’ given in the bold as used in the passage.

(a)  Illiterate

(b)  Ignorant

(c)  Talled

(d)  Pushed

(e)  Silly

Answer: (d)

56. Choose the word which is most similar in meaning to the word ‘Warned’ given in bold as used in the passage.

(a)  Threatened

(b)  Cautioned

(c)  Pressurized

(d)  Intermediated

(e)  Bulfied

Answer: (b)

57. Choose the word which is most similar in meaning to the word ‘Dynamic’ given in the bold as used in the passage.

(a)  Vibrant

(b)  Colourful

(c)  Explosive

(d)  Different

(e)  Noisy

Answer: (a)

58. Which of the following is true according to the story?

(a)  The task was successful only after the employees were shown the right way to go about it.

(b)  The only way to make a task successful is by helping others, otherwise the task fails.

(c)  The employees who were asked to leave the game were disinterested in it.

(d)  There were more than four levels in the task.

(e)  All the given statements are true.

Answer: (a)

59. Choose the word which is most opposite in meaning to the word ‘Frantically’ given in bold as used in the passage.

(a)  Nervously

(b)  Secretively

(c)  Crazily

(d)  Calmly

(e)  Minutely

Answer: (d)

60. Which of the following can be the most appropriate title for the story?

(a)  Sharing and caring

(b)  The game of balloons

(c)  The company with a team leader

(d)  A friend in need is a friend indeed

(e)  The twenty-five employees

Answer: (a)

Directions (Q. Nos. 61-65) Rearrange the given sex sentences/group of sentences A, B, C, D, E and F in a proper sequence so as to form a meaningful  paragraph and then answer the given questions.

(A) Beena grow fertilizers by the day and was charming and polite but as the goddess predicted she has to have a new outfit everyday and for a while the king and queen were happy to let their way but soon realized that this had to stop.

(B) They longed for the child and prayed and prayed till finally one day their prayers were but the goddess warned,” you will soon have a little girl and though she will be a loving child. She will love new clothes too much and will be cursed if she covert another’s clothes.

(C) They tried to get mend her ways but in vain and one day when Beena saw a beautiful girl simply dressed in a cotton saree and noticed how people were admiring the simply dressed girl she demanded the dress.

(D) The kingdom was at peace, subjects were happy and there was a bumper crop but  the King and Queen was very sad.

(E) As foretold, Beena turned into an onion. A plant with many layers to symbolize the many dresses she had.

(F) The King and Queen were ready to accept this flow and when the Queen gave birth to a baby girl the entire kingdom replaced.

61. Which of the following should be FIFTH sentence after rearrangement?

(a)  A

(b)  C

(c)  D

(d)  E

(e)  F

Answer: (b)

62. Which of the following should be SIXTH (LAST) sentence after rearrangement?

(a)  A

(b)  B

(c)  C

(d)  D

(e)  E

Answer: (e)

63. Which of the following should be FIRST sentence after rearrangement?

(a)  A

(b)  B

(c)  C

(d)  D

(e)  F

Answer: (d)

64. Which of the following should be SECOND sentence after rearrangement?

(a)  A

(b)  B

(c)  C

(d)  D

(e)  F

Answer: (b)

65. Which of the following should be THIRD sentence after rearrangement?

(a)  A

(b)  B

(c)  D

(d)  E

(e)  F

Answer: (e)

Directions (Q. Nos. 66-70) Read the following sentences to find out whether there is any grammatical mistake/error in them. The error, if any, will be in one part of the sentence. Mark the  part with the error as your answer. If there is no error mark ‘no error’ as your answer. (Ignore the errors of punctuations if any.)

66. One night, friddle the fox was/prowling around/ an actor’s house when he/ found an open window.

(a)  One night, friddle the fox was

(b)  prowling around

(c)  an actor’s house when  he

(d)  found and open window

(e)  No error

Answer: (e)

67. Millie was/ a mouse who had/ a cousin named Tora/ living from the city.

(a)  Millie was

(b)  a mouse who had

(c)  a cousin named Tora

(d)  living from the city.

(e)  No error

Answer: (d)

68. In a jungle, there lived/ a lion who felt ashamed/ of himself because/ he was scared on chickens.

(a)  In a jungle, there lived

(b)  a lion who felt ashamed

(c)  of himself because

(d)  he was scared on chickens

(e)  No error

Answer: (d)

69. The big bad wolf gobbled/ up the little lamb so fast/ that a bone was stick/ in his throat.

(a)  The big bad wolf gobbled

(b)  up the little lamb so fast

(c)  that a bone was stick

(d)  in his throat.

(e)  No error

Answer: (c)

70. Hares are rather/ noise animals who/ love to bear/ themselves talking.

(a)  hares are rather

(b)  noise animals who

(c)  love to bear

(d)  themselves talking

(e)  No error

Answer: (b)

Directions (Q. Nos. 71-80) In the given passage there are blanks, each of which are numbered. Against each five words are suggested, one of which fits the blank appropriately. Find out the appropriate word in each case.

Once in a town of Shimpola, there lived a rich and cunning money lender called Vidushan. He (71) money to poor people and (72) them to give their lands. The trouble people went to Subhadhar who (73) to visit their town soon.

One morning, a jeweler visted Vidushan with some expensive looking jewellery. When Vidushan asked for its price, the jeweler said it was worth a handful grain. he also said if you are (74) to give me a handful grain, give me a thousand gold coins and return to poor people’s land as well. Greedy Vidushan was very (75). He gave the jeweler a handful of grains. The jeweler (76) to take it and took him to court. There he told the judge, ‘My Lord, I had asked him for a single grain that was the (77) of a full hand and he gave me a handful of grains. As per the (78) he should pay me a thousand of gold coins and return the poor people’s land to them.”

The Judge (79) and adjudged the case in his (80). The jeweler was actually Subhadhar and he had helped the poor to get their lands back.

71.

(a)  lent

(b)  borrowed

(c)  grant

(d)  presented

(e)  showed

Answer: (a)

72.

(a)  empowered

(b)  forced

(c)  punished

(d)  prevent

(e)  destroy

Answer: (b)

73.

(a)  said

(b)  assuring

(c)  promise

(d)  guarantee

(e)  vouched

Answer: (e)

74.

(a)  not

(b)  powerless

(c)  unqualified

(d)  unable

(e)  there

Answer: (d)

75.

(a)  contents

(b)  delightful

(c)  blessed

(d)  happy

(e)  gladly

Answer: (d)

76. 

(a)  accepted

(b)  denies

(c)  loved

(d)  wanted

(e)  refused

Answer: (e)

77.

(a)  lengthy

(b)  growth

(c)  extent

(d)  size

(e)  shapes

Answer: (d)

78.

(a)  conditions

(b)  demands

(c)  position

(d)  state

(e)  will

Answer: (b)

79.

(a)  yes

(b)  agreed

(c)  supporting

(d)  chose

(e)  ordered

Answer: (b)

80.

(a)  honour

(b)  side

(c)  favour

(d)  worth

(e)  service

Answer: (c)

Part C Quantitative Aptitude

81. If one-third of one-seventh of a number is 15, then two-fifth of that number is

(a)  158

(b)  165

(c)  142

(d)  136

(e)  126

Answer: (e)

82. The simple interest obtained when a sum of money is invested for 4 years at 18% per annum is Rs 427 more than the simple interest obtained if the same sum of money is invested for 2 years at 22% per annum. What is the amount obtained when the same sum of money is invested for 4 years at 18% per annum?

(a)  Rs 2130

(b)  Rs 2623

(c)  Rs 1096

(d)  Rs 1854

(e)  Rs 2475

Answer: (b)

83. Train A, whose length is 328 m, can cross a 354 m long platform in 11 seconds. Train B can cross the same platform in 12 seconds. If the speed of Train B is 7/8th of the speed of Train A, what is the length of Train B?

(a)  321 m

(b)  303 m

(c)  297 m

(d)  273 m

(e)  309 m

Answer: (c)

84. The ratio of number of males and number of females in village X is 11:7 respectively. If in village Y, the number of males is 20% more than the number of males in village X and the number of females is 12% less than those in village X, then what will be the respective ratio of males to females in village Y?

(a)  15 : 11

(b)  17 : 5

(c)  17 : 11

(d)  23 : 9

(e)  15 : 7

Answer: (e)

85. The areas of two circular fields are in the ratio of 25 : 64 respectively. If the radius of the latter is 16 m then what is the radius of the former?

(a)  14 m

(b)  10 m

(c)  6 m

(d)  18 m

(e)  8 m

Answer: (b)

86. 16 time X is equal to 5 times Y. If 8 is subtracted from Y it is 10 more than 2 times X. What is the sum of X and Y?

(a)  78

(b)  39

(c)  48

(d)  92

(e)  63

Answer: (e)

Directions (Q. Nos. 87-91) Study the table and answer the given questions.

87. If the total number of pages printed by all the given printers together on Saturday was 18% more than the total number of pages printed by all the given printers together on Thursday, what was the total number of pages printed by all the given printers together on Saturday?

(a)  1379

(b)  1299

(c)  1357

(d)  1289

(e)  1331

Answer: (c)

88. What is the respective ratio between total number of pages printed by printer O on Tuesday and Wednesday together and total number of pages printed by printer Q on Monday and Tuesday together?

(a)  17 : 21

(b)  17 : 19

(c)  19: 21

(d)  17 : 23

(e)  19 : 23

Answer: (a)

89. What is the sum of 58% of the total number of pages printed by printers M and O together on Monday and 65% of the total number of pages printed by printers N and Q together on Friday?

(a)  610

(b)  655

(c)  575

(d)  625

(e)  640

Answer: (d)

90. Number of pages printed by printer N on Tuesday is what percent of the number of pages printed by printer M on Friday?

(a) 

(b) 

(c) 

(d) 

(e) 

Answer: (a)

91. What is the average number of pages printed by printer P during all the given days together?

(a)  214

(b)  204

(c)  208

(d)  218

(e)  198

Answer: (c)

92. Glenn labelled the price of an article in such a way so to earn 25% profit. However, while selling he offered 6% discount on the labelled price. If the sold it for Rs 10340, what was the cost price of the article?

(a)  Rs 9200

(b)  Rs 8000

(c)  Rs 8800

(d)  Rs 8600

(e)  Rs 8400

Answer: (c)

93. In what ratio Martina should mix two varieties of juice worth Rs 52 per litre and Rs 74 per litre so that by selling the mixture at Rs 71.28 per litre she can earn a profit of 8%?

(a)  6 : 7

(b)  3 : 7

(c)  5 : 8

(d)  4 : 7

(e)  3 : 8

Answer: (d)

94. The difference between length and breadth of a rectangle is 17 m and the perimeter is 106 m. Find the area of rectangle?

(a)  716 m2

(b)  660 m2

(c)  630 m2

(d)  740 m2

(e)  524 m2

Answer: (c)

95. Present ages of three persons A, B, C are in the ratio of 6 : 7 : 9 respectively. After 5 years, sum of their ages will be 81 years. Find the present age of B?

(a)  21 years

(b)  28 years

(c)  35 years

(d)  14 years

(e)  42 years

Answer: (a)

96. Two pipes can fill a tank in 10 hours and 16 hours respectively. A third pipe can empty the tank in 40 hours. If all the three pipes are opened and function simultaneously then in how much time the tank will be full?

(a) 

(b) 

(c) 

(d) 

(e) 

Answer: (d)

97. The average monthly income of A and B is Rs 7760. The average monthly income of B and C is Rs 10990 and that of C and A is Rs 9070. What is the annual income of B?

(a)  Rs 120240

(b)  Rs 124480

(c)  Rs 112360

(d)  Rs 128120

(e)  Rs 116160

Answer: (e)

98. Scheme A offers compound interest (compounded annually) at a certain rate of interest (percent per annum) When a sum was invested in the scheme it amounted to Rs 14112 after 2 years and Rs 16934.40 after 3 years. What was the sum of money invested?

(a)  Rs 9000

(b)  Rs 10200

(c)  Rs 8800

(d)  Rs 9400

(e)  Rs 9800

Answer: (e)

99. Sharma distributed an amount of money among his wife, one son and one daughter and kept some money for himself. 12% of the total amount he gave to his wife and kept 22% for himself. Out of the remaining amount, he gave to his son and daughter in the ratio of 7 : 8 respectively. If the daughter got Rs 3806 more than the son, what was the total amount that Mrs. Sharma had?

(a)  Rs 60000

(b)  Rs 78650

(c)  Rs 86500

(d)  Rs 80500

(e)  Rs 72400

Answer: (c)

Directions (Q. Nos. 100-119) What will come in  place of question marks in the given questions?

100. 

(a)  2

(b)  5

(c)  3

(d)  6

(e)  4

Answer: (a)

101. (349 + 583) / 23.3 + 428 = ?/9

(a)  4068

(b)  3870

(c)  4203

(d)  4158

(e)  4212

Answer: (e)

102. 56 – 742/53/2 = ?2

(a)  3

(b)  4

(c)  6

(d)  7

(e)  2

Answer: (d)

103. (1496 – 392) / 23 × 15 = 213 + ?

(a)  477

(b)  437

(c)  539

(d)  507

(e)  517

Answer: (d)

104. 

(a)  14

(b)  9

(c)  6

(d)  12

(e)  20

Answer: (b)

105. 5136/(523 + 333) of 3/4 + 459 = ?

(a)  520

(b)  541

(c)  513

(d)  493

(e)  467

Answer: (e)

106. 642 – 362 = ? × 25

(a)  118

(b)  212

(c)  112

(d)  134

(e)  106

Answer: (c)

107. 2/3 – 5/7 of 21/23 of 46/51 = ? of 8/85

(a)  1/6

(b)  5/6

(c)  5/7

(d)  2/17

(e)  1/3

Answer: (b)

108. 43 × 26 × 82/3 = 2?

(a)  10

(b)  16

(c)  14

(d)  8

(e)  12

Answer: (c)

109. 

(a) 

(b) 

(c) 

(d) 

(e) 

Answer: (d)

110. 2560/32 + 23/45 of 855 = 7/11 of ? + 391

(a)  297

(b)  198

(c)  132

(d)  176

(e)  275

Answer: (b)

111. 156.25/12.5 × 1000/2500 = ?/28

(a)  147

(b)  140

(c)  112

(d)  168

(e)  196

Answer: (b)

112. 

(a)  51

(b)  59

(c)  53

(d)  42

(e)  45

Answer: (c)

113. 

(a)  129

(b)  139

(c)  121

(d)  153

(e)  119

Answer: (e)

114. 150% of 240 + 125% of 1684 = [? – 345] × 493

(a)  350

(b)  450

(c)  370

(d)  368

(e)  410

Answer: (a)

115. 319, 320, 347, 472, 815, ?

(a)  1322

(b)  1265

(c)  1435

(d)  1148

(e)  1544

Answer: (e)

116. 12, 6.5, 7.5, 12.75, 27.5, ?

(a)  65.5

(b)  58.25

(c)  71.25

(d)  55.75

(e)  52

Answer: (c)

117. 5, 6, 14,45, ?, 925

(a)  138

(b)  184

(c)  172

(d)  98

(e)  154

Answer: (b)

118. 112, 117, 127, 142, 162, ?

(a)  214

(b)  187

(c)  194

(d)  182

(e)  205

Answer: (b)

119. 4, 7, 20, 79, ? 2363

(a)  234

(b)  148

(c)  296

(d)  482

(e)  394

Answer: (e)

120. A, B and C started a business together. The respective ratio of investments of A and B was 3 : 5 and the respective ratio of investments of B and C was 10 : 13. If at the end of the year C received Rs 5876 as his share of annual profit, what was the total annual profit earned by all of them together?

(a)  Rs 13108

(b)  Rs 12756

(c)  Rs 13224

(d)  Rs 12984

(e)  Rs 12188

Answer: (a)

Part D Computer Knowledge & Marketing Awareness

121. …………. connects CPU to various internal components to expansion cards for graphics and sound.

(a)  RAM

(b)  Internal buses

(c)  BIOS

(d)  Chip set

(e)  ROM

Answer: (b)

122. As per SEBI regulation, every applicant shall pay a non-refundable application fee of …………. along with application for grant or renewal of certification of registration for investments advisors.

(a)  Rs 2500

(b)  Rs 15000

(c)  Rs 7500

(d)  Rs 5000

(e)  Rs 10000

Answer: (d)

123. In order to grow task, if a company takes its existing product/service to new markets, it is said to be deploying

(a)  Market penetration strategy

(b)  Market development strategy

(c)  Product development strategy

(d)  All those given as options

(e)  Diversification strategy

Answer: (b)

124. ……………. are used to measure both computer memory (RAM) and storage capacity of floppy disks, CD-ROM drives and hard drives.

(a)  Bytes

(b)  Bites

(c)  Octal numbers

(d)  Hexadecimal numbers

(e)  Binary numbers

Answer: (a)

125. Which of the following is not included in the four strategies to assign objective, strategy and budget to each SBU as per BCG growth share matrix?

(a)  Invest

(b)  Divest

(c)  Harvest

(d)  Hold

(e)  Bold

Answer: (e)

126. …………… is not the stage of product life-cycle in marketing.

(a)  Maturity

(b)  Growth

(c)  Introduction

(d)  Decline

(e)  Planning

Answer: (e)

127. The marketing concept for a cluster of complimentary products and services that are closely related in the minds of consumers but are spread across a diverse set of industries, is

(a)  Metamarket

(b)  Syncrhonised marketing

(c)  Vertical integration

(d)  Horizontal integration

(e)  Betamarket

Answer: (a)

128. ……………… includes boot firmware and power management.

(a)  CD-ROM

(b)  Internal buses

(c)  BIOS

(d)  Chip set

(e)  RAM

Answer: (c)

129. In case of open ended scheme, the mutual funds shall publish in a daily newspaper of all India circulation at least once in …………… the sale and repurchase priceoff units.

(a)  a week

(b)  three days

(c)  two days

(d)  a fortnight

(e)  a month

Answer: (a)

130. ……………. is not a basic element of marketing mix.

(a)  Promotion

(b)  Place

(c)  Price

(d)  Product

(e)  Publication

Answer: (e)

131. What is an on-screen display listing of available options or functions on a computer?

(a)  Document

(b)  View

(c)  Tool

(d)  Format

(e)  Menu

Answer: (e)

132. Which concept is practiced for the products that buyers normally do not think of buying, such as insurance?

(a)  Other than those given as options

(b)  Trading concept

(c)  Selling concept

(d)  Product concept

(e)  Production concept

Answer: (c)

133. Compact disk that can store approximately 650-800 MB of data or 74-80 minutes of music are

(a)  Zip disks

(b)  CD-ROM

(c)  Video cards

(d)  Pressing machines

(e)  Floppy diskettes

Answer: (b)

134. What do we call a network whose elements may be separated by some distance? It usually involves two or more networks and dedicated high speed telephone lines.

(a)  LAN

(b)  WAN

(c)  URL

(d)  Server

(e)  World wide web

Answer: (a)

135. Printing orientation that is wider that it is taller is

(a)  portrait concentration

(b)  icon

(c)  page set

(d)  landscape orientation

(e)  wide-angle

Answer: (d)

136. A collection of various programs that helps control your computer is called

(a)  System software

(b)  Application software

(c)  Microsoft Excel

(d)  Microsoft Word

(e)  Microsoft Outlook

Answer: (a)

137. Social marketing emerged during the year

(a)  1982

(b)  1972

(c)  1961

(d)  Other than those given as options

(e)  1951

Answer: (b)

138. Which of the following is a parameter for determining competitive position of company’s products?

(a)  Historical profit margins

(b)  Market growth rate

(c)  Market share rate

(d)  Overall market size

(e)  Number of brands in portfolio

Answer: (c)

139. …………… is the standard input device of most computers.

(a)  Scanner

(b)  Joystick

(c)  Speaker

(d)  Microphone

(e)  Keyboard

Answer: (e)

140. What is MS-Word?

(a)  It is a calculating tool.

(b)  It is planning tool.

(c)  It is a chart.

(d)  It is a networking tool.

(e)  It is a document typing tool.

Answer: (e)

141. …………. is considered as debit market securities.

(a)  Fixed deposit of  bank

(b)  Certificate of deposit

(c)  Shares stock

(d)  Preference stock

(e)  Mortgage loans

Answer: (b)

142. What is the temporary storage memory chips that form the computers primary work space? The contents are lost if power is disrupted.

(a)  Outputs

(b)  Windows

(c)  ROM

(d)  Hard drive memory

(e)  RAM

Answer: (e)

143. Computer software is

(a)  used only for output

(b)  a computer peripheral

(c)  used for input

(d)  a set of instructions

(e)  used only in operating systems

Answer: (d)

144. Which of the following does not come under ‘general insurance’?

(a)  Marine insurance

(b)  Fire insurance

(c)  Life insurance

(d)  Fidelity insurance

(e)  Health insurance

Answer: (c)

145. What do we call for the arrangement when two or more computers physically connected by cables to share information or hard ware?

(a)  URL

(b)  Network

(c)  Server

(d)  Internet

(e)  Modem

Answer: (b)

146. In growth-share matrix approach of Boston Consulting Group, ……… are businesses with weak market share in low-growth market generating low profit.

(a)  Stars

(b)  other than those given as options

(c)  Dogs

(d)  Questions marks

(e)  Cash cows

Answer: (c)

147. A word (or words) used in computing in a search to match a desired topic is called

(a)  ID name

(b)  Hint

(c)  Password

(d)  User

(e)  Keyword

Answer: (e)

148. What is the stored question about information in a database?

(a)  Query

(b)  Sort

(c)  Report

(d)  Record

(e)  Field

Answer: (a)

149. Merrill Lynch uses a symbol of bull to implify strength and leadership. This is an example of ………… in service marketing.

(a)  Visualization

(b)  Other than those given as options

(c)  Association

(d)  Documentation

(e)  Physical representation

Answer: (e)

150. Which of the following is a tax saving mutual fund scheme?

(a)  Gilt fund

(b)  ELSS

(c)  Debt funds

(d)  Gold funds

(e)  Balanced funds

Answer: (b)

151. Formatting a disk means

(a)  programming the disk so data on it can’t be erased

(b)  cleaning the disks from any dust contaminates

(c)  installing an operating system on it

(d)  setting up sections on the disk to store the files in

(e)  installing files on it

Answer: (d)

152. Keyboard shortcut for CUT command is

(a)  Ctrl + W

(b)  Ctrl + Y

(c)  Ctrl + Z

(d)  Ctrl + X

(e)  Ctrl + V

Answer: (d)

153. The main circuit board in the computer that connects the parts of the computer is the

(a)  Mother board

(b)  Father board

(c)  Match board

(d)  Hard drive

(e)  Special board

Answer: (a)

154. Dividing the market into segments and charging the same price for every one in a segment is known as

(a)  Steaming pricing

(b)  Cost-based pricing

(c)  Personal pricing

(d)  Group pricing

(e)  Individual pricing

Answer: (d)

155. What is the name of the device that links your computer with other computers and information services through telephone lines?

(a)  Modem

(b)  LAN

(c)  URL

(d)  WAN

(e)  Server

Answer: (a)

156. ………….. provides hard copy output on paper.

(a)  Mouse

(b)  Keyboard

(c)  LCD monitor

(d)  Scanner

(e)  Printer

Answer: (e)

157. What is the validity period for the certificate of registration issued by SEBI for investment advisors for mutual funds?

(a)  4 years

(b)  5 years

(c)  3 years

(d)  2 years

(e)  1 year

Answer: (b)

158. ………. concept holds that firms must strive to deliver value to customers in a way that maintains or improves both the consumers as well as society’s well being.

(a)  Product

(b)  Societal networking

(c)  Selling

(d)  Marketing

(e)  Equity

Answer: (b)

159. The financial contracts whose value are obtained from the values of underlying assets are

(a)  Bonds

(b)  Mortgages

(c)  Derivatives

(d)  Stocks

(e)  Commercial papers

Answer: (c)

160. When backed by an ability to pay, want for specific product becomes

(a)  need

(b)  satisfaction

(c)  Other than those given as options

(d)  demand

(e)  esteem

Answer: (d)

Part E General Knowledge

161. ‘Jim Corbett National Park’ is the oldest national park in India. It is situated in the Indian State of

(a)  Himachal Pradesh

(b)  Uttar Pradesh

(c)  Punjab

(d)  Uttarakhand

(e)  Arunachal Pradesh

Answer: (d)

162. An independent authority appointed by the RBI for resolution of complaints against deficiency in banking services, is known as

(a)  Chief Manager Complaints in each bank

(b)  Chief Vigilance Officer

(c)  No such authority appointed.

(d)  Banking Ombudsmen

(e)  Compliance Officer

Answer: (d)

163. The capital ‘Kingdom of Denmark’ is

(a)  Munich

(b)  Prague

(c)  Hamburg

(d)  Heisenburg

(e)  Copenhegan

Answer: (e)

164. In a bank’s balance sheet, which of the following is an asset?

(a)  Its  paid up capital

(b)  Its savings deposits

(c)  Its investment in government securities

(d)  Its accumulated reserve funds

(e)  Its current deposits

Answer: (a)

165. The United Nations Children Funds (UNICEF) is a United Nations programme that provides humanitarian and development assistance to children and mothers in developing countries. Its headquarters is in

(a)  Geneva (Switzerland)

(b)  Paris (France)

(c)  Vienna (Austria)

(d)  New York (USA)

(e)  Rome (Italy)

Answer: (d)

166. The book ‘Odyssey of My Life’ is the autobiography of

(a)  Shivraj Patil

(b)  RR Patil

(c)  Sandeep Patil

(d)  Smita Patil

(e)  Pratibha Patil

Answer: (b)

167. Union HRD Minister Smriti Irani laid the foundation stone of ‘Indian Institute of Management in the State of Andhra Pradesh. It is being set-up in

(a)  Tirupathi

(b)  Nellore

(c)  Vishakhapatnam

(d)  Guntur

(e)  Vijaywada

Answer: (c)

168. Space probe Beagle 2’ has been found on the surface of Mars, 11 years after it was thought that it was lost on Mars. It was originally launched in 2003 by

(a)  Soviet Space Programme

(b)  Israeli Space Agency

(c)  Japan Aerospace Exploration Agency

(d)  China National Space Administration

(e)  European Space Agency

Answer: (e)

169. In order to raise awareness, conserve and use water resources in an integrated manpower, the government of India observed “India Water Week” in the month of

(a)  December, 2014

(b)  February, 20156

(c)  October, 2014

(d)  January, 2015

(e)  November, 2014

Answer: (d)

170. The men’s final of 10th Savio Cup All India Basketball Tournament has recently been won by

(a)  Oil and Natural Gas Commission (ONGC)

(b)  Indian Overseas Bank (IOB)

(c)  Hindustan  Petroleum Corporation Limited (HPCL)

(d)  Indian Oil Corporation Limited (IOCL)

(e)  State Bank of India (SBI)

Answer: (a)

171. Deep Kumar Upadhyay has recently been named as its amabassador to India by the government of

(a)  Mauritius

(b)  Nepal

(c)  Bhutan

(d)  Fiji

(e)  Maldives

Answer: (b)

172. The ‘Indian Army Day’ is observed every year on

(a)  March 15

(b)  September 15

(c)  January 15

(d)  August 15

(e)  October 15

Answer: (c)

173. ‘Raksha Blu’ is the recently launched vaccine aimed at minimizing the economic loss of the animal farming community from the strains of the

(a)  Foot and Mouth virus

(b)  Blue tongue virus

(c)  Bird flu virus

(d)  Road water disease

(e)  Swine flu virus

Answer: (b)

174. As per 2011 Census, the child sex ratio in the (0-6) age group stood at

(a)  969 females against 1000 males

(b)  939 females against 1000 males

(c)  919 females against 1000 males

(d)  959 females against 1000 males

(e)  929 females against 1000 males

Answer: (c)

175. India’s indigenously developed nuclear capable sub-sonic cruise missile ‘Nirbhay’ can strike targets more than

(a)  1000 km away

(b)  3700 km away

(c)  700 km away

(d)  1700 km away

(e)  2700 km away

Answer: (d)

176. Which of the following countries is the world’s largest producer of gold?

(a)  Canada

(b)  China

(c)  Peru

(d)  Australia

(e)  Colombia

Answer: (b)

177. Who amongst the following has recently been appointed as the Chairperson of the Central Board of Film Certification (CBFC)?

(a)  Pahlaj Nihalani

(b)  Aakash Nihalani

(c)  Govind Nihalani

(d)  Shibha Nihalani

(e)  Dayal Nihalani

Answer: (a)

178. Which of the following is not a primary function of a bank?

(a)  Accepting deposit

(b)  Clearing of cehques

(c)  Providing reference on financial standing on its customer

(d)  Financing of foreign trade

(e)  Granting loans and advances

Answer: (d)

179. As per 2011 Census, the proportion of economically active population in the 15-50 age group in the total population was

(a)  55.5%

(b)  52.5%

(c)  42.5%

(d)  62.5%

(e)  48.5%

Answer: (d)

180. Meterological satellite ‘Fengyun-II 08’ has recently been launched by

(a)  Japan

(b)  China

(c)  Russia

(d)  France

(e)  Germany

Answer: (b)

181. Who amongst the following is the regulator in the financial system of the country?

(a)  Other, than those given as options

(b)  SEBI

(c)  CRISIL

(d)  TRAN

(e)  CERC

Answer: (c)

182. ‘Shivalik Hills’ is a mountain range of the outer Himalayas. The range is about 2400 km long passing through the Indian States of Arunachal Pradesh, West Bengal, Himachal Pradesh, Uttarakhand and

(a)  Punjab

(b)  Uttar Pradesh

(c)  Haryana

(d)  Jammu & Kashmir

(e)  Rajasthan

Answer: (d)

183. The abbreviation ‘PLA’ stands for

(a)  Private Liberation Army

(b)  People’s Liberations Armed force

(c)  Public Liberation Army

(d)  People’s Liberation Army

(e)  People’s Liberty Army

Answer: (d)

184. “SARFAESI Act 2001” is applicable in banking industry for

(a)  regulating the service conditions of employee

(b)  appointment of nominee directions on the bank boards

(c)  Other than those given as options

(d)  streamlining documentation

(e)  facilitating enforcement of security for recovery of bad loans

Answer: (e)

185. The RBI on January 15, 2015 has reduced ‘Repo rate’ by how much percentage points?

(a)  0.60

(b)  0.25

(c)  1.00

(d)  0.75

(e)  0.40

Answer: (b)

186. According to the scientists of NASA’s Goddard Institute for Space Studies in Manhattan, the hottest year in Earth’s recorded history since 1990 was

(a)  2001

(b)  2014

(c)  1997

(d)  2005

(e)  2010

Answer: (b)

187. ‘Mohiniyattam’ is a classical dance from the Indian State of

(a)  Assam

(b)  Kerala

(c)  Odisha

(d)  Arunachal Pradesh

(e)  Telangana

Answer: (b)

188. Who amongst the following won the Nobel Peace Prize for pioneering the concepts of micro credit and micro finance?

(a)  Prof. Mohummad Yunus

(b)  Other than those given as options

(c)  Prof. Amartya Sen

(d)  Jean Tirole

(e)  Prof. Jagdish Bhagwati

Answer: (a)

189. ‘Renminbi’ is the currency of the

(a)  People’s Republic of China

(b)  People’s Republic of Bulgaria

(c)  People’s Republic of Korea

(d)  Peoples’ Republic of Ethiopia

(e)  People’s Republic of Bangladesh

Answer: (a)

190. Loans granted by a bank to an exporter popularly known as ‘Export credit’ is guaranteed, in case of default, by which of the following?

(a)  EXIM Bank

(b)  Minister of Internal Trade, Goa

(c)  ECGC

(d)  DICGC

(e)  Other than those given as options

Answer: (a)

191. ‘Spartan’ is the code-name for an upcoming web browser being developed by Microsoft to replace the existing user?

(a)  Safari

(b)  Opera

(c)  Internet Explorer

(d)  Firefox

(e)  Chrome

Answer: (c)

192. Manohar Gopalkrishna Prabhu Parrikar is the Union Cabinet Minister for

(a)  Railways

(b)  Law and Justice

(c)  Mines and Steel

(d)  Defence

(e)  Civil Aviation

Answer: (d)

193. Which of the following forms of securing a loan is the safest one?

(a)  Lien

(b)  Assignment

(c)  Pledge

(d)  Hypothecation

(e)  Mortgage

Answer: (e)

194. The Asian News International (ANI) is a news agency based in

(a)  Singapore

(b)  New Delhi (India)

(c)  Rawalpindi (Pakistan)

(d)  Tokyo (Japan)

(e)  Beijing (China)

Answer: (b)

195. With a view to encourage entrepreneurship among the schedule castes, ‘a venture fund for the SCs’ has recently been launched by the

(a)  State government of Bihar

(b)  State government of Uttar Pradesh

(c)  State government of Rajasthan

(d)  government of India

(e)  State government of Maharashtra

Answer: (d)

196. The ratio of deposits to loans of a bank is known as

(a)  NPA coverage ratio

(b)  Return on asset ratio

(c)  Asset coverage ratio

(d)  CD ratio

(e)  Other than those given as options

Answer: (b)

197. The loan limit for affordable housing (Valuing up to Rs 425 lakh) in six Metropolitan centres specified by RBI in

(a)  Rs 7000 lakh

(b)  Rs 4000 lakh

(c)  Rs 5000 lakh

(d)  Rs 2000 lakh

(e)  Rs 3000 lakh

Answer: (c)

198. Which of the following is the negotiable instrument?

(a)  Fixed deposit of a bank

(b)  Share certificates issued by a PSU

(c)  Demant draft issued by a bank

(d)  Debenture of a company

(e)  Airway receipt

Answer: (d)

199. ‘Jimjang Deru’ of Arunachal Pradesh has recently broken his own national record set by him last year in

(a)  boxing

(b)  archery

(c)  wrestling

(d)  athletics

(e)  weightlifting

Answer: (e)

200. The scientists at the Harvard Smithsonian Centre for Astrophysics (CIA) have recently found two most Earth like planets. The planets have been named by

(a)  Kinston 410b and Kinston 440b

(b)  Kepler 438b and Kepler 442b

(c)  Kirakon 420b and Kirakon 440b

(d)  Litmus 414b and Litmus 441b

(e)  Luilin 410b and Luilin 421b

Answer: (b)

State Bank of India PO (Pre.) Examination Held on 21-6-2015 Reasoning Question Paper With Answer Key

State Bank of India PO (Pre.) Examination Held on 21-6-2015 Reasoning
State Bank of India PO (Pre.) Examination Held on 21-6-2015 Reasoning Question Paper With Answer Key

State Bank of India PO (Pre.) Examination Held on 21-6-2015

Reasoning

Directions- (Q. 1-5) Study the information carefully and answer the given question.

     A, B, C, D, E, F, G and H are sitting around a circular area of equal distances between each other, but not necessarily in the same order. Some of the people are facing the centre while some face outside. (i.e., in a direction opposite to the centre).

    Note- – Same direction means if one person is facing the centre then the other also faces the centre and vice-versa Opposite direction means that if one person is facing the centre then the other faces outside and vice-versa.

     D sits third to the right of B. E sits second to the left of B. Immediate neighbours of B face the same direction (i.e., if one neighbour faces the centre the other neighbour also faces the centre and vice-versa) C sits second to the left of E. E faces the centre. F sits third to the right of C. G sits second to the left of H. H is not an immediate neighbour of B. G faces the same direction as D. Immediate neighbours of E face opposite directions. (i.e., if one neighbour faces the centre the other neighbour faces outside and vice-versa). Immediate neighbours of F face opposite directions (i.e., if one neighbour faces the centre the other neighbour face outside and vice-versa).

1. How many people in the given arrangement face the centre ?

(A)  Two

(B)  Three

(C)  One

(D)  Four

(E)  Five

Answer: (E)

2. Which of the following is true regarding A as per the given seating arrangement ?

(A)  H sits second to the left of A

(B)  A faces outside

(C)  Only two people sit between A and B

(D)  C is one of the immediate neighbours of A

(E)  Only three people sit between A and G

Answer: (B)

3. Four of the following five are alike in a certain way based on the given seating arrangement and so form a group. Which is the one that does not belong to that group ?

(A)  F

(B)  G

(C)  B

(D)  G

(E)  D

Answer: (B)

4. What is E’s position with respect to H ?

(A)  Third to the left

(B)  To the immediate left

(C)  To the immediate right

(D)  Second to the right

(E)  Third to the right

Answer: (B)

5. Who sits second to the left of G?

(A)  H

(B)  A

(C)  B

(D)  Other than those given as options

(E)  F

Answer: (A)

Directions – (Q. 6-10) Study the given information carefully to answer the given question.

   A, B, C, D, K, L and M live on seven different floors of a building but not necessarily in the same order. The lowermost floor of the building is numbered one, the one above that is numbered two and so on till the topmost floor is numbered seven. Each one of them also likes different games namely. Snooker, Badminton, Chess, Ludo, Cricket, Hockey and Polo (but not necessarily in the same order.)

   Only three people live between B and K. B lives on one of the floors above K. K does not live on the lowermost floor. Only one person lives between B and the one who like Chess. The one who likes Polo lives on one of the even numbered floors above the one who like Chess.

    Only two people live between M and the one who likes Chess. The one who likes Snooker lives immediately above M. A lives immediately above L. A does not like Chess.

    The one who likes Ludo lives on one of the odd numbered floors below L. M does not like Ludo. D lives on one of the Floors above C. Only one person lives between the one who likes Cricket and the one who likes Hockey. D does not like Cricket. M does not like Badminton.

6. Which of the following games does B like ?

(A)  Snooker

(B)  Ludo

(C)  Polo

(D)  Badminton

(E)  Chess

Answer: (D)

7. Who amongst the following lives on the floor numbered 4 ?

(A)  The one who likes Hockey

(B)  The one who likes Chess

(C)  A

(D)  L

(E)  B

Answer: (A)

8. Which of the following statements is true with respect to the given arrangement ?

(A)  Only two people live between K and M

(B)  The one who likes Hockey lives immediately above K

(C)  C likes Chess

(D)  C lives on an even numbered floor

(E)  None of the given options is true

Answer: (B)

9. If all the people are made to sit in alphabetical order from top to bottom, the positions of how many people will remain unchanged ?

(A)  None

(B)  Three

(C)  Two

(D)  One

(E)  Four

Answer: (D)

10. Which of the following combinations is true with respect to the given arrangement ?

(A)  Polo-C

(B)  Ludo-B

(C)  Cricket-K

(D)  Chess-L

(E)  Snooker-A

Answer: (D)

11. A person starts walking from his office towards a party has. He walks for 30 m towards East. He takes a 90 right turn and walks for 15 m. He again takes a 90° right turn, and walks for another 20 m. He then walks for 25 m after taking a 90 left turn. Turning 90° towards his right he walks for 10 m to reach the party hall. How far and in which direction is the party hall from his office ?

(A)  40 m towards West

(B)  40 m towards South

(C)  45 m towards South

(D)  45 m towards North

(E)  40 m towards North

Answer: (B)

Directions – (Q. 12-16) In this question. two/three statements followed by two conclusions numbered I and II have been given. You have to take the given statements to be true even if they seem to be at variance from the commonly facts and then decide which of the given conclusions logically follows from the given statements disregarding commonly known facts.

12. Statements :

Some wins are trophies.

Some trophies are cups.

No cup is a prize.

Conclusions :

I. Atleast some cups are wins.

II. All prizes being trophies is a possibility

(A)  Only conclusion I is true

(B)  Only conclusion II is true

(C)  Either conclusion I or II is true

(D)  Both conclusion I and II are true

(E)  Neither conclusion I nor II is true

Answer: (B)

13. Statement :

No layer is a coat.

All coats are deposits.

All deposits are sheets.

Conclusions :

I. All coats are sheets.

II. All deposits can never be layers.

(A)  Only conclusion I is true

(B)  Only conclusion II is true

(C)  Either conclusion I or II is true

(D)  Both conclusion I and II are true

(E)  Neither conclusion I nor II is true

Answer: (D)

14. Statements :

Some flats are apartments.

No apartment is a hall.

Some halls are rooms.

Conclusions :

I. Atleast some rooms are flats.

II. No apartment is a room.

(A)  Only conclusion I is true

(B)  Only conclusion II is true

(C)  Either conclusion I or II is true

(D)  Both conclusion I and II are true

(E)  Neither conclusion I nor II is true

Answer: (E)

15. Statements :

Some wins are trophies.

Some trophies are cups.

No cup is a prize.

Conclusions :

I. No trophy is a prize.

II. No prize is a win.

(A)  Only conclusion I is true

(B)  Only conclusion II is true

(C)  Either conclusion I or II is true

(D)  Both conclusion I and II are true

(E)  Neither conclusion I nor II is true

Answer: (E)

16. Statements :

Some codes are secrets.

All secrets are puzzles.

Conclusions :

I. All secrets being codes is a possibility

II. Atleast some puzzles are codes.

(A)  Only conclusion I is true

(B)  Only conclusion II is true

(C)  Either conclusion I or II is true

(D)  Both conclusion I and II are true

(E)  Neither conclusion I nor II is true

Answer: (B)

17. Point A is 40 m to the north of point B. Point C is 20 m to the west of point A. Point D is 30 m to the south of point C. Point E is exactly midway between points D and F in such a manner that Point D, E and F form a horizontal straight line of 40 m. Point F is to the east of point D. Point G is 30 m to the north of  point F. How far and in which direction is point G from point A ?

(A)  30 m towards west

(B)  40 m towards north

(C)  20 m towards east

(D)  20 m towards east

(E)  30 m towards east

Answer: (D)

18. Statements : L = P ≤ W < V ≤ K ≥ Q; B < L; K = M

Conclusions : I. B < V

II. M > P

(A)  Only conclusion I is true

(B)  Only conclusion II is true

(C)  Either conclusion I or II is true

(D)  Both conclusion I and II are true

(E)  Neither conclusion I nor II is true

Answer: (D)

19. Statements : L = P ≤ W < V ≤ K ≥ Q; B < L; K = M

Conclusions : I. L ≥ Q

II. W = M

(A)  Only conclusion I is true

(B)  Only conclusion II is true

(C)  Either conclusion I or II is true

(D)  Both conclusion I and II are true

(E)  Neither conclusion I nor II is true

Answer: (E)

20. Statements : R ≤ U = B < S; B ≤ X

Conclusions : I. X > R

II. X = R

(A)  Only conclusion I is true

(B)  Only conclusion II is true

(C)  Either conclusion I or II is true

(D)  Both conclusion I and II are true

(E)  Neither conclusion I nor II is true

Answer: (C)

21. Statements : C > U ≤ S < T = O > D ≥ Y; Z = O ≤ P

Conclusions : I. U > D

II. S < P

(A)  Only conclusion I is true

(B)  Only conclusion II is true

(C)  Either conclusion I or II is true

(D)  Both conclusion I and II are true

(E)  Neither conclusion I nor II is true

Answer: (B)

22. Statements : C > U ≤ S < T = O > D ≥ Y; Z = O ≤ P

Conclusions : I. Z > Y

II. C < O

(A)  Only conclusion I is true

(B)  Only conclusion II is true

(C)  Either conclusion I or II is true

(D)  Both conclusion I and II are true

(E)  Neither conclusion I nor II is true

Answer: (A)

Directions – (Q. 23 to 25) Study the following information and answer the given question.

   J is the sister of T. T is married to D. D is the father of R.

   M is the son of H. T is the mother-in-law of H.

   D has only one son and no daughter.

   J is married to B. L is the daughter of B.

23. How is L related to T ?

(A)  Niece

(B)  Sister

(C)  Cannot be determined

(D)  Daughter

(E)  Mother

Answer: (A)

24. How is M related to D ?

(A)  Nephew

(B)  Uncle

(C)  Brother

(D)  Son

(E)  Grandson

Answer: (E)

25. How is J related to R ?

(A)  Sister

(B)  Aunt

(C)  Mother-in-law

(D)  Mother

(E)  Cannot be determined

Answer: (B)

Directions – (Q. 26 to 30) Study the given information carefully to answer the given question in a certain code language.

   ‘efforts required in teaching’ is written as ‘dp kj te hy’

   ‘nothing kids is tough’ is written as ‘dp op cs mb’

    ‘kids heart is huge’ is written as ‘cs re mb gr’

    ‘task required huge efforts’ is written as ‘hy kj re ba’

   (All codes are two letter codes only)

26. In the given code language, what does the code ‘kj stand for ?

(A)  in

(B)  either ‘required’ or ‘efforts’

(C)  either ‘huge’ or ‘kids’

(D)  task

(E)  huge

Answer: (B)

27. What is the code for ‘teaching’ in the given code language ?

(A)  hy

(B)  te

(C)  op

(D)  other than those given as options

(E)  kj

Answer: (D)

28. What is the code for ‘kids’ in the given code language ?

(A)  either ‘mb’ or ‘cs’

(B)  either ‘op’ or ‘gr’

(C)  dp

(D)  other than those given as options

(E)  re

Answer: (A)

29. What will be the code for ‘in task’ in the given code language ?

(A)  te hy

(B)  dp kj

(C)  ba kj

(D)  ba te

(E)  other than those given as options

Answer: (D)

30. What may be the possible code for ‘quite tough heart’ in the given code language ?

(A)  uc ba re

(B)  gr uc re

(C)  op uc gr

(D)  op kj hy

(E)  op gr kj

Answer: (C)

Directions – (Q. 31 to 35) Study the following information to answer the given question.

    Eight friends E, F, G, H, L, M, N and O are seated in a straight line, facing North, but not necessarily in the same order.

    O sits at the extreme right end of the line. Only four people sit between O and G.

    Both B and M are immediate neighbours of G.

    Only two people sit between M and L. L is not an immediate neighbour of O.

     N sits second to left of E.

31. What is the position of L with respect to G ?

(A)  Third to the right

(B)  To the immediate left

(C)  Second to the right

(D)  Fourth to the right

(E)  Second to the left

Answer: (E)

32. Based on the given arrangement, which of the following is true with respect to N ?

(A)  Only three persons sit between N and O

(B)  None of the given options is true

(C)  Only one person sits to the right of N

(D)  E sits to immediate right of N

(E)  Both L and F are immediate neighbours of N

Answer: (B)

33. Who amongst the following people represents the person seated at the extreme left end of the line ?

(A)  E

(B)  N

(C)  L

(D)  F

(E)  H

Answer: (C)

34. How many persons are seated between O and E ?

(A)  Two

(B)  Three

(C)  None

(D)  Four

(E)  One

Answer: (C)

35. Who amongst the following sit exactly between M and L ?

(A)  E, N

(B)  F, O

(C)  F, G

(D)  E, G

(E)  H, N

Answer: (C)

State Bank of India PO Examination Held on 21-6-2015 Quantitative Aptitude Question Paper With Answer Key

State Bank of India PO Examination Held on 21-6-2015 Quantitative Aptitude
State Bank of India PO Examination Held on 21-6-2015 Quantitative Aptitude Question Paper With Answer Key

State Bank of India PO Examination Held on 21-6-2015

Quantitative Aptitude

1. As present Pia is 5 years older to Ray. The respective ratio between the present ages of Pia and Mini is 3 : 4. At present, Ray is 14 years younger to Mini. What is the Ray’s present age ?

(A)  16 years

(B)  20 years

(C)  14 years

(D)  18 years

(E)  24 years

Answer: (D)

Directions- (Q. 2-6) In the questions two equations numbered (I) and (II) are given. You have to solve both the equations and mark the appropriate answer.

2. (I) 4x2 + 17x + 15 = 0

(II) 3y2 + 19y + 28 = 0

(A)  x ≥ y

(B)  x ≤ y

(C)  x > y

(D)  x < y

(E)  relationship between x and y cannot be established

Answer: (E)

3. (I) 3x2 – 17x + 22 = 0

(II) 5y2 – 21y + 22 = 0

(A)  x ≥ y

(B)  x ≤ y

(C)  x > y

(D)  x < y

(E)  relationship between x and y cannot be established

Answer: (A)

4. (I) 3x2 + 11x + 10 = 0

(II) 2y2 + 13y + 21 = 0

(A)  x ≥ y

(B)  x ≤ y

(C)  x > y

(D)  x < y

(E)  relationship between x and y cannot be established

Answer: (C)

5. (I) 3x2 + 13x + 14 = 0

(II) 8y2 + 26y + 21 = 0

(A)  x ≥ y

(B)  x ≤ y

(C)  x > y

(D)  x < y

(E)  relationship between x and y cannot be established

Answer: (D)

6. (I) 3x2 – 14x + 15 = 0

(II) 15y2 – 34y + 15 = 0

(A)  x ≥ y

(B)  x ≤ y

(C)  x > y

(D)  x < y

(E)  relationship between x and y cannot be established

Answer: (A)

Directions-(Q. 7-11) Study the table to answer the given question.

Data related to expenses by companies A, B, C, D and E in the April 2012.

(Note- Other than the mentioned expenses under the given heads, there may also be some other expense. For calculation purposes, please consider the given expense heads only.)

7. What is the respective ratio between the marketing expense of company A and the marketing expenses of company C?

(A)  34 : 35

(B)  32 : 33

(C)  35 : 36

(D)  36 : 37

(E)  37 : 38

Answer: (D)

8. Total expenses of Company E in October 2012 was 20% more than the total given expenses of the same company in April 2012. How much was the Marketing expenses of the same Company in October 2012, if it was 25% of the total expenses of the same month ? (in Rs Lakhs)

(A)  27

(B)  24

(C)  26

(D)  21

(E)  18

Answer: (D)

9. Marketing expenses of company D is less than marketing expenses of Company B, by approximately what per cent ?

(A)  20

(B)  40

(C)  39

(D)  59

(E)  29

Answer: (E)

10. What is the average total expense of all the companies ? (in Rs. Lakhs)

(A)  75.25

(B)  75.8

(C)  75

(D)  72.8

(E)  72.25

Answer: (B)

11. What is the difference between the total expenses made by company C in legal and marketing together and the total expenses made by company E in legal and marketing together ? (in Rs Lakhs)

(A)  24.5

(B)  23.5

(C)  22.9

(D)  23

(E)  23.9

Answer: (E)

Directions- (Q. 12-21) What approximate value will come in p lace of the question mark (?) in the given question ? (You are not expected to calculate the exact value).

12. 

(A)  14

(B)  22

(C)  2

(D)  16

(E)  8

Answer: (B)

13. 

(A)  670

(B)  570

(C)  710

(D)  510

(E)  610

Answer: (A)

14. 

(A)  625

(B)  400

(C)  25

(D)  900

(E)  225

Answer: (E)

15. 85% of 679 + 19.9% of 219.89 = ?

(A)  115

(B)  145

(C)  65

(D)  105

(E)  85

Answer: (B)

16. 1441 ÷ ? + 149.888 ×99 = 3006 – 254.91 –

(A)  35

(B)  15

(C)  25

(D)  45

(E)  3

Answer: (E)

17. 11 31   69   131  223  ?

(A)  257

(B)  351

(C)  349

(D)  231

(E)  254

Answer: (B)

18. 12 15  20   27  36  ?

(A)  47

(B)  59

(C)  43

(D)  49

(E)  51

Answer: (A)

19. 5  9   18   34   59   95  ?

(A)  272

(B)  168

(C)  116

(D)  148

(E)  144

Answer: (E)

20. 19 20   16   25  9  ?

(A)  32

(B)  38

(C)  34

(D)  42

(E)  48

Answer: (C)

21. 65 70    77   86   97   ?

(A)  110

(B)  121

(C)  100

(D)  112

(E)  90

Answer: (A)

22. A bag contains 5 red balls, 6 yellow balls and 3 green balls. If two balsa re picked at random, what is the probability that either both are red or both are green in colour ?

(A)  3/7

(B)  5/14

(C)  1/7

(D)  2/7

(E)  3/14

Answer: (C)

23. In a 140 litres of mixture of milk and water, percentage of water is only 30%. The milkman gave 20 litres of this mixture to a customer. Then he added equal quantities of pure milk and water to the remaining mixture. As a result the respective ratio of milk and water in the mixture became 2 : 1. What was the quantity of milk added ? (in litres)

(A)  12

(B)  16

(C)  18

(D)  8

(E)  10

Answer: (C)

24. A boat can travel 4.2 km upstream in 14 minutes. If the respective ratio of speed of the boat in still water and speed of the stream is 7 : 1. How much time will the boat take to cover 17.6 km downstream ? (in minutes)

(A)  52

(B)  44

(C)  48

(D)  46

(E)  54

Answer: (B)

25. A starts a business with a capital of Rs 1500. B joins the business 6 months later from the start of the business and C joins the business 8 months later from the start of the business. End of the year their respective share in  profit was in ratio of 5 : 3 : 3. What is the sum of amount put in the business by B and C together ?

(A)  Rs 3300

(B)  Rs 3500

(C)  Rs 4200

(D)  Rs 4800

(E)  Rs 4500

Answer: (E)

26. The length of a rectangle is 4 m more than the side of a square and the breadth of the rectangle is 4 m less than side of the same square. If the area of the square is 576 sq. m., what i s the area of the rectangle ? (in sq. m)

(A)  549

(B)  545

(C)  557

(D)  560

(E)  551

Answer: (D)

27. A sells a bicycle to B at a profit of 20% of B sells it to C at a profit of 25%. If C pays Rs 1500, what did A pay for it ?

(A)  Rs 1000

(B)  Rs 1500

(C)  Rs 600

(D)  Rs 2000

(E)  Rs 500

Answer: (A)

28. 10 men can complete a work in 7 days. But 10 women need 14 days to complete the same work. How many days will 5 men and 10 women need to complete the work ?

(A)  5

(B)  6

(C)  7

(D)  8

(E)  9

Answer: (C)

29. A sum of Rs 6400 earns a compound interest of Rs 1008.80 in 18 months, when the interest is reckoned half yearly. Find the rate interest.

(A)  20%

(B)  15%

(C)  10%

(D)  25%

(E)  30%

Answer: (C)

Directions- (Q. 30-34) Refer to the graph and answer the given question.

30. Number of hats sold by store M during January is what per cent of the total number of hats sold by the store N during March and April together ?

(A) 

(B) 

(C) 

(D) 

(E) 

Answer: (D)

31. What is the average number of hats sold by store N during January, March, May and June ?

(A)  535

(B)  525

(C)  515

(D)  500

(E)  530

Answer: (B)

32. Stores M and N sells only two types of hats : Fedora Hats and Trilby Hats. If the respective ratio of total number of Fedora hats to total number of Trilby hats sold by stores M and N together during March is 9 : 5, what is the total number of Fedora hats sold by stores M and N together during March ?

(A)  990

(B)  900

(C)  720

(D)  630

(E)  810

Answer: (E)

33. If the total number of hats sold by stores M and N together in July is 15% more than the total number of hats sold by the same stores together during June, what is the total number of hats sold by the same stores together during July?

(A)  1298

(B)  1316

(C)  1356

(D)  1284

(E)  1334

Answer: (E)

34. Number of hats sold by store M increased by what per cent from February to May ?

(A)  75

(B)  55

(C)  65

(D)  70

(E)  60

Answer: (C)

35. Monthly salary of Dex is 1/4th of his father’s monthly salary. Dex’s sister’s monthly salary is 2/5th of their father’s monthly salary. Dex’s sister pays Rs 12800, which is 1/4th of her monthly salary as study loan. Savings and expenses made out of the monthly salary, by Dex is in the respective ratio 3 : 5. How much does Dex save each month ?

(A)  Rs 12000

(B)  Rs 10600

(C)  Rs 10400

(D)  Rs 12600

(E)  Rs 12400

Answer: (A)

SBI Assistant Manager System (ITSO) Examination-2015 Held on 17-1-2016 English Language Question Paper With Answer Key

SBI Assistant Manager System (ITSO) Examination-2015 Held on 17-1-2016 English Language
SBI Assistant Manager System (ITSO) Examination-2015 Held on 17-1-2016 English Language Question Paper With Answer Key

SBI Assistant Manager System (ITSO) Examination-2015 Held on 17-1-2016

English Language

 

Directions – (Q. 1 to 5) This question has two blanks, each blank indicating that something has been omitted. Choose the set of words for each blank that best fits the meaning of the sentence as a whole.

1. Soon a gentle long press of the digit 9 on your existing mobile phone will send an immediate ………… to the police as the Government rolls out the panic button……

(A)  figures, system

(B)  alarm, gear

(C)  result, steady

(D)  alert, facility

(E)  signals, system

Answer: (D)

2. Citizens can now …. of various government services by ……. up simplified forms that may be compressed to a single-paper format.

(A)  opt, cleaning

(B)  profit, adding

(C)  yield, drawing

(D)  benefit, writing

(E)  avail, making

Answer: (E)

3. Around 400 dabbawalas have started an ……….. called ‘Rob Bank’ to ………. criminal wastage of food at social functions.

(A)  initiative, prevent

(B)  associate, stop

(C)  effort, abrupt

(D)  program, halt

(E)  attempt, deal

Answer: (A)

4. …………. intense use of energy and materials, levels of pollution are ………….. low in developed economies.

(A)  Inspite of, beyond

(B)  Without, between

(C)  Despite, relatively

(D)  Regardless, still

(E)  Notwithstanding, dangerous

Answer: (C)

5. Phases of economic uncertainty ………… to be a reminder about ……….. having a financial plan and sticking to it are important.

(A)  seems, when

(B)  incline, how

(C)  serve, what

(D)  tend, why

(E)  fail, which

Answer: (D)

Directions – (Q. 6 to 10) Read the sentence to find out whether there is any grammatical mistake/error in it. The error, if any, will be in one part of the sentence. Mark the part with the error as you answer, if there  is no error, mark – No error, as your answer. (Ignore the errors of punctuation if any).

6. It was evident that / the man could not control / his emotions as he thanked /

       (A)                                     (B)                                             (C)  

the donor’s family for saving his life. No error

                   (D)                                     (E)   

Answer: (E)

7. Eating a lot of / processed meat can / led to micro-nutrient deficiencies/

      (A)                          (B)                                         (C)

and cause hunger. No error

         (D)                  (E)  

Answer: (C)

8. Children will / be provided with / energy-dense oral nutritional supplements/

        (A)                   (B)                                 (C)  

and medicines. No error

        (D)             (E)        

Answer: (B)

9. Aided by the cheerful company / of her new found friends, /

           (A)                                           (B)    

the actress opens up about / coming to terms on her father’s demise.

       (C)                                                      (D)       

No error

   (E)   

Answer: (C)

10. Ministers and Officers have been / asked on refraining from /

                  (A)                                          (B)        

making any statement which could / damage the peace process. No error

                  (C)                                                    (D)                                (E)         

Answer: (B)

Directions – (Q. 11 to 15) Which of the phrases given against the sentence should replace the word/ phrase given in bold in sentence to make it grammatically correct ? If the sentence is correct as it is given and no correction is required, select-No correction is required as the answer.

11. If you are one of these who actually include vegetables in the diet, chances are you may just have to rein in your carnivorous instincts.

(A)  has to rein

(B)  had to reining

(C)  rein in

(D)  have to rein

(E)  No correction required

Answer: (E)

12. Good ar quality fundamentally is to good health and the authorities should also let people know this.

(A)  is fundamental.

(B)  fundamentally

(C)  fundamental is

(D)  is fundamentally

(E)  No correction required

Answer: (D)

13. Many upset fans took to social media to expressing their anguish over the secret meeting.

(A)  anguish expressing

(B)  express their anguish

(C)  expressed

(D)  express anguish on

(E)  No correction required

Answer: (B)

14. Depression, clinical or physiological, takes a great toll on your system in more ways then ones.

(A)  more ways than one

(B)  most way

(C)  many ways and one

(D)  many way then

(E)  No correction required

Answer: (A)

15. Even the many of diligent and health conscious people end up giving in to temptation.

(A)  Most of

(B)  Even

(C)  Even the most

(D)  Each of the

(E)  No correction required

Answer: (C)

Directions – (Q. 16 to 25) Read the following passage carefully and answer the question. Certain words/ phrases are given in bold to help you locate them while answering some of the questions.

   Earlier this year, Indonesia’s President promised a ‘massive deregulation’ aimed at attracting foreign investment. Outsiders were thrilled. His predecessor, left the country’s business climate chocking on what Adam Schwarz, a consultant, calls ‘a regulatory miasma’ that strongly discouraged investment, whereas the new President, has openly courted foreign capital. Over the past six weeks his administration has unveiled a series of deregulatory measures. The government made it easier for foreigners to open bank accounts, struck down import restrictions on goods such as tyres and cosmetics that were designed to protect local industries and eliminated some onerous and silly business regulations. No longer, for instance, most Indonesian-language labels be affixed to imported goods before they arrive; now they can be printed in Indonesia and attached before public circulation. The time required to process some investment permits was cut and taxes were cut for exporters who deposit foreign-exchange revenue in Indonesia or convert it into rupiah-a move to shore up the country’s wobbly currency.

   These are the sort of simple, pratical measures that are completely and directly felt by industry. And to its credit, Indonesia has resisted the temptation to panic in the face of a plunging currency and rising bond yields. It has, for instance, maintained fiscal discipline-aided by a law that caps the budget deficit at 3%. Markets nonetheless seem unconvinced. The rupiah continued its slide after the first two announcements. It has recovered some ground this month, along with other emerging-market currencies, but has still fallen by 8% against the dollar this year. Economic growth is at its slowest since 2009. Nobody doubts the new deregulatory measures are better than nothing, but they are hardly ‘massive’. One foreign businessman, long resident in Indonesia, assesses them as resulting from “bureaucrats talking to themselves about how we can be a better bureaucracy rather than how we can be more receptive to foreign investment.” For the most part, the President’s new measures remove regulations that should never have  been implemented in the first place. They neither fundamentally change Indonesia’s investment climate nor signal to investors that Indonesia is preparing for bigger reforms.

    Indonesia’s negative-investment list, which details the sectors that are barred to foreign capital, remains sizeable. Hiring foreigners is still a burdensome process : one rule requires businesses to hire ten Indonesians for every foreign worker. Businesses complain that bureaucrats pass rules hastily, without even trying to understand their effect on the private sector. A rule banning metal-ore exports remains in place and will continue to remain so; it was intended to encourage a domestic smelting industry but instead has cost thousands of jobs and billions in export revenue.

    Infrastructure development-the centerpiece of the President’s ambitious economic plans-has begun to pick up, but only after severe delays, and the programme remains well below its targets for this year. Perhaps most damaging is a pervasive sense of disarray. Policies are announced and the scrapped,, whether because objections that should have been aired before, as with a law to force foreigners to pass a language test, or because they conflict with other plans, as happened with a proposed road tax. Ministries seem to pass rules independently, without consulting each other or the President. Decentralization-meaning a huge devolution of power from the national government to the regional level-may have held the country together in the early 2000s, but today it impedes infrastructure development and  hinders policy coordination. Poor communication from the President compounds these  problems. The good news, as Mr. Schwarz notes, “is that country has come to an intersection and the President has said, “I’ve got to do something different because what we’ve been doing isn’t working.” These bold words are welcome. But bold actions would be better still.

16. Which of the following can be inferred from the passage?

(A)  Indonesia’s  new government has adopted communications about implementing schemes

(B)  Indonesia has too many unnecessary regulatory procedures and systems.

(C)  Indonesian economy is heavily dependent on exports of goods lie cosmetics

(D)  The current government has n intention of truly reforming Indonesia.

(E)  None of the given options can be inferred from the passage

Answer: (A)

17. Why according to the author, are foreign investors not attracted to Indonesia?

1. Time consuming regulations

2. Stock market crash in 2009

3. Lack of political stability

(A)  Only 1

(B)  Only 2

(C)  All 1, 2 and 3

(D)  Only 1 and 3

(E)  Only 2 and 3

Answer: (A)

18. Choose the world which is mot nearly the same in meaning to the word DISCIPLINE given in bold as used in the passage.

(A)  punishment

(B)  specially

(C)  order

(D)  classification

(E)  train

Answer: (C)

19. According to the author how can the Indonesian economy regain health ?

(A)  Putting a brake on its massive reform agenda

(B)  Removal of concessions to exporters

(C)  Stricter control of the private sector

(D)  Ensure stability by implementing the previous government’s policies

(E)  Other than those given as options

Answer: (E)

20. According to the passage, which of the following is/are (a) measure(s) that has/ have been implemented by the present inclination government?

1. Reduction in imports to reduce the fiscal deficit

2. Reducing the red tape for business.

3. Opening up of the mining sector by encouraging foreign investment in it.

(A)  Only 1

(B)  Only 2

(C)  All 1, 2 and 3

(D)  Only 1 and 3

(E)  Only 2 and3

Answer: (E)

21. Which of the following is the central idea of the passage ?

(A)  Indonesia needs more meaningful and developing reforms to boost economic growth

(B)  Indonesia’s present government is not doing enough and should be voted out of power

(C)  Indonesia is headed for a financial crash and foreign investors are pulling out quickly

(D)  Indonesia’s reforms are too radical in nature and must destabilize its economy

(E)  Other than those given as options

Answer: (A)

22. What do the statistics cited about Indonesia’s economy reveal ?

(A)  Indonesia’s economy was not affected by the financial crisis of 2008

(B)  The measures taken by Indonesia’s new government have ensured economic recovery

(C)  The Indonesian government has not revealed the true extent of its economic recession

(D)  Indonesia’s economy is not flourishing at present outside the government’s efforts

(E)  Other than those given as options

Answer: (B)

23. Choose the word which is most opposite in meaning to the word PERVASIVE given in bold as used in the passage.

(A)  invasive

(B)  determinant

(C)  restrictive

(D)  insensitive

(E)  continual

Answer: (C)

24. Which of the following describes Indonesia’s labour market at present ?

(A)  Its labour force is not skilled and foreign workers are much in demand

(B)  It will  provide employment to millions of workers in the mining industry

(C)  It provides the right to locate the cost of hiring foreign workers

(D)  It has implemented an ambitious programme to skill Indonesian workers

(E)  Not clearly mentioned in the passage

Answer: (E)

25. Which of the following best describes the author’s views of Indonesia’s attempt at decentralization of powers?

(A)  It has been well implemented as the President cannot interfere at the regional level

(B)  It will foster corruption and  has hampered development of the lucrative mining industry

(C)  It has many ministries accountable for their decisions and improved coordination among them

(D)  It is a bold move which he hopes will succeed when implemented

(E)  Other than those given as option

Answer: (A)

Directions- (Q. 26 to 35) In the given passage there are blanks, each of which has been numbered. Against each, five words are suggested, one of which fits the blank appropriately. Find out the appropriate word in each case.

    Many multinationals think they understand and have tried to ……..(26)……. the serious risks posed by operating in China-intellectual-property-rights violations, corruption, lack of transparency, potential political instability ….(27)…. one of the highest risks of all China’s massive environmental degradation-in …..(28)…. discussed in cooperate boardrooms. China’s environmental problems are ….(29)…. the point where they could ….(30)…. its GDP growth. Its State Environmental Protection Administration (SEPA) concluded in June 2006 that environmental degradation and pollution …(31)…. the Chinese economy the equivalent of 10% of GDP annually. This figure is echoed in more specific costs reported in the Chinese press : up to $ 13 billion from the degradation and health impact of acid rain, $ 6 billion from the spread of desert regions, and the list goes on. The effect on the population is …..(32)… Already more than 4,00,000 people die each year as a result of the country’s air pollution, according to environmental experts and an estimated 190 million people drink water so contaminated that it makes them sick. Some 40 million people have had to migrate because their local ecology can no longer ….(33)…. them. The Chinese leadership is now concerned that environmental …..(34)… is leading to social unrest. The domestic media reported 50,000 environmental protests in 2005. Such protests are usually small in scale, but some have engaged upwards of 30,000 to 40,000 people, some have been violent, and they are increasing in…(35)…

26.

(A)  elevate

(B)  mitigate

(C)  amplify

(D)  cure

(E)  relieve

Answer: (B)

27.

(A)  Because

(B)  Thus

(C)  Yet

(D)  Till

(E)  Until

Answer: (C)

28.

(A)  almost

(B)  scanty

(C)  popularly

(D)  just

(E)  barely

Answer: (E)

29.

(A)  reaching

(B)  beyond

(C)  arriving

(D)  joining

(E)  coming

Answer: (A)

30.

(A)  stay

(B)  drive

(C)  curl

(D)  constrain

(E)  sustain

Answer: (D)

31.

(A)  compensate

(B)  amount

(C)  cost

(D)  affect

(E)  vitiate

Answer: (D)

32.

(A)  alarming

(B)  distressed

(C)  countering

(D)  delightful

(E)  shoddy

Answer: (A)

33.

(A)  approve

(B)  bare

(C)  harm

(D)  sustain

(E)  survive

Answer: (D)

34.

(A)  leading

(B)  causing

(C)  appealing

(D)  advancing

(E)  bulging

Answer: (E)

35..

(A)  frequent

(B)  number

(C)  sum

(D)  numerical

(E)  representation

Answer: (B)

SBI Clerical Cadre Preliminary Examination Held on May 22, 2016 Question Paper With Answer Key

SBI Clerical Cadre Preliminary Examination Held on May 22, 2016
SBI Clerical Cadre Preliminary Examination Held on May 22, 2016 Question Paper With Answer Key

SBI Clerical Cadre Preliminary Examination Held on May 22, 2016

Part I English Language

Directions (Q. Nos. 1-10) Read the passage carefully and answer the questions given below it Certain words/phrases have been given in bold to help you locate them while answering some of the questions.

Once upon a time, there lives a washerman in a village. He had a donkey by the name of Udhata. He used to carry loads of clothes to the river bank and back home everyday. The donkey was not satisfied with the food, that was given to him by his master to eat So he wandered into the nearby fields stealthily and ate the crops growing there. Once, the donkey, while wandering around, happened to meet a fox. Soon, both of them became friends and beg a in to wander together in search of delicious food.

One night, the donkey and the fox were eating water-melons in a field. The water-melons were to tasty, that the donkey ate in a large quantity. Having eaten to his appetite, the donkey became so happy that he was compelled by an intense desire to sing, He told the fox that he was in such a good mood that he had to express his happiness in a melodious tone. ‘Don’t be a fool. If you sing, the people sleeping in and around this field will wake up and beat us  black and blue with sticks:’ said the fox worriedly. ‘You are a dull fellow’, the donkey said hearing the words of fox. ‘Singing make one happy and healthy. No matter what comes.

I’ll definitely sing a song. “The box became worried to see the donkey adamant to sing a song in the midst of the field, while the owner was still sleeping only a little distance away. Seeing his adamance, he said to the donkey, ‘Friend, wait a minute before you s tart First, let me Jump over to the other side of the fence for my safety’. Saying so the fox jumped over to the other side of the fence without losing a moment. The donkey began in his so-called melodious tone. Hearing suddenly, a donkey braying in the field, the owner woke up from his sleep. He picked up his stick lying by his side and ran towards the donkey who as still braying happily. The owner of the field looked around and saw the loss caused by the donkey. He became very angry and beat him so ruthlessly that the donkey was physically incapacitated temporarily.

He, somehow, managed to drag himself out of the field with great difficult. The fox looked at the donkey and said in a sympathetic tone, ‘I’m sorry to see you in this pitiable condition. I had already warned you, but you didn’t listen to my advice. ‘The donkey too realized his folly and hung his head in shame.

1. Why did donkey want to sing?

(a)  Donkey was so happy that he was compelled by an intense desire to sing.

(b)  He wanted to wake up the people sleeping around.

(c)  Fox told the donkey to sing a song.

(d)  He wanted to prove that he was a good singer.

(e)  None of the above

Answer: (a)

2. What was donkey’s reaction when fox suggested him not to sing a song?

(a)  Donkey agreed with fox and did not sing a song.

(b)  Donkey chose the dance instead of singing a song.

(c)  Donkey became angry and did no t talk to fox after that

(d)  Donkey didn’t pay attention to fox words and sang a song.

(e)  None of the above

Answer: (d)

3. What happened when donkey sang a song?

(a)  The fox clapped for the donkey.

(b)  The villagers praised donkey and gave him a gift.

(c)  The fox started dancing

(d)  The owner woke up and beat the donkey ruthlessly.

(e)  None of the above

Answer: (d)

4. How did donkey came to know the fox?

(a)  Both where childhood friends.

(b)  Donkey happened to meet the fox in the field while wandering around in the search of delicious food.

(c)  They used to work together for the washerman from the starting.

(d)  Fox once saved donkey life.

(e)  None of the above

Answer: (b)

5. Which of the following statement is false according to the passage?

(a)  The donkey realized his mistake in the end.

(b)  The donkey was not satisfied with the food given to him by his master.

(c)  Donkey didn’t want to sing but he was compelled to do that

(d)  The owner of the field woke LIP after hearing to the donkey is voice

(e)  None of the above

Answer: (c)

Directions (Q. Nos. 6-8) Choose the word/group of words which is most similar in meaning to the word/group of words as used in the passage.

6. Stealthily

(a)  Superstitiously

(b)  Secretly

(c)  Honestly

(d)  Openly

(e)  Overtly

Answer: (b)

7. Compelled

(a)  Discouraged

(b)  Delayed

(c)  Suspended

(d)  Forced

(e)  Demanded

Answer: (d)

8. Adamance

(a)  Easy going

(b)  Yielding

(c)  Flexible

(d)  Mischievous

(e)  Rigid

Answer: (e)

Directions (Q. Nos. 9-10) Choose the word/group of words which is most opposite in meaning of the word/group of words as used in the passage.

9. Frolly

(a)  Stupidity

(b)  Madness

(c)  Advanceness

(d)  Sageness

(e)  Sharpness

Answer: (c)

10. Pitiable

(a)  Miserable

(b)  Misfortunate

(c)  Pathetic

(d)  Honourable

(e)  Commendable

Answer: (e)

Directions (Q. Nos. 11-15) Rearrange the following six sentence A, B, C, D, E and F in the proper sequence to form a meaningful paragraph and then answer the questions given below.

A. Anything that you touch will turn into gold. The king was delighted with his good fortune.

B. Even though he was very rich he always craved for more and more.

C. Everything he touched turned into gold. He turned trees, grass, tables, chairs, flowers, and vases into gold.

D. One day, he called his court magician and commanded, Find me a spell that can get me more treasures than I already have.

E. King Midas was a very greedy king.

F. The magician said, Your majesty, I can given you a power that no one else in this world has.

11. Which of the following should be the FIRST sentence after rearrangement?

(a)  F

(b)  E

(c)  B

(d)  A

(e)  C

Answer: (b)

12. Which of the following should be the SECOND sentence after rearrangement?

(a)  A

(b)  B

(c)  C

(d)  E

(e)  D

Answer: (e)

13. Which of the following should be the THIRD sentence after rearrangement?

(a)  E

(b)  F

(c)  D

(d)  C

(e)  B

Answer: (b)

14. Which of the following should be the FOURTH sentence after rearrangement?

(a)  A

(b)  B

(c)  F

(d)  E

(e)  CC

Answer: (a)

15. Which of the following should be the LAST (SIXTH) sentence after rearrangement?

(a)  C

(b)  A

(c)  B

(d)  D

(e)  F

Answer: (b)

Directions (Q. Nos. 1-20) Read each sentence to find out whether there is any grammatical or idiomatic error in it. The error, if any, will be in one part of the sentence. The number of that part is the answer. If there is ‘No error’, the answer is (e). (Ignore errors of punctuation, if any.)

16. (a) Harsh Vardhan said he was looking forward / (b) to serve his roots, as he had spend / (c) the first 15 years of his life / (d) in the streets of old Delhi. (e) No error

Answer: (b)

17. (a) A family office has to / (b) execute all tasks related to / (c) manage the wealth of / (d) the upper-rich family. (e) No error

Answer: (c)

18. (a) Service providers in the financial industry, specially private banks / (b) and wealth management firms, are increasingly / (c) forced to standardize their / (d) products to optimize profit. (e) No error

Answer: (e)

19. (a) Social or impact funds are private equity-like funds /(b) that pool money from investors / (c) and put it to work in a portfolio of ventures / (d) that meet the funds objective. (e) No error

Answer: (a)

20. (a) While residential land purchases/ (b) have benefitted many investors/ (c) buying land can be risky / (d) especially for a NRI. (e) No error.

Answer: (c)

Directions (Q. Nos. 21-25) In each question below, four words printed in bold type are given. These are indicated as (a), (b), (c) and (d). One of these words printed in bold may either be wrongly spelt or in appropriate in the context of the sentence. Find out the word that is inappropriate or wrongly spelt, if any. The number of that word is your answer. If all the words printed in bold are correctly spelt and appropriate in the context of the sentence then mark (e) i.e. ‘All correct’, as your answer.

21. All the competitors (a)/ completed (b) / the race (c) / with just one (d) All correct (e)

Answer: (d)

22. Poor posture (a) / can lead (b) / to muscular (c) / problems (d)/ in later life. All correct (e)

Answer: (e)

23. The pump (a) / shut off (b) / as a result (c) / of a mechanikal (d) / failure. All correct (e)

Answer: (d)

24. The Principal (a) / gave a very pompous (b) / speach (c) / about ‘The portals of learning’. (d) All correct (e)

Answer: (c)

25. Copeing (a)/ with her mother’s long illness (b)/ was a heavy load (c)/ to (d) All correct (e)

Answer: (a)

Directions (Q. Nos. 26-30) In the following passage, some of the words have been left out. each of which is indicated by a number. Find the suitable word from the options given against each number and fill up the blanks with appropriate words to make the paragraph meaningfully complete.

A camel and a jackal were friends. One day the jackal …(26)… his friend to a big sugarcane farm. It was on the opposite side of a river. After a …(27)… meal the Jackal began to howl loudly. The frightened camel pleaded …(28)… the Jackal not to do so. The jackal said, “Friend, I have this habit after every meal. I cannot help it.” Soon the farmers arrived and gave a sound thrashing to the camel.

When the camel crossed the river the jackal joined him on his back. In the midstream, the camel took a deliberate dip …(29)… the water. When the jackal cried out in terror, the camel said casually “I have the habit of rolling in the water after every meal.” The poor jackal was …(30)… .

26.

(a)  brought

(b)  bought

(c)  took

(d)  taken

(e)  invites

Answer: (c)

27.

(a)  dally

(b)  desirous

(c)  delicacy

(d)  dinner

(e)  sumptuous

Answer: (e)

28.

(a)  to

(b)  for

(c)  on

(d)  with

(e)  before

Answer: (e)

29.

(a)  onto

(b)  in

(c)  within

(d)  down

(e)  on

Answer: (b)

30.

(a)  sank

(b)  sinking

(c)  drowning

(d)  drowned

(e)  wetting

Answer: (d)

Part II Reasoning Ability

Directions (Q. Nos. 31-35) To answer these questions study carefully the following arrangement of letters, digits and symbols.

M 7 ∑ 8 L P @ ? 6 N B T Y 3 2 = E $ 4 9 © G H 5

31. How many such letters are there in the arrangement each of which is immediately followed by a number?

(a)  Three

(b)  Four

(c)  One

(d)  Two

(e)  None of these

Answer: (a)

32. How many such symbols are there in the arrangement each of which is immediately preceded by a number?

(a)  Two

(b)  Three

(c)  Four

(d)  Nil

(e)  None of these

Answer: (b)

33. If all the symbols are deleted from the arrangement, then which of the following will be fourth to the left of the 17th element from the left end?

(a)  9

(b)  E

(c)  2

(d)  Y

(e)  None of these

Answer: (b)

34. ‘78’ is to ‘P?6’ and ‘?N’ is to ‘T32’ in the same way as ‘2E’ is to ……….. in the arrangement.

(a)  4©H

(b)  49G

(c)  4©G

(d)  9GH

(e)  None of these

Answer: (c)

35. If all the numbers are deleted from the arrangement then which of the following will be fifth to the right of the 13th element from the right end?

(a)  B

(b)  N

(c)  Y

(d)  T

(e)  None of these

Answer: (a)

Directions (Q. Nos. 36-40) In these questions, a relationship between different elements is shown in the statements. The statements are followed by two conclusions.

Give answer

a. if only conclusion I is true

b. if only conclusion II is true

c. if either conclusion I or II is true

d. if neither conclusion I nor II is true

e. if both conclusions are true

36. Statements A > B > C < D,

C = E > G

Conclusions I. D > E

II. B > E

Answer: (b)

37. Statements P > Q > M > N, Q = S

Conclusions I. S > P        II. N < S

Answer: (b)

38. Statement S > M = Z > T < Q > V

Conclusions I. V = S        II. Q > M

Answer: (d)

39. Statement T < U = V < S > P > Q

Conclusions I. S > T        II. V < Q

Answer: (e)

40. Statements M < N > R > W,

E = J > L > W

Conclusions I. E > W       II. M > L

Answer: (a)

Directions (Q. Nos. 41-45) The following questions are based on the five three-digit numbers given  below:

684      512    437    385    296

41. If 2 is added to the first digit of each of the numbers how many numbers thus formed will be divisible by three?

(a)  None

(b)  One

(c)  Two

(d)  Three

(e)  None of these

Answer: (b)

42. If all the digits in each of the numbers are arranged in descending order within the number, which of the following will be the highest number in the new arrangement of numbers?

(a)  684

(b)  385

(c)  296

(d)  437

(e)  None of these

Answer: (c)

43. What will be the resultant number if the second digit of the second lowest number is divided by the third digit of the highest number?

(a)  2

(b)  3

(c)  0

(d)  1

(e)  4

Answer: (a)

44. If 1 is added to the first digit and 2 is added to the last digit of each of the numbers then which of the following n umbers will be the second highest number?

(a)  385

(b)  684

(c)  437

(d)  296

(e)  512

Answer: (e)

45. If in each number the first and second digits are interchanged then which will be the highest number?

(a)  296

(b)  512

(c)  437

(d)  684

(e)  385

Answer: (a)

Directions (Q. Nos. 46-47) Study the following information carefully and answer the questions given below:

P is to the North of Q and S is to the East of P, who is to the South of W. T is to the West of P.

46. Who among the following is towards South of W and North of Q?

(a)  P

(b)  T

(c)  S

(d)  Q

(e)  None of these

Answer: (a)

47. W is in which direction with respect to T?

(a)  North

(b)  North-East

(c)  South-West

(d)  West

(e)  None of these

Answer: (b)

Directions (Q. Nos. 48-52) Study the following information carefully and answer the questions given below:

Dhondu, Chintu, Titu, Chiku, Sonu, Monu, Bittu and Sonty are sitting around a circular table facing the centre. Sonty is third to the right of Titu and second to the left of Sonu. Chintu is not an immediate neighbour of Sonty or Titu. Monu is second to the right of Chiku and is in immediate neighbour of Titu. Bittu is not the neighbour of Sonu.

48. Who among the following is second to the right of Titu?

(a)  Sonty

(b)  Bittu

(c)  Monu

(d)  Sonu

(e)  None of these

Answer: (b)

49. Who among the following is an immediate neighbour of Sonty and Sonu?

(a)  Dhondu

(b)  Chintu

(c)  Titu

(d)  Bittu

(e)  None of these

Answer: (a)

50. In which of the following pairs the second person is sitting on the immediate right of the first person?

(a)  Dhondu, Sonty

(b)  Titu, Chiku

(c)  Bittu, Sonty

(d)  Sonu, Sonty

(e)  Monu, Titu

Answer: (c)

51. Who among the following is second to the left of Chintu?

(a)  Titu

(b)  Sonty

(c)  Monu

(d)  Dhondu

(e)  None of these

Answer: (d)

52. Who among the following is opposite to Chiku?

(a)  Dhondu

(b)  Bittu

(c)  Sonty

(d)  Sonu

(e)  None of these

Answer: (c)

Directions (Q. Nos. 53-57) Study the following information carefully to answer the given questions.

Seven neighbours S, P, L, Q, R, M and I live on different floors in the same building having seven floors numbered one to seven. (The first floor is numbered one, the floor above it is numbered two and so on and the topmost floor is numbered as seven.) Three persons live between I and M. M lives on the floor above S, who does not live on an odd-numbered floor. P is neither live on odd number nor on topmost floor. I does not live on the first floor. Two persons live between I and S. Q lives neither on the first floor nor on the fourth floor.

53. Who lives on the floor just above M?

(a)  I

(b)  P

(c)  Q

(d)  R

(e)  None of these

Answer: (c)

54. How many persons live between L and P?

(a)  None

(b)  One

(c)  Two

(d)  Three

(e)  Can’t be determined

Answer: (c)

55. Which of the following pairs live on the first floor and the topmost floor respectively?

(a)  L, Q

(b)  Q, P

(c)  I, Q

(d)  L, I

(e)  Can’t be determined

Answer: (a)

56. Who among the following lives on the topmost floor?

(a)  I

(b)  Q

(c)  P

(d)  L

(e)  None of these

Answer: (b)

57. Which of the following combinations is true?

(a)  First floor – S

(b)  Fourth floor – R

(c)  Third floor – M

(d)  Sixth floor – I

(e)  None of the above

Answer: (e)

58. How many pairs of letters are there in the word (in forward direction) APPLICATION, each of which have as many letters between then in the word as they have between then in the English alphabet?

(a)  One

(b)  Two

(c)  Three

(d)  Four

(e)  None of these

Answer: (a)

59. In a certain coding system, PAPER is written as PERPA and SUBJECT is written as JECTSUB, what should be the code for COUNCIL?

(a)  NCILCOU

(b)  LICNOUC

(c)  NCOUCIL

(d)  NLICUOC

(e)  None of these

Answer: (a)

60. In a certain code language ‘lu ja ka hu’ means’ will you meet us’, ‘lu ka hu p a’ means ‘will you sold us’. Then what is the code of ‘meet’ in this code languge?

(a)  ja

(b)  lu

(c)  ka

(d)  hu

(e)  cannot be determined

Answer: (a)

61. In a certain code language, COMBINE is written as XLNYRMV. How will TOWARDS be written in the that code language?

(a)  FLDZIWJ

(b)  GLDZIWH

(c)  GLEZJWH

(d)  FLEZIWH

(e)  None of these

Answer: (b)

62. 37 girls are standing in a row facing the school building. Ayesha is fifteenth from the left end. If she is shifted six places to the right what is her position from the right end?

(a)  16th

(b)  21st

(c)  20th

(d)  18th

(e)  None of these

Answer: (e)

63. X is mother is the mother-in-law of the father of Z. Z is the brother of Y while X is the father of M. How is X related to Z?

(a)  Paternal uncle

(b)  Maternal uncle

(c)  Cousin

(d)  Grandfather

(e)  Brother-in-law

Answer: (b)

64. If A is a brother of B, C is t he sister of A, D is the brother of E, E is the daughter of B, F is the father of C. Who is the uncle of D?

(a)  A

(b)  C

(c)  B

(d)  None of these

(e)  Can’t be determined

Answer: (a)

65. A said to B that B’s mother was the mother-in-law of A’s mother. How is A’s mother related to B’s mother?

(a)  Daughter-in-law

(b)  Mother-in-law

(c)  Sister

(d)  Aunt

(e)  None of these

Answer: (a)

Part III Numerical Ability

Directions (Q. Nos. 66-70) What should come in the place of question marks in the following questions?

66. 

(a)  133.4

(b)  137.2

(c)  127.8

(d)  131.6

(e)  None of these

Answer: (a)

67. 12% of 555 + 15% of 666 = ?

(a)  166.5

(b)  167.5

(c)  168.5

(d)  169.5

(e)  None of these

Answer: (a)

68. 84368 + 65466 – 72009 – 13964 = ?

(a)  61481

(b)  62921

(c)  63861

(d)  64241

(e)  None of these

Answer: (c)

69. 338 × 331.2 ÷ 335 = 33 × 33?

(a)  2.8

(b)  3

(c)  3.2

(d)  4

(e)  6

Answer: (b)

70. 

(a)  404

(b)  408

(c)  410

(d)  414

(e)  416

Answer: (e)

Directions (Q. Nos. 71-75) Study the following table carefully and answer the given questions.

The number of various crimes, as supplied by national crime record, reported in different States in the year 2012-13

71. The total number of various crimes in Himchal Pradesh is

(a)  37803

(b)  38903

(c)  37903

(d)  36903

(e)  37003

Answer: (d)

72. Find the ratio of Stalking and Assault in Uttar Pradesh to Theft and Criminal Trespass in Haryana.

(a)  28 : 51

(b)  21 : 52

(c)  52 : 21

(d)  14 : 55

(e)  55 : 14

Answer: (b)

73. Find the approximate average of Murder and Theft in all the eight States together.

(a)  1141

(b)  1132

(c)  1311

(d)  941

(e)  1021

Answer: (e)

74. The total number of Assaults and Murders together in Bihar is what percent of the total number of crimes in that State?

(a)  29.82%

(b)  39.82%

(c)  25%

(d)  21.82%

(e)  25.5%

Answer: (b)

75. Find the difference between the number of various crimes committed in Bihar and that in Rajasthan.

(a)  105

(b)  98

(c)  145

(d)  139

(e)  104

Answer: (c)

Directions (Q. Nos. 76-80) What should come in the place of question marks in the following questions.

76. 4376 + 3209 – 1784 + 97 = 3125 + ?

(a)  2713

(b)  2743

(c)  2773

(d)  2793

(e)  2737

Answer: (c)

77. 

(a)  1521

(b)  1369

(c)  1225

(d)  961

(e)  1296

Answer: (b)

78. 85% of 420 + ?% of 1080 = 735

(a)  25

(b)  30

(c)  35

(d)  40

(e)  45

Answer: (c)

79. 

(a)  8920

(b)  8940

(c)  8960

(d)  8820

(e)  8640

Answer: (d)

80. 30% of 1225 – 64% of 555 = ?

(a)  10.7

(b)  12.3

(c)  13.4

(d)  17.5

(e)  None of these

Answer: (b)

81. How many litres of water should be added to a 30 L mixture of milk and water containing milk and water in the ratio of 7 : 3 such that the resultant mixture has 40% water in it?

(a)  5 L

(b)  2 L

(c)  3 L

(d)  8 L

(e)  7 L

Answer: (c)

82. The SI on certain sum of money for 15 months at rate o f 7.5% per annum exceed the SI on same sum at 12.5% per annum for 8 months by Rs 3250 find sum?

(a)  Rs 160000

(b)  Rs 20000

(c)  Rs 170000

(d)  Rs 18000

(e)  Rs 312000

Answer: (e)

83. 4 men and 3 women finish a job in 6 days, and 5 men and 7 women can do the same job in 4 days. How long will 1 man and 1 woman take to do the work?

(a) 

(b) 

(c) 

(d) 

(e)  None of these

Answer: (a)

84. A and B started a business with initial investments in the ratio 5 : 7. If after one year their profits were in the ratio 1 : 2 and the period for A’s investment was 7 months, B invested the money for

(a)  6 months

(b) 

(c)  10 months

(d)  4 months

(e)  7 months

Answer: (c)

85. An army lost 10% its men in war, 10% of the remaining due to diseases and 10% of the rest were disabled. Thus, the strength was reduced to 729000 active men. Find the original strength.

(a)  1000000

(b)  1200000

(c)  1500000

(d)  180000

(e)  None of these

Answer: (a)

86. What is the difference between the compound interests on Rs 5000 for 1 years at 4% per annum compounded yearly and half-yearly?

(a)  Rs 2

(b)  Rs 3

(c)  Rs 4

(d)  Rs 8

(e)  None of these

Answer: (a)

87. The speeds of John and Max are 30 km/h and 40 km/h. Initially, Max is at a place L and John is at a place M. The distance between L and M is 650 km. John started his journey 3 hours earlier than Max to meet each other. If they meet each other at a place P some where between L and M, then the distance between P and M is

(a)  220 km

(b)  250 km

(c)  330 km

(d)  320 km

(e)  None of these

Answer: (c)

88. The average weight of boys in a class is 30 kg and the average weight of girls in the same class is 20 kg. If the average weight of the whole class is 23.25 kg, what could be the possible strength of boys and girls respectively in the same class?

(a)  14 and 26

(b)  13 and 27

(c)  17 and 27

(d)  19 and 21

(e)  None of these

Answer: (b)

89. A profit of 8% is made by selling a shirt after offering a discount of 12%. If the marked price of the shirt is Rs 1080, find it cost price.

(a)  Rs 890

(b)  Rs 780

(c)  Rs 880

(d)  Rs 900

(e)  None of these

Answer: (c)

90. The difference between 4/5 of a number and 45% of the number is 56. What is 65% of the number?

(a)  96

(b)  104

(c)  112

(d)  120

(e)  None of these

Answer: (b)

91. A man can row 24 km upstream and 54 km downstream in 6 hours. He can also row 36 km upstream and 48 km downstream in 8 hours. What is the speed of the man in still water?

(a)  18.75 km/h

(b)  19.25 km/h

(c)  17.65 km/h

(d)  15.55 km/h

(e)  22.75 km/h

Answer: (b)

92. The numerator of a fraction is decreased by 25% and the denominator is increased by 250%. If the resultant fraction is 6/5, what is the original fraction?

(a)  22/5

(b)  24/5

(c)  27/6

(d)  28/5

(e)  30/11

Answer: (d)

93. What would be the area of a rectangle whose area is equal to the area of circle of radius 7 cm?

(a)  77 cm2

(b)  154 cm2

(c)  184 cm2

(d)  180 cm2

(e)  150 cm2

Answer: (b)

94. In a village, three people contested for post of village Pradhan. Due to their own interest, all the voters voted and no one vote was invalid. The losing candidate got 30% votes. What could be the minimum absolute margin of votes by, which the winning candidate led by the nearest rival, if each candidate got an integral percent of votes?

(a)  4

(b)  2

(c)  1

(d)  3

(e)  None of these

Answer: (b)

95. The price of the article is first increased by 20% and later on the price were decreased by 25% due to reduction in sales. Find the net percentage change in final p rice of article.

(a)  20%

(b)  18%

(c)  38%

(d)  10%

(e)  None of these

Answer: (d)

Directions (Q. Nos. 96-100) What will come in the place of the question marks in the following number series?

96. 48, 23, ? 4.25, 1.125

(a)  10.5

(b)  10

(c)  2.5

(d)  11

(e)  None of these

Answer: (a)

97. 2, 15, 41, 80, 132, ?

(a)  197

(b)  150

(c)  178

(d)  180

(e)  None of these

Answer: (a)

98. ?, 15, 75, 525, 4725, 51975

(a)  5

(b)  10

(c)  8

(d)  6

(e)  None of these

Answer: (a)

99. 4, 19, 49, ?, 229

(a)  75

(b)  109

(c)  65

(d)  169

(e)  None of these

Answer: (b)

100. 840, ?, 420, 140, 35, 7

(a)  408

(b)  840

(c)  480

(d)  804

(e)  None of these

Answer: (b)

SBI Clerical Cadre Phase-I Recruitment Preliminary Online Examination Held on May 29, 2016 Question Paper With Answer Key

SBI Clerical Cadre Phase-I Recruitment Preliminary Online Examination Held on May 29, 2016
SBI Clerical Cadre Phase-I Recruitment Preliminary Online Examination Held on May 29, 2016 Question Paper With Answer Key

SBI Clerical Cadre Phase-I Recruitment Preliminary Online Examination Held on May 29, 2016

Part I English Language

 

Directions (Q. Nos. 1-5) In each of the following questions, a sentence with four words in bold type is given. One of these four words may be either wrongly spelt or inappropriate in the context of the sentence. Find out the word which is wrongly spelt or inappropriate, if any. That word is your answer. If all the words are correctly spelt and also appropriate in the context of the sentence, mark ‘All correct’ as your answer.

1. The Asian Development Bank has supported the government’s latest efforts to deliver quality health services to all sections of society.

(a)  supported

(b)  latest

(c)  deliver

(d)  sections

(e)  All correct

Answer: (e)

2. On the basis of the forecast by the Indian Meteorological Department the government ordered district collectors to visit districts, which are expected to receive heavy

(a)  ordered

(b)  district

(c)  expected

(d)  rainfal

(e)  All correct

Answer: (d)

3. The economy is on the part to recovery and slowly, but surely the banking sector will perform

(a)  part

(b)  recovery

(c)  surely

(d)  perform

(e)  All correct

Answer: (a)

4. During the course of the day many meetings were held but the agenda was not shared with us.

(a)  During

(b)  course

(c)  held

(d)  agenda

(e)  All correct

Answer: (e)

5. Meazures to increase both financial inclusion and investment in infrastructure will result in gains for the country.

(a)  Meazures

(b)  inclusion

(c)  investment

(d)  gains

(e)  All correct

Answer: (a)

Directions (Q. Nos. 6-15) Read the following story carefully and answer the questions given. Certain words/phrases have been given in bold to help you locate them while answering some of the questions.

Once there was a king who had been trying really hard to capture an elephant, but that prize had proved elusive. All the hunters in the kingdom had tried but their efforts were ‘in vain’. So the king announced that anyone who could capture the elephant would get half his kingdom.

The tortoise heard about this and went to the king to accept the challenge. The king was very amused. ‘All my hunters have failed to capture the elephant and you think you can succeed?’, asked the king. The tortoise insisted that he was up to the task and promised to deliver that elephant to the king within forty eight hours. The tortoise the dug a big hole, big enough to hold the elephant along a path leading to the village. Then he covered the hole with sticks and leaves so that it was not visible unless inspected closely. When this was done, the tortoise went in search of the elephant. When the tortoise met the elephant, he told him, ‘You know you are the largest animal in the forest and you should be a king?’ The elephant had never considered this before but the thought it was not a bad idea. The tortoise told the elephant that the villagers had decided to sale the largest animal their king and were all expecting the elephant to come to the village and be crowned their king. The more the elephant heard, the more excited he became.

The tortoise adorned the elephant with colourful beads and beating a gong, he sang songs praising the elephant while he led the way to t he village. Soon they approached the trap and the tortoise being lighter and smaller walked over the trap. The elephant who was following him fell through the sticks and leaves into the deep hole and thus helped the tortoise in achieving his goal well within the time limit.

6. As mentioned in the story, despite the king doubling his ability, the tortoise accepted the challenge

A. he was forced to do s by the villagers.

B. to prove to the king that only a tortoise and his family could complete the challenge.

C. he wanted the entire kingdom to himself.

(a)  Only A

(b)  A and B

(c)  Only B

(d)  Only C

(e)  None of these

Answer: (e)

7. Which of the following correctly explains the meaning of the phrase, ‘in vain’ as used in the story?

(a)  Were painful

(b)  Were successful

(c)  Were in order

(d)  Were untruthful

(e)  Were simple

Answer: (e)

8. Which of the following could be a suitable title to the story?

(a)  The Elephant who Became the Most Popular King

(b)  The Wise King

(c)  The Clever Tortoise

(d)  The Villagers and their Love for the Elephant

(e)  The Selfish Villagers

Answer: (c)

9. Which of the following is most nearly the opposite in meaning as the word ‘Excited’ as used in the story?

(a)  Bored

(b)  Interested

(c)  Affected

(d)  Feeling

(e)  Merciless

Answer: (a)

10. Which of the following statements is true in the context of the story?

(a)  The king refused to share his kingdom with the tortoise.

(b)  The tortoise kept his word by providing the king with what he desired.

(c)  It was the kings idea that helped the tortoise achieve his target.

(d)  The elephant was popular for his beautiful skin.

(e)  None of the given statements is true.

Answer: (b)

11. As mentioned in the story, the elephant followed tortoise’s advice because

A. for him, the tortoise was the wisest animal in the village

B. he really believed that the villagers wanted him as the king

C. he had always wanted to become rich and powerful

(a)  Only A

(b)  A and C

(c)  A and B

(d)  Only B

(e)  All of these

Answer: (d)

12. Which of the following is most nearly the same in meaning as the word ‘Insisted’ as used in the story?

(a)  Requested

(b)  Revived

(c)  Repeated

(d)  Invited

(e)  Showed

Answer: (c)

13. Which of the following is most nearly the opposite in meaning as the word ‘Praising’ as used in the story?

(a)  Insulting

(b)  Concealing

(c)  Hiding

(d)  Refusing

(e)  Removing

Answer: (a)

14. As mentioned in the story, which of the following can be said about the elephant?

(a)  He had good sense of humour.

(b)  He was foolish.

(c)  He was balanced.

(d)  He was logical.

(e)  He was short tempered.

Answer: (b)

15. Which of the following is mot nearly the same in meaning as the word ‘Adorned’ as used in the story?

(a)  Touched

(b)  Adored

(c)  Charmed

(d)  Decorated

(e)  Stuffed

Answer: (d)

Directions (Q. Nos. 16-20) Rearrange the given six sentences/group of sentences, A, B, C, D, E and F in a proper sequences so as to form a meaningful paragraph and then answer the given questions.

A. The next day Shiva appeared at Raghu’s house and spotted a metal chest on the rooftop which had all the money and ornaments that Raghu owned.

B. When Raghu arrived at Shiva’s house, Shiva asked him to guess what fruit it was, saying that if Raghu failed he (Shiva) would carry on thing out of Raghu’s house and if Raghu guessed correctly then Raghu could carry one thing out of Shiva’s house.

C. Seeing him Raghu thanked him saying, ‘Thank you for sparing my wealth and carrying out only the ladder!’

D. One day Shiva hatched a plan to outwit Raghu. He plucked a mango and wrapped it well in a piece of cloth.

E. ‘Oh ho!’ You thought that you’d hide all your wealth from me said Shiva going into Raghu’s house and bringing out a ladder began to climb up onto the roof.

F. But Raghu’s guesses were incorrect.

16. Which of the following should be the FIFTH sentence after the rearrangement?

(a)  A

(b)  B

(c)  D

(d)  E

(e)  F

Answer: (d)

17. Which of the following should be the SECOND sentence after the rearrangement?

(a)  A

(b)  B

(c)  C

(d)  E

(e)  F

Answer: (b)

18. Which of the following should be the THIRD sentence after the rearrangement?

(a)  A

(b)  B

(c)  C

(d)  D

(e)  F

Answer: (e)

19. Which of the following should be the SIXTH (LAST) sentence after the rearrangement?

(a)  A

(b)  B

(c)  C

(d)  D

(e)  E

Answer: (c)

20. Which of the following should be the FIRST sentence after the rearrangement?

(a)  A

(b)  C

(c)  D

(d)  E

(e)  F

Answer: (d)

Directions (Q. Nos. 21-25) Read the following sentences to find out whether there is any grammatical error in them. The error, if any, will be one part of the sentence. Mark the part with the error as your answer. If there is no error, mark ‘No error’ as your answer. (Ignore errors of punctuation, if any)

21. One should always/ be happy from / whatever one has/ and should not be greedy.

(a)  One should always

(b)  be happy from

(c)  whatever one has

(d)  and should not be greedy.

(e)  No error

Answer: (b)

22. My friend lost / his temper on the / shopkeeper and / slapped him.

(a)  My friend lost

(b)  his temper on the

(c)  shopkeeper and

(d)  slapped him.

(e)  No error

Answer: (e)

23. Luxurious and / expensive lights were / imported from / various countries.

(a)  Luxurious and

(b)  expensive lights were

(c)  imported from

(d)  various countries

(e)  No error

Answer: (b)

24. They were unhappy because/ the person who had a palace / had no gold and the person / who had gold had no palace.

(a)  They were unhappy because

(b)  the person who had a palace

(c)  had no gold and the person

(d)  who had gold had no palace

(e)  No error

Answer: (e)

25. Despite his / poor financial condition, / the auto-driver remained/ honest and hard work.

(a)  despite his

(b)  poor financial condition,

(c)  the auto-driver remained

(d)  honest and  hard work

(e)  No error

Answer: (d)

Directions (Q. Nos. 26-30) In the following passage, there are blanks, each of which has been numbered. Against each, five words are suggested, one of which fits the blank appropriately. Find out the appropriate word in each case.

As Tom was climbing only a few feet away from the top of the mountain, he (26) and fell in to the air, falling at great speed. He could only see black spots as he went down. he kept failing, when all of a (27) he felt the rope tied to his waist pull at him very hard. His body was hanging in the air. Only the rope was holding him and in that moment of stillness, he had no other (28) but to scream, ‘Help me God’. All of a sudden a deep voice coming from the sky answered. ‘What did you want me to do?’ ‘Save me God,’ said Tom. ‘Do you really think I can save you?’ Save me God,’ said Tom. ‘Do you really think I can save you?’ asked the voice. ‘Of course you can, ‘(29) Tom. ‘Then cut the rope tied to your waist,’ said the voice. There was a moment of silence as Tom decided to hold on to the rope with ‘all his (30). The rescue team said that the next day a climber was found and frozen, his body hanging from a rope

26.

(a)  stumble

(b)  talked

(c)  tried

(d)  slapped

(e)  caused

Answer: (a)

27.

(a)  time

(b)  instant

(c)  sudden

(d)  shock

(e)  moment

Answer: (c)

28.

(a)  asked

(b)  questioned

(c)  replied

(d)  denied

(e)  doubted

Answer: (a)

29.

(a)  asked

(b)  questioned

(c)  replied

(d)  denied

(e)  doubted

Answer: (c)

30.

(a)  strength

(b)  feet

(c)  family

(d)  reason

(e)  portions

Answer: (a)

Part II Reasoning

31. Among five friends – O, P, Q, R and S, each scored different marks in an examination. P scored more than R and Q, but less than O. O did not score the highest marks and did not score the lowest marks. Who amongst the following scored the third highest?

(a)  Cannot be determined

(b)  S

(c)  P

(d)  Q

(e)  O

Answer: (c)

Directions (Q. Nos. 32-36) Study the following information carefully and answer the given questions.

E, F, G, H, I, J, K and L are sitting around a circular table facing the centre but not necessarily in the same order. Only three people sit between K and L. Only two people sit between L and H. Only three people sit between H and G. J sits second to the right of H. I is one of the immediate neighbours of H. F is neither an immediate neighbour of K nor G.

32. Which amongst the following statements is true regarding F, as per the given arrangement?

(a)  None of the given statements is true.

(b)  K is one of the immediate neighbours of F.

(c)  F sits second to the left of H

(d)  F sits third to the right of J.

(e)  Only three people sit between F and E.

Answer: (c)

33. If all the persons are made to sit in alphabetical order in clockwise direction, starting from E, the positions of how many, excluding E, would remain unchanged?

(a)  Three

(b)  Four

(c)  One

(d)  Two

(e)  None

Answer: (d)

34. Four of the following five are alike in a certain way based on their positions in the given arrangement and so form a group. Which is the one that does not belong to the group?

(a)  JHG

(b)  KIE

(c)  GJL

(d)  HFJ

(e)  ILK

Answer: (c)

35. Who among the following sits to the immediate left of L?

(a)  H

(b)  G

(c)  I

(d)  F

(e)  J

Answer: (b)

36. What is the position of H with respect to E?

(a)  Second to the left

(b)  Second to the right

(c)  To the immediate right

(d)  Third to the left

(e)  Third to the right

Answer: (d)

37. In a certain code language, CLUB is coded as ‘FKTE’ and THEN coded as ‘WGDQ’. In the same code language, ARID will be coded as

(a)  DQHG

(b)  TQHG

(c)  YQKE

(d)  DULC

(e)  DQLC

Answer: (a)

38. In a certain code if ‘a fast train’ is coded as ‘1 5 ‘ and ‘train to Mumbai’ is coded as ‘9 3 1’, then what is the code for ‘train’ in the given code language?

(a)  8

(b)  3

(c)  5

(d)  1

(e)  Either 3 or 9

Answer: (d)

Directions (Q. Nos. 39-43) Study the following information and answer the questions.

Seven people, namely J, K, L, M, N, O and P watch seven different channels on seven different days of the same week starting from Monday and ending on Sunday, not necessarily in the same order. K watches a channel on Saturday. More than two people watches a channel between N and L. J watches a channel on one of the days before L but not on Wednesday. As many people watch a channel between L and P as between J and L. O watches a channel immediately after P.

39. On which of the following days does M watch a channel?

(a)  Friday

(b)  Monday

(c)  Sunday

(d)  Wednesday

(e)  Tuesday

Answer: (c)

40. Which of the following p airs represents the persons who watch a channel immediately before and immediately after K?

(a)  J, P

(b)  M, J

(c)  L, O

(d)  P, L

(e)  O, M

Answer: (e)

41. Which of the following statements is not true as per the given arrangement?

(a)  All the given statements are true.

(b)  Only two people watch a channel after O.

(c)  Only two people watch a channel between M and P.

(d)  N watches a channel immediately before J.

(e)  L watches a channel on Wednesday.

Answer: (a)

42. Who amongst the following watches a channel on Thursday?

(a)  M

(b)  J

(c)  L

(d)  O

(e)  P

Answer: (e)

43. Four of the following five are alike in a certain way and thus form a group as per the given arrangement. Which of the following does not belong to that group?

(a)  OP

(b)  NJ

(c)  LP

(d)  MK

(e)  JK

Answer: (e)

44. If there is added to each even digit and three is subtracted from each odd digit in the number 254673 and then in the new number thus formed all the digits are arranged in descending order, which of the following will be third from the right?

(a)  0

(b)  2

(c)  9

(d)  4

(e)  7

Answer: (d)

Directions (Q. Nos. 45-49) In these questions, relationship between different elements is shown in the statements. The statements are followed by conclusions. Study the conclusions based on the given statements and select the appropriate answer.

Give answer

a. if only conclusion I is true

b. if only conclusion II is true

c. if either conclusion I or II is true

d. if neither conclusion I nor II is true

e. if both conclusions are true

45. Statements F ≤ L < U = K ≥ E

Conclusions I. U ≥ F       II. F < K

Answer: (e)

46. Statements B ≥ O ≥ N < K ≤ R; N ≥ F

Conclusions I. O < R       II. F ≤ B

Answer: (b)

47. Statement C ≤ D = E > F ≥ G

Conclusions I. C ≤ F       II. G ≥ D

Answer: (d)

48. Statements L > A ≥ M > P; R ≤ A ≤ N

Conclusions I. M ≤ N      II. P > R

Answer: (a)

49. Statement P ≤ Q ≤ R = S < T

Conclusions I. P = T        II. P < T

Answer: (b)

50. How many such pairs of letters are there in the word ‘TEACH’ each of which has a many letters between them in the word (in both forward and backward directions) as they have between them in the English alphabetical series?

(a)  More than three

(b)  None

(c)  Two

(d)  Three

(e)  One

Answer: (c)

Directions (Q. Nos. 51-55) Study the following arrangement carefully and answer the questions.

Y B @ 3 E P ⋆ $ 5 T C Z J 4 L 7 R K 8 V ^ F X Q U π M 6 G N % 9

51. Four of the following five are alike in a certain way based on their positions in the given arrangement and hence form a group. Which one does not belong to that group?

(a)  $5⋆

(b)  6GM

(c)  ZLC

(d)  7RL

(e)  BY@

Answer: (e)

52. How many alphabets are there in the English alphabetical series between the sixth element from the left end and the sixth element from the right end of the given arrangement?

(a)  None

(b)  Two

(c)  One

(d)  Three

(e)  More than three

Answer: (b)

53. Which one of the following will come next in the given sequence?

YE3     @⋆P            E5$  ⋆CT   ?

(a)  CL4

(b)  T4J

(c)  54J

(d)  C4Z

(e)  5JZ

Answer: (e)

54. If all the symbols are deleted from the given arrangement then which of the following will be the ninth element from the right end?

(a)  F

(b)  K

(c)  V

(d)  8

(e)  7

Answer: (a)

55. How many such alphabets are there in the given arrangement each of which is immediately preceded by a number as well as immediately followed by a symbol?

(a)  None

(b)  Three

(c)  More than three

(d)  Two

(e)  One

Answer: (e)

Directions (Q. Nos. 56-57) Study the given information carefully to answer the given questions.

Marcus, who is standing at Point D, walks 11 m towards South and reaches Point E. He then takes a left turn and walks 7 m. He then takes a left turn, walks 5 m and reaches Point F. Point R is 18 m to the East of Point F. Point S is 6 m to the North to Point R.

56. In which direction is Point R with respect to Point D?

(a)  South-West

(b)  East

(c)  West

(d)  North

(e)  South-East

Answer: (e)

57. How far and in which direction is Point S with respect to Point D?

(a)  11 m towards East

(b)  11 m towards West

(c)  17 m towards West

(d)  17 m towards East

(e)  25 m towards East

Answer: (e)

Directions (Q. Nos. 58-60) Study the following information and answer the given questions.

S is the father of R. R is the only son of U. U is daughter of J. J has only two children. K is the son of J.

58. If X is the grand-daughter of J, then how is U related to X?

(a)  Sister

(b)  Mother-in-law

(c)  Grandfather

(d)  Sister-in-law

(e)  Uncle

Answer: (e)

59. How is S related to K?

(a)  Brother

(b)  Brother-in-law

(c)  Cousin

(d)  Father

(e)  Uncle

Answer: (b)

60. How is R related to J?

(a)  Grandfather

(b)  Son

(c)  Nephew

(d)  Grandson

(e)  Son-in-law

Answer: (d)

Directions (Q. Nos. 61-65) The following questions are based on the given 3-digit numbers

            821    547    452    935    368

61. The product of the second and the third digit in which of the following numbers is the second highest?

(a)  547

(b)  935

(c)  368

(d)  452

(e)  821

Answer: (a)

62. The difference of the first and the second digits in how many of the given numbers is greater than the third digit of the same number?

(a)  Two

(b)  Three

(c)  More than three

(d)  One

(e)  None

Answer: (a)

63. If all the digits of the given number arranged in ascending order within the number, what will be the sum of the second and third digits of the second highest number thus formed?

(a)  12

(b)  10

(c)  9

(d)  14

(e)  15

Answer: (e)

64. If the first digit and the third digit of all the given numbers are interchanged, which of the following will become the second lowest number after performing the said operation?

(a)  452

(b)  547

(c)  935

(d)  368

(e)  821

Answer: (a)

65. If ‘2’ is added to the second digit of all even numbers and ‘3’ is subtracted from the first digit of all odd numbers, in how many numbers thus formed will a digit appear twice?

(a)  None

(b)  Three

(c)  More than three

(d)  Two

(e)  One

Answer: (e)

Part III Quantitative Aptitude

66. The circumference of circle A is 75 m more than its diameter. If the radius of circle B is 3.5 m more than the radius of circle A, what is the circumference of circle B?

(a)  110 m

(b)  140 m

(c)  163 m

(d)  96 m

(e)  132 m

Answer: (e)

67. A shopkeeper sold an article at 20% discount and earned a profit of 4%. By what percent the marked price of the article more than the cost price of the article?

(a)  20%

(b)  15%

(c)  40%

(d)  25%

(e)  30%

Answer: (e)

Directions (Q. Nos. 68-72) What will come in  place of question marks in the given numbers series?

68. 89   86      78      63      39      ?

(a)  19

(b)  4

(c)  9

(d)  13

(e)  28

Answer: (b)

69. 142   70      34      16      ?        5

(a)  5

(b)  7

(c)  3

(d)  12

(e)  8

Answer: (b)

70. 21   37      2   88.2   94.6   ?

(a)  211.6

(b)  224.5

(c)  192.3

(d)  174.6

(e)  182.8

Answer: (d)

71. 17   9        10      5   35      ?

(a)  84

(b)  102.5

(c)  90

(d)  92.5

(e)  72

Answer: (c)

72. 75   ?        6        36      288    2880

(a)  3

(b)  1.5

(c)  2

(d)  4.5

(e)  5

Answer: (b)

73. Pihu and Rani start a business together by investing Rs 9000 and Rs 6300 respectively. After a certain period, Pihu withdrew from the business completely. If at the end of 2 years, the total profit earned was Rs 13050 and Pihu’s share was Rs 6750, after how many months did Pihu withdraw from business?

(a)  23 months

(b)  14 months

(c)  20 months

(d)  18 months

(e)  12 months

Answer: (d)

74. Jar A contains ‘X’ L of pure milk only. A 27 L mixture of milk and water in the respective ratio of 4 : 5, is added to jar A. The new mixture thus formed in jar A contains 70% milk, what is the value of X?

(a)  23 L

(b)  30 L

(c)  27 L

(d)  48 L

(e)  28 L

Answer: (a)

75. The speed of a boat in still water is 500% more than the speed of the current. What is the respective ratio between the speed of the boat downstream and speed of the boat upstream?

(a)  9 : 2

(b)  7 : 3

(c)  7 : 5

(d)  9 : 4

(e)  4 : 3

Answer: (c)

Directions (Q. Nos. 76-80) Study the table and answer the given questions

76. Number of employees in organizations E increased by, what percent from 2003 to 2006?

(a)  105%

(b)  110%

(c)  115%

(d)  102%

(e)  125%

Answer: (a)

77. Number of employees in organization B and C decreased by 5% and 10% respectively from 2007 to 2008. What was the total number of employees in organization B and C together in 2008?

(a)  177

(b)  179

(c)  175

(d)  181

(e)  163

Answer: (a)

78. What is the difference between total number of employees in organization A in 2003 and 2004 together and that in organization D in the same years together?

(a)  77

(b)  79

(c)  73

(d)  83

(e)  67

Answer: (c)

79. What is the respective ratio between total number of employees in organizations B and C together in 2004 and total number of employees in organizations D and E together in 2007?

(a)  21 : 22

(b)  23 : 29

(c)  23 : 27

(d)  21 : 25

(e)  21 : 23

Answer: (e)

80. What is the average number of employees in organization A, B and E in 2005?

(a)  98

(b)  96

(c)  92

(d)  88

(e)  94

Answer: (e)

81. The average runs of a cricketer in a tournament, in which he played 14 matches, are 47. His average runs in the first seven matches are 57 and that in the last five matches are 44. If the runs made by him in 8th match are 15, how many runs did he make in 9th match?

(a)  24

(b)  32

(c)  26

(d)  22

(e)  28

Answer: (a)

82. An equal amount of sum, Rs P is invested in scheme A and Scheme B. Both the schemes A and B offer simply interest at the rate of 12% and 9% respectively. If at the end of two years total amount received from both the schemes together was Rs 21780, what is the value of P?

(a)  Rs 9000

(b)  Rs 9600

(c)  Rs 12000

(d)  Rs 8400

(e)  Rs 8000

Answer: (a)

Directions (Q. Nos. 83-92) What will come in place of question marks in the given questions?

83. 

(a)  15

(b)  5

(c)  100

(d)  25

(e)  10

Answer: (e)

84. 80 ÷4 × ? = 600

(a)  16

(b)  12

(c)  18

(d)  14

(e)  24

Answer: (c)

85. 57 – 1725 ÷ 69 = 4 × ?

(a)  9

(b)  11

(c)  22

(d)  28

(e)  25

Answer: (e)

86. 

(a)  26

(b)  24

(c)  22

(d)  28

(e)  25

Answer: (a)

87.

(a)  112

(b)  164

(c)  128

(d)  116

(e)  136

Answer: (c)

88. 

(a)  15

(b)  225

(c)  16

(d)  256

(e)  289

Answer: (d)

89. 

(a)  6

(b)  6.4

(c)  8

(d)  4.2

(e)  4

Answer: (e)

90.

(a)  3.1

(b)  2.53

(c)  2

(d)  2.48

(e)  3.6

Answer: (b)

91. 

(a)  18

(b)  324

(c)  14

(d)  144

(e)  196

Answer: (a)

92. 

(a)  2/3

(b)  2/5

(c)  1/3

(d)  3/5

(e)  4/5

Answer: (d)

93. The simple interest (@ 6% per annum) received on a principal of Rs X was Rs 482.40 when invested for 3 years in scheme A. If scheme B offered compound interest (compounded annually) at 10% per annum, what was the interest received by investing Rs ‘X-680’ for 2 years in scheme B?

(a)  Rs 420

(b)  Rs 490

(c)  Rs 530

(d)  Rs 540

(e)  Rs 650

Answer: (a)

94. Mary paid 15% of her monthly salary towards an EMI. From the remaining salary, she paid 10% as internet bill and 20% as rent. If after the mentioned expenses she was left with Rs 24990, what was Mary’s monthly salary?

(a)  Rs 45000

(b)  Rs 48000

(c)  Rs 42000

(d)  Rs 36000

(e)  Rs 40000

Answer: (c)

95. The sum of 4 consecutive even numbers is 107 more than the sum of three consecutive odd numbers. If the sum of smallest odd number and the smallest even number is 55, what is the smallest even number?

(a)  36

(b)  40

(c)  32

(d)  38

(e)  34

Answer: (d)

96. A, B and C, each working alone, can finish a piece of work in 27, 33 and 45 days respectively. A starts by working alone for 12 days, then B takes over from A and works for 11 days. At this stage C takes over from B and completes the remaining work. In how many days the whole work was completed?

(a)  33 days

(b)  31 days

(c)  39 days

(d)  35 days

(e)  37 days

Answer: (a)

97. A rectangular garden of length 12 m is surrounded by a 2 m wide path. If the area of the garden is 84 m2 and the cost of gravelling is Rs 8 per m2, what is the total cost of gravelling the path?

(a)  Rs 780

(b)  Rs 742

(c)  Rs 724

(d)  Rs 775

(e)  Rs 736

Answer: (e)

98. Train A, travelling at S m/sec, can cross a platform double its length in 21 seconds. The same train, travelling at S + 5 m/sec, can cross the same platform in 18 seconds. What is the value of S?

(a)  27.5 m/sec

(b)  32.5 m/sec

(c)  30 m/sec

(d)  35 m/sec

(e)  25 m/sec

Answer: (c)

99. The present age of Charu is 2.5 times the present age of Harsh. Had Harsh been two years younger and Charu been 13 years older, Charu’s age would have been 3.5 times Harsh’s age. What is Harsh’s present age?

(a)  48 years

(b)  25 years

(c)  20 years

(d)  28 years

(e)  36 years

Answer: (c)

100. Divide 27 into two parts so that 5 times the first and 11 times the second together equal to 195. Then ratio of the first and second part is

(a)  17 : 10

(b)  3 : 2

(c)  2 : 7

(d)  5 : 4

(e)  5 : 2

Answer: (a)

© Copyright Entrance India - Engineering and Medical Entrance Exams in India | Website Maintained by Firewall Firm - IT Monteur